Die dunkle Materie existiert jetzt doch nicht. Zu dem Schluß könnte man zumindest kommen, wenn man in letzter Zeit die Berichterstattung in den Medien verfolgt hat. Bonner Astronomen haben Beobachtungen gemacht, die in Widerspruch zur bisherigen Auffassungen der dunklen Materie stehen. Ist die dunkle Materie nun also vom Tisch? Nein, nicht wirklich…


Wie die Astronomen auf die Idee gekommen sind, dass es sowas wie dunkle Materie geben muss, habe ich schonmal in einem eigenen Artikel beschrieben. Die Kurzfassung: Bei der Beobachtung von Galaxien, Galaxienhaufen und Sternhaufen hat man bemerkt, dass sich die Dinge nicht so bewegen wie sie es eigentlich tun sollten. Das kann nun zwei Gründe haben: entweder wir benutzen zur Beschreibung der Bewegung die falschen Formeln oder da ist noch irgendetwas, das wir nicht berücksichtigt haben und das mit seiner Masse die Bewegung der Himmelskörper beeinflusst. Also entweder das Newtonsche Gravitationsgesetz bzw. die Einsteinschen Formeln der Allgemeinen Relativitätstheorie mit denen wir beschreiben, wie sich Objekte unter dem Einfluss der Gravitation bewegen sind falsch. Dann müssen wir neue Gleichungen suchen, die dann hoffentlich alles richtig beschreiben und bei denen die Diskrepanz zwischen Beobachtung und Theorie nicht mehr auftritt. Oder die Gleichungen sind richtig. Dann muss da aber im Universum noch viel mehr Materie existieren, die die Bewegung der Sterne und Galaxien beeinflusst – die wir aber nicht sehen können. Eben “dunkle Materie”.

In den letzten Jahrzehnten wurden beide Ansätze von den Astronomen verfolgt. Die einen beschäftigten sich mit alternativen Gravitationsgesetzen – hier sind z.B. MOND (MOdifizierte Newtonsche Dynamik) oder TeVeS (Tensor-Vektor-Skalar-Gravitationstheorie; eine alternative zur allgemeinen Relativitätstheorie) entstanden. Auf der anderen Seite haben die Astronomen immer genauere “Beobachtungen” der dunklen Materie gemacht. Natürlich nicht direkt – wir können sie ja nicht sehen. Aber man kann auf verschiedenste Weise die Gravitationskraft messen, die auf Objekte im All wirkt und so hat man immer mehr Informationen über die Verteilung der hypothetischen dunklen Materie bekommen. In der Zwischenzeit haben sich Theoretiker Gedanken über die Natur der dunklen Materie gemacht und dabei interessante Ergebnisse erhalten. Heutzutage laufen einige Experimente (u.a. auch am großen Teilchenbeschleuniger LHC) die bisher unbekannte Teilchen finden können – mit vielversprechenden Ergebnissen. Auch aktuelle kosmologische Theorien haben Hinweise auf die Existenz der dunklen Materie geliefert.

Heute geht die große die Mehrheit der Astronomen davon aus, dass die Unterschiede zwischen Beobachtung und Theorie tatsächlich auf die Existenz einer bisher unbekannten Form von Materie zurückzuführen sind. Aber die “Gegenseite” forscht immer noch an alternativen Gravitationstheorien und dazu gehören auch Pavel Kroupa von der Universität Bonn und seine Kollegen. Kürzlich haben sie eine neue Arbeit mit dem Titel “Local-Group tests of dark-matter Concordance Cosmology: Towards a new paradigm for structure formation?” veröffentlicht. Dabei haben sie Beobachtungen gemacht, die bisherigen Vorstellungen über die dunkle Materie widersprechen.

i-39750e38c3c631c6fe21889b52158381-macs_420.jpg
Dunkle Materie im Bullet-Cluster (Bild: X-ray(NASA/CXC/Stanford/S.Allen); Optical/Lensing(NASA/STScI/UC Santa Barbara/M.Bradac)

Zum Beispiel sollten gewissen Galaxien umso heller sein, je mehr dunkle Materie sie enthalten. Denn man geht davon aus, dass sich ursprünglich zuerst die dunkle Materie zu “Klumpen” zusammengefunden hat die dann die normale Materie angezogen haben. Je mehr dunkle Materie desto mehr normale, helle Materie sollte es also geben. Das konnte aber bei Beobachtungen nicht bestätigt werden. Dazu kommen noch andere Gründe die ebenfalls zeigen, dass sich dunkle Materie anscheinend nicht so verhält, wie man gedacht hatte. Ich will das jetzt nicht im Detail erklären – ich bin auch kein Experte für dunkle Materie und möchte hier nicht noch mehr Verwirrung stiften als sowieso schon herrscht.

Eines ist aber klar – die dunkle Materie ist jetzt wegen dieser Arbeit noch lange nicht vom Tisch. Wir wissen einfach noch nicht genug. Wir wissen nun, dass bestimmte Modelle der dunklen Materie (es gibt ja viele verschiedene) in bestimmten Situationen nicht funktionieren. Aber das ist jetzt nicht so tragisch, wie es klingt. Und wenn Pavel Kroupa sagt:

1 / 2 / 3 / Auf einer Seite lesen

Kommentare (338)

  1. #1 rauskucker
    15. Juni 2010

    Ich vermute, man wird auf einen ganz anderen Zusammenhang stoßen, der das erklärt, was wir jetzt noch “dunkle Materie” nennen. Und zwar geht es um die Rotation der zentralen schwarzen Löcher in den Galaxien.
    Kannst du mir erklären, wo deren Drehimpuls bleibt? Und warum sich alle davor drücken, darüber nachzudenken, wie schnell so ein ZSL sich drehen könnte?

    Wenn so ein ZSL eine größere Ausdehnung hat als ein Punkt, muß seine Materie sich mit Überlichtgeschwindigkeit um den Mittelpunkt drehen. Und daraus entsteht eine Fernwirkung, die anders ist als normale Gravitation. Keine Ahnung, wie. Aber irgendeiner wird das irgendwann berechnen und dafür den Nobelpreis kriegen.

    Vermutlich wird man dann auch verstehen, warum manche Galaxien keine Spiralen (mehr) bilden: weil da mehrere ZSL sich gegenseitig ausgebremst haben.

  2. #2 perk
    15. Juni 2010

    ich weiß nicht wie kroupa auf das kommt:
    “Jede einzelne dieser Beobachtungen stellt das Dunkle-Materie-Modell vor Probleme Zusammengenommen kollidieren sie so stark mit der Theorie, dass diese nicht mehr zu halten scheint. Wir müssen uns auf die Suche nach Alternativen machen.”
    genau das steht ja im paper nicht drin..

    im paper steht dass noch viel genauer beobachtet werden muss.. vor allem die bulgemass-satellite relation auch an anderen galaxien zu überprüfen

    außerdem meinen sie dass die modelle innerhalb der theorie um die dm noch verbessert werden müssen

    On the theoretical side, more inclusive
    modelling will still be needed to address these challenges
    within the CCM[Concordance Cosmological Model] while, at the same time, existing viable alternatives
    will have to be further explored with some emphasis.

    das klingt schon wieder ganz anders als nicht mehr zu halten…

    was mich an dem paper am meisten gewundert hat ist dass sie die abstände zwischen den daten so definiert haben dass sie das maximum nehmen (zwischen einer glatten kurve aus nem theoretischen modell und einer anzahl von messwerten)

    das führt zb bei figure 2 zu der absurden situation dass bei 20 messwerten der größte statistische ausreißer zur grundlage für die beurteilung genommen wird.. ich kann aus der formulierung im paper auch nicht erkennen wie diese willkürliche entscheidung motiviert sein soll..

    warum nicht die messwerte mit dem ergebnis der dm-modelle fitten und die prozentuale abweichung der fitparameter von denen im ccm framework angeben..
    das dürfte sowohl genauer als auch theorierelevanter sein.. so wie mir die kurve aussieht wird es da aber für die dm nicht mehr mit 99,7 % schlecht aussehen sondern vllt noch mit 30-50% was bei so großen fehlermargen und so wenig daten nicht wirklich schlecht wäre…

    ich warte gespannt auf antwortpapers

  3. #3 JB
    15. Juni 2010

    Am schlimmsten sind die Arbeitsgruppen die über die MONDler lachen … das ist einfach unwissenschaftlich. Solang es keine Gegenbeweise gibt, soll alles verfolgt werden können.

    Hier sieht man wie Unwissenschaftlichkeit (Mode) Fortschritt hindern kann:

  4. #4 H.M.Voynich
    15. Juni 2010

    MOND scheint speziell darauf konzipiert zu sein, die Rotationsgeschwindigkeit in Relation zum Abstand des Galaktischen Zentrums zu erklären.
    Wie gut funktioniert MOND mit anderen Daten, die man der Dunklen Materie zuschreibt, insbesondere auf viel größeren Skalen?

  5. #5 Pheromonik
    15. Juni 2010

    @rauskucker: Ohne jetzt im entferntesten Experte zu sein: Ein Schwarzes Loch ist kein Punkt. Der Abstand zwischen den Materieteilchen ist auf 0 gesunken, aber die Elementarteilchen selber haben immernoch eine Ausdehnung(also Elektronen, Quarks etc… oder die noch darunter liegenden Elementarteilchen, falls die genannten unechte wären). Von daher ist eine Überlichtgeschwindigkeit nicht von Nöten. Im Übrigen hat man Schwarzen Löchern schon längst einen Drehimpuls zugeordnet (gemessen natürlich noch nicht).
    Mal davon abgesehen würden “gegeneinander laufende Drehimpulse” keine wie auch immer geartete Fernwirkung hervorrufen. Gravitation lässt sich durch Drehimpuls nicht beeinflussen und elektromagnetische Kräfte wirken nur auf geladene Körper.

  6. #6 BerndB
    15. Juni 2010

    @Pheromonik: Aber die Rotation eines Körpers verursacht eine Verdrillung des Raumes um den Körper. Aber ob sich Millionen von Lichtjahren entfernte Galaxien da nun beeinflussen wage ich natürlich auch zu bezweifeln. Andererseits reihen diese sich ja nun auch auf den Rändern der großen Blasen auf.

    Ich weiß jetzt natürlich nicht, ob man wirklich schon alle Effekte der Relativitätstheorie kennt. Zumal wir ja noch nicht einmal mit Sicherheit wissen, in wievielen Dimensionen wir leben. Also könnte es durchaus sein, dass wir wirklich noch Effekte finden, die beides möglich machen. Sowohl eine abgewandelte Gravitation bei großen rotierenden “Körpern” wie Galaxien und eine neue exotische Materie.

    Es dürfte somit noch einiges geben, was wir noch nicht wissen und was unseren Verstand womöglich überfordert (rein Vorstellungs-, nicht rechenmäßig).

  7. #7 Walter Orlov
    15. Juni 2010

    Eigentlich sehe ich überhaupt keine Widersprüche. Man berichtet lediglich über das Scheitern von den Computer-Modellen der Entwicklung des Universums nach dem angeblichen Urknall. Sonst nichts.

  8. #8 Reset
    15. Juni 2010

    Diesen Artikel find ich sehr schön und ausgewogen. Löwenhoek hat einaml gesagt:
    “Ach, ich bin viel zu wenig Forscher, noch bin ich zu sehr damit beschäftigt meine Theorien zu beweisen, statt das ich mich leiten lasse, wohin mich meine Ergebnisse führen. Wissenschaft muss aufgeschlossen sein gegen mich.”

  9. #9 KommentarAbo
    15. Juni 2010

    ….

  10. #10 nihil jie
    15. Juni 2010

    aber wenn das sich erweisen sollte, dass das team der bonner astronomen recht hat… wie kann man sich dann die notwendigen modifikationen der newtonschen und einsteinschen gleichungen vorstellen ? nimmt die gravitationskraft dann zb. nicht mehr mit dem quadrat der entfernung ab, oder welchen satz an variablen muss dann modifiziert werden ?

  11. #11 Frank Wappler
    15. Juni 2010

    Florian Freistetter schrieb (im Artikel):
    > Kurzfassung: Bei der Beobachtung von Galaxien, Galaxienhaufen und Sternhaufen hat man bemerkt, dass sich die Dinge nicht so bewegen wie sie es eigentlich tun sollten.

    Oder etwas weniger verkürzt:
    Man hat durch Auswertung von Beobachtungen bestimmte Bewegungen ermittelt, dann bestimmte Erwartungen zur wahrscheinlichen weiteren Bewegung erhalten, und schließlich festgestellt, dass die tatsächlich ermittelten weiteren Bewegungen nicht den Erwartungen entsprachen.

    > Das kann nun zwei Gründe haben: entweder wir benutzen zur Beschreibung der Bewegung die falschen Formeln oder da ist noch irgendetwas, das wir nicht berücksichtigt haben und das mit seiner Masse die Bewegung der Himmelskörper beeinflusst.

    Das suggeriert, es gäbe „richtige Formeln“ aus denen sich vollkommen zuverlässige Erwartungen über Bewegungen gewinnen ließen, oder die zumindest geeignet wären, das, was bislang noch nicht berücksichtigt wurde, aufgrund der ermittelten Bewegungen immer genauer einzugrenzen.

    Ob es derartige „richtige Formeln“ überhaupt geben kann, hängt allerdings nicht zuletzt von den Definitionen von „Bewegung“, „Masse“ usw. als Messgrößen ab, d.h. davon, wie man aus gegebenen Beobachtungsdaten auf die entsprechenden Messwerte schließt.

    > Wir wissen auch, dass die allgemeine Relativitätstheorie unter gewissen extremen Bedingungen nicht mehr funktioniert

    Jedenfalls sollte man in der Beurteilung der RT sorgfältig (wie immer) zwischen „kinematischem Teil“, „dynamischem Teil“, und darauf beruhenden Modellen (der Kosmologie, der Astromonomie, …) unterscheiden.

    Ein bestimmtes Modell kann richtig oder falsch sein, wie sich durch Vergleich mit Messwerten entscheiden lässt;
    eine bestimmte Theorie (insbeondere einschließlich der Definitionen von Messgrößen, deren Werte in gewissen Formeln auftauchen) ist allenfalls danach zu beurteilen, ob sie nachvollziehbar und zur Erstellung falsifizierbarer und relevanter Modelle zu gebrauchen ist.

  12. #12 Bullet
    15. Juni 2010

    @nihil jie: es ist denkbar, daß irgendwo in den Gleichungen plötzlich Terme auftauchen, die vorher nicht “da waren”, weil sie =1 waren. Wenn sich jetzt herausstellen sollte, daß das, was wir bisher beobachtet haben, nur für den Spezialfall “1” gelten, dann können andere nicht so spezielle Fälle

    • Konstanten mit anderen Werten (z.B. “2”)
    • Variablen mit treffenden Koeffizienten (x², denn das ist für x=1 ja auch “1”)
      oder auch
    • beliebig komplizierte Terme sein (die für einen ganz bestimmten Parameter gerade “1” ergeben).

      Und dann hast du plötzlich eine ganz andere Funktion.

  13. #13 Florian Freistetter
    15. Juni 2010

    @nihil jie: Wie die MOND-Gleichungen aussehen kann man hier sehen: https://de.wikipedia.org/wiki/Modifizierte_Newtonsche_Dynamik

  14. #14 Walter Orlov
    15. Juni 2010

    Erst sagt Professor Dr. Pavel Kroupa: “Wir haben untersucht, wie sich die Vorhersagen der Dunklen-Materie-Theorie mit tatsächlichen Beobachtungsdaten decken”, dann erfahren wir um welche Theorie es eigentlich geht: “Der Theorie zufolge entstanden nach dem Urknall zunächst Klumpen Dunkler Materie.”
    https://www3.uni-bonn.de/Pressemitteilungen/158-2010

    Das ist aber keine Theorie der Dunklen Materie, sondern des Urknalls! Warum sieht ja keiner diese Unterschiebung?!! Ja, er ist ein Professor… soll das auch heißen, dass man auch sofort eigenen Denkapparat abschalten muss?

  15. #15 Bullet
    15. Juni 2010

    Wo ist da jetzt das Problem?

  16. #16 Florian Freistetter
    15. Juni 2010

    @Orlov: “Das ist aber keine Theorie der Dunklen Materie, sondern des Urknalls! Warum sieht ja keiner diese Unterschiebung?!!”

    Nur weil das Wort “Urknall” irgendwo vorkommt muss es sich noch lange nicht um eine Urknalltheorie handeln. Dunkle Materie hat sich nach dem Urknall zu Klumpen angeordnet. Und? Wo ist das Problem. Sie gehören wahrscheinlich zu den Leuten, die meinen, die Kosmologen hätten keine Ahnung und vermutlich haben sie auch eine eigene “Theorie” die alles viel besser erklärt, nicht wahr?

  17. #17 Eddi Blitzfrosch
    15. Juni 2010

    Hi,hi es sind die Morphischen Felder die alles zusammenhalten ^^

  18. #18 nihil jie
    15. Juni 2010

    thenx… an Bullet & Florian 🙂 jetzt fange ich an einiges besser zu verstehen… auch die mathematik da zu ist für mich auch gut überschaubar 😉
    aber ich glaube im allgemeinen, ich sollte mal wieder ein paar gleichungen täglich lösen um wieder bisschen mehr übung drin zu haben… *lach man merkt gar nicht wie einem manches wissen schnell entweichen kann nutzt man es nicht öffters. tja… und schon wieder so ein laster des gehirns… genau wie das erkennen von freiliegenden schädeln auf dem mars 🙂

    so.. ich muss noch was tun 🙁 wünsche euch allen noch einen schönen tag 😉

  19. #19 Walter Orlov
    15. Juni 2010

    Nein, hier geht es nicht nur um ein Wort. Es ist ganzes Konzept. In thermischen Fluktuationen der Hintergrundstrahlung erraten die Urknall-Theoretiker die Teeblätter, ups, Entschuldigung, die Strukturen, die auf bestimmtes Verhältnis zwischen Dunkler Materie und normaler Materie weisen. Selbstverständlich gehen sie davon aus, dass die Dunkle Materie prinzipiell etwas anders ist, als normale Materie, unentdeckte Teilchen zum Beispiel. Sie erraten sogar, genau zum welchen Zeitpunkt sie zu existieren begann… Man führt entsprechende Computer-Simulationen durch und bekommt man ganz anderes Universum… Also, schließen manche, es gibt keine Dunkle Materie… Man kann aber auch genauso gut behauptet, dass es eben mit der Urknall-Theorie etwas nicht stimmt. Und das ist eher der Fall, denn die Existenz Dunkler Materie durch viele Beobachtungen, ich betone, nicht durch Computer-Simulationen und Erraten die Teeblätter, sondern durch reale Beobachtungen, belegt ist.

  20. #20 Bjoern
    15. Juni 2010

    @Orlov:

    In thermischen Fluktuationen der Hintergrundstrahlung erraten die Urknall-Theoretiker die Teeblätter, ups, Entschuldigung, die Strukturen, die auf bestimmtes Verhältnis zwischen Dunkler Materie und normaler Materie weisen.

    “erraten”?!? Haben Sie sich auch nur ein einziges Mal die Mathematik angeschaut, die da dahinter steckt? Haben Sie sich auch nur ein einziges Mal die zahlreichen Konsistenzchecks angeschaut, die da gemacht werden, um Fehler zu vermeiden?

    Selbstverständlich gehen sie davon aus, dass die Dunkle Materie prinzipiell etwas anders ist, als normale Materie, unentdeckte Teilchen zum Beispiel.

    Sie gehen davon aus, weil die Beobachtungsdaten stark darauf hinweisen!

    Sie erraten sogar, genau zum welchen Zeitpunkt sie zu existieren begann…

    Äh, wovon reden Sie?

    Man führt entsprechende Computer-Simulationen durch und bekommt man ganz anderes Universum…

    Äh, wovon reden Sie?

    Man kann aber auch genauso gut behauptet, dass es eben mit der Urknall-Theorie etwas nicht stimmt.

    Wenn’s Ihnen gefällt, können Sie das. Wenn Sie dann keiner mehr ernst nimmt, weil Sie offensichtlich die zahlreichen Belege für die Theorie nicht kennen, ist das Ihr Problem.

    Und das ist eher der Fall, denn die Existenz Dunkler Materie durch viele Beobachtungen, ich betone, nicht durch Computer-Simulationen und Erraten die Teeblätter, sondern durch reale Beobachtungen, belegt ist.

    Der Satz ergibt zwar sprachlich und grammatikalisch nicht viel Sinn, aber anscheinend bemängeln Sie, dass die Dunkle Materie noch nicht direkt beobachtet wurde. Dazu kann ich nur sagen: (1) könnte vielleicht daran liegen, dass die entsprechenden Teilchen sehr schwer sind und kaum wechselwirken – schon mal was von WIMPs gehört? Vorschlag: warten wir doch die Ergebnisse vom LHC ab – wenn der keine Neutralinos findet, dann werden sicher die meisten Physiker zugeben, dass es mit der Idee der Dunklen Materie wohl nicht klappt! (2) kleiner Tipp: Quarks wurden auch noch nicht direkt beobachtet – bezweifeln Sie deren Existenz auch? (wenn man’s genau nimmt, wurden noch nicht mal Atome direkt beobachtet…)

  21. #21 Marcel
    15. Juni 2010

    Hallo Florian,

    Schoen dass du der Studien einen Artikel widmest, noch dazu einen so ausgewogenen. Du hast recht, die Ergebnisse schliessen nicht jede Form dunkler Materie aus. Zum Beispiel koennte es ja heisse Dunkle Materie (in Form von z.B. Neutrinos) geben. Aber die Probleme der vorherrschenden Theorie scheinen tatsaechlich so weitgehend, dass eine kleine Modifikation hier und da vermutlich nicht mehr ausreicht.

    Sobald ich zwischendurch die Zeit finde werde dann versuchen auf die Fragen einzugehen, sofern sie nicht schon beantwortet wurden. Ich behalte mir aber vor die Crackpots zu ignorieren, ok? Das ist das schoene wenn sie nicht in meinem Blog auftauchen. 🙂

  22. #22 nihil jie
    15. Juni 2010

    @Marcel

    Ich behalte mir aber vor die Crackpots zu ignorieren, ok? Das ist das schoene wenn sie nicht in meinem Blog auftauchen. 🙂

    na… ob das jetzt nicht bereits der erste fehler war… *lach manch einer wird das sicherlich als eine art provokante einladung verstehen 😉

  23. #23 Walter Orlov
    15. Juni 2010

    Ja, die Wissenschaftler bemühen sich sehr um die Urknall-Theorie. Viel Mathematik ist aber nicht immer gleich physikalischer Sinn. Es gibt doch noch andere Deutungen der Fluktuationen von der Hintergrundstrahlung:

    “Verschuur hatte die im Rahmen mehrerer Radio-Durchmusterungen des Himmels gemessene Verteilung des Wasserstoff in der Galaxis mit den Temperaturschwankungen in den von dem Satelliten WMAP gelieferten Karten der Hintergrundstrahlung verglichen.

    “Bei diesem Vergleich bin ich auf hunderte von Fällen gestoßen, bei denen die Strukturen der Hintergrundstrahlung und die Verteilung des Wasserstoffs sich auffällig ähneln”, so Verschuur.”
    https://www.astronews.com/news/artikel/2007/11/0711-039.shtml

    Ich meine, eines Tages wird auf einmal klar, dass all diese Bemühungen um die tiefere Bedeutung der Fluktuationen Hintergrundstrahlung umsonst waren.

    Ihre äh’s kann ich aber leider nicht verstehen. Ich habe an diesen Stellen lediglich die Medienberichte wiederholt und Sie ähen… Ich bin also ratlos.

    Und eigentlich bezweifele ich die Existenz der Dunklen Materie gar nicht 😉

  24. #24 Marcel
    15. Juni 2010

    So, jetzt endlich ein Schwung Antworten:

    @ perk: Zunächst einmal muss natürlich immer noch mehr beobachtet werden. Die Untersuchung von Satellitenanzahl und Bulgemasse hat bisher, meines Wissens nach, noch niemand anderes durchgeführt. Dass wir da mehr Daten brauchen ist klar. Davon abgesehen, auf dem gesamten Gebiet werden in den nächsten Jahren viele neue Ergebnisse zu erwarten sein. Dennoch kann man aus den bestehenden Daten bereits deutliche Probleme ablesen.
    Zur fig. 2: Du gehst, so schein mir, von einer falschen Vorstellung unserer Methodik aus. Wir führen, wie im Text beschrieben, einen KS-Test durch. Daher wählen wir nicht “willkürlich” den größten Abstand, sondern dies wird vom statistischen Verfahren gefordert. Steht übrigens auch im Paper. Ausserdem handelt es sich um eine kumulative Verteilungsfunktion, nicht um einen “einzelnen statistischen Ausreisser”.

    @ H.M. Voynich: Die Stärke von MOND liegt definitiv im Bereich von Galaxien. Nicht nur die grobe Form der Rotationskurven lässt sich erklären. So scheint die Dunkle Materie zum Beispiel sehr eng der Verteilung der sichtbaren Materie zu folgen, was sich durch MOND ganz natürlich ergibt. Ferner liefert MOND z.B. die Tully-Fisher Relation (Zusammenhang von aus Galaxienleuchtkraft berechneter Masse und Rotationsgeschwindigkeit), oder die bevorzugte Flächendichte, die in Galaxien beobachtet wird. Auf ganz grossen Skalen funktioniert MOND momentan tatsächlich nicht genauso gut. Die Untersuchung von MOND ist auch etwas schwieriger. Das liegt zum einen daran, dass Simulationsrechnungen in MOND komplizierter sind und dafür auch nicht so ausgereifte Codes existieren und einfach länger dauern. Aber der Bullet-Cluster, zum Beispiel, lässt sich auch in MOND verstehen. Man benoetigt dann aber wohl Neutrinos, die nicht masselos sind.

    @Frank Wappler: Man kann das natürlich alles genauer erläutern und schliesslich sogar auf eine philosophische Grundsatzdiskussion herunter führen. Ich denke aber Florian hat die Situation recht treffend zusammengefasst.

    @nihil jie: Hm… ich fürchte du hast recht. Was stösst du sie aber auch noch auf diese Interpretation ;-). Egal, sollen sie doch kommen. Wären nicht die ersten.

    Ich hoffe ich habe die Fragen, auf die noch niemand eingegangen ist beantwortet. Sollte sich jemand übergangen fühlen dann melde er/sie sich. 🙂

  25. #25 miesepeter3
    15. Juni 2010

    @Florian Freistetter

    Es gibt wahrscheinlich schwarze Materie. Es gibt schwarze Materie!
    Wahrscheinlich gibt es doch keine schwarze Materie.
    Es hat nie schwarze Materie gegeben!
    Das liebe ich ich so an der Wissenschaft: Science by potatoes.
    Auf norddeutsch : Rin in de Kartüffel, ruut ut de Kartüffel.
    Weiter so.

  26. #26 Bjoern
    15. Juni 2010

    @Orlov:

    Viel Mathematik ist aber nicht immer gleich physikalischer Sinn.

    In der Tat, offensichtlich nicht immer. Tipp: deshalb vergleicht man ja auch die mathematischen Ergebnisse mit den Beobachtungsdaten – und, Überraschung: es passt!

    “Verschuur hatte die im Rahmen mehrerer Radio-Durchmusterungen des Himmels gemessene Verteilung des Wasserstoff in der Galaxis mit den Temperaturschwankungen in den von dem Satelliten WMAP gelieferten Karten der Hintergrundstrahlung verglichen. “Bei diesem Vergleich bin ich auf hunderte von Fällen gestoßen, bei denen die Strukturen der Hintergrundstrahlung und die Verteilung des Wasserstoffs sich auffällig ähneln”, so Verschuur.”
    https://www.astronews.com/news/artikel/2007/11/0711-039.shtml

    Äh, dass dieser Artikel gleich selbst (in den letzten beiden Absätzen) sehr überzeugende Gegenargumente liefert, ist Ihnen nicht aufgefallen? Oder wollten Sie’s nicht sehen? Oder finden Sie die Gegenargumente nicht überzeugend? Falls es letzteres ist, dann erklären Sie doch mal, warum Sie Verschuurs Behauptungen überzeugender finden als die Gegenargumente… (vor allem das im letzten Absatz!)

    Ihre äh’s kann ich aber leider nicht verstehen. Ich habe an diesen Stellen lediglich die Medienberichte wiederholt und Sie ähen… Ich bin also ratlos.

    Wie wär’s denn, wenn Sie mir einfach sagen, wo ich die entsprechenden Medienberichte finden kann? Ich habe jedenfalls von dem, was Sie da behaupten, noch nichts gehört.

    Und eigentlich bezweifele ich die Existenz der Dunklen Materie gar nicht 😉

    Das klang oben aber noch anders – vor allem Ihr letzter Satz um 14:20 Uhr…

  27. #27 Bjoern
    15. Juni 2010

    @Marcel: Danke für die Informationen!

    Die Stärke von MOND liegt definitiv im Bereich von Galaxien. Nicht nur die grobe Form der Rotationskurven lässt sich erklären. So scheint die Dunkle Materie zum Beispiel sehr eng der Verteilung der sichtbaren Materie zu folgen, was sich durch MOND ganz natürlich ergibt. Ferner liefert MOND z.B. die Tully-Fisher Relation (Zusammenhang von aus Galaxienleuchtkraft berechneter Masse und Rotationsgeschwindigkeit), oder die bevorzugte Flächendichte, die in Galaxien beobachtet wird. Auf ganz grossen Skalen funktioniert MOND momentan tatsächlich nicht genauso gut. Die Untersuchung von MOND ist auch etwas schwieriger. Das liegt zum einen daran, dass Simulationsrechnungen in MOND komplizierter sind und dafür auch nicht so ausgereifte Codes existieren und einfach länger dauern. Aber der Bullet-Cluster, zum Beispiel, lässt sich auch in MOND verstehen. Man benoetigt dann aber wohl Neutrinos, die nicht masselos sind.

    Das klingt jetzt wirklich so, als wäre ein Modell mit MOND *und* dunkler Materie sinnvoll… na ja, warten wir mal die Daten der nächsten Jahre ab.

    Off topic: wenn du dich mit Satellitengalaxien beschäftigst – kennst du vielleicht Kristin Riebe (ehemals Warnick) vom astrophysikalischen Institut der Uni Potsdam?

  28. #28 Marcel
    15. Juni 2010

    @ Bjoern: Ich habe hier eine schoene Einleitung in den Simulationscode SUPERBOX von Kirsten herumliegen. Hat mir am Anfang meiner Diplomarbeit sehr geholfen. Sie persoenlich habe ich aber leider noch nicht kennen gelernt.

  29. #29 Walter Orlov
    15. Juni 2010

    Bjoern:
    (vor allem das im letzten Absatz!)

    “wenig glaubhaft” – soll dies etwa ein Gegenargument bedeuten?.. Aber Charles Bennett hat sich auf diese Weise verraten – der Urknall ist tatsächlich nicht zuletzt eine Glaubenssache 🙂

  30. #30 Bjoern
    15. Juni 2010

    @Walter Orlov: “wenig glaubhaft” ist hier offensichtlich eine Umschreibung für “sehr unwahrscheinlich” – es ist extrem unwahrscheinlich, dass die WMAP-Daten so konsistent mit allen möglichen anderen Beobachtungsdaten wären, wenn das ganze nur H-Wolken in unserer Milchstrasse wären! Haben Sie das Argument wirklich nicht verstanden, oder stellen Sie es absichtlich falsch dar?

  31. #31 Bullet
    15. Juni 2010

    Ich störe ja nur ungern … aber wenn tatsächlich Wasserstoffwolken die Messungen dominieren würde … müßten dann nicht die WMAP-Daten in der Gegend von M31 einen krassen Peak zeigen? Immerhin überlagern sich an dieser Stelle des Himmels ja dann die Wolken der Milchstraße mit denen in M31. Und daß deren Strahlung detektiert werden können, halte ich erstmal für trivial.
    Oder?

  32. #32 Bjoern
    15. Juni 2010

    @Bullet: Ich würde vermuten, M31 ist so weit weg, dass die Intensität der Strahlung von deren Wasserstoffwolken zu gering ist. Aber ob die geringer ist als die Intensität der kosmischen Hintergrundstrahlung, weiss ich nicht. (könnte man sicher rauskriegen, würde aber eine ganze Menge Arbeit erfordern… bin ich echt grad zu faul dazu)

  33. #33 Walter Orlov
    15. Juni 2010

    Welche “anderen Beobachtungsdaten”? Wie wir gerade erfahren haben, passt Dunkle Materie in das Urknall-Modell nicht mehr.

  34. #34 Bjoern
    15. Juni 2010

    @Orlov:

    Welche “anderen Beobachtungsdaten”?

    Fangen Sie mal hier an: https://www.talkorigins.org/faqs/astronomy/bigbang.html

    Wie wir gerade erfahren haben, passt Dunkle Materie in das Urknall-Modell nicht mehr.

    Das ist eine grob falsche Darstellung / Interpretation dessen, was Florian in diesem Artikel ausführlich erläutert hat. Lesen Sie ihn vielleicht einfach noch mal…

  35. #35 Walter Orlov
    15. Juni 2010

    Fangen Sie mal hier an: https://www.talkorigins.org/faqs/astronomy/bigbang.html

    Gut, dass es in Englisch verfasst ist, sonst fühlte ich mich wirklich verpflichtet, dies zu lesen ;D

    Lesen Sie ihn vielleicht einfach noch mal…

    Meine Meinung habe ich bereits geäußert: Herr Professor hat den Spieß umgedreht. Um vom Verfall der Urknalltheorie abzulenken, greift er die Dunkle Materie an.

  36. #36 H.M.Voynich
    16. Juni 2010

    @Orlov:
    Ja, und Sie haben auch deutlich gemacht, auf welcher Wissensbasis Sie Ihre Meinung gebildet haben.
    Sie sind froh darüber, die Hintergrundinformationen, die Sie selbst erfragt haben, nicht lesen zu müssen. Hut ab, das nenn ich konsequent …

  37. #37 Bullet
    16. Juni 2010

    @Bjoern: also wenns beispielsweise um H-Alpha geht, scheint Entfernung nicht so die Rolle zu spielen … siehe hier:
    https://antwrp.gsfc.nasa.gov/apod/ap100611.html
    Und seds.org sagt zu M51 “Distance: 37 000 kly”
    Aber WMAP war auf Mikrowellen aus, wenn ich mich recht erinnere. Und die Sensorik war eine zart andere.
    Hm.

  38. #38 Walter Orlov
    16. Juni 2010

    Jetzt verstehe: Herr Professor hat nun sehr große “Wissensbasis”, deswegen frisst man alles aus seiner Hand. Jetzt darf ich mich nicht mehr wundern und kann endlich ruhig schlafen. Danke für den Tipp.

    P.S.: Aber vielleicht sollen Sie Ihre Vorstellung über die “Unfehlbarkeit” von den Professoren doch überdenken: https://www.walter-orlov.wg.am/buch/Luege_als_Beweis.pdf

  39. #39 Bullet
    16. Juni 2010

    @Orlov:
    1) welcher Professor ist hier gemeint?
    2) “Gut, dass es in Englisch verfasst ist, sonst fühlte ich mich wirklich verpflichtet, dies zu lesen ;D
    Nanu? Dipl.Ing. und keine Englischkenntnisse?

  40. #40 cydonia
    16. Juni 2010

    Und da kommt Bremer II……..Ich bin weg!

  41. #41 Walter Orlov
    16. Juni 2010

    welcher Professor ist hier gemeint?

    Pavel Kroupa. Er behauptet doch, dass die “Dunkle Materie-Theorie” nicht hält.

    Nanu? Dipl.Ing. und keine Englischkenntnisse?

    Erwischt 🙂 Ich mag einfach nicht, wenn ich weggeschubst werde: “Lies dies und das und lass uns lieber in Ruhe!”

  42. #42 Tim
    16. Juni 2010

    Das Internet ist voll mit Typen ala Bremer, Orlov etc. Scheint eine Krankheit zu sein. 🙂

  43. #43 Bjoern
    16. Juni 2010

    @Orlov: Also das nenne ich mal unverschämt – Sie fragen nach Beobachtungsdaten, und wenn ich Ihnen sage, wo Sie diese nachlesen können, kommen Sie nur mit der faulen Ausrede, es wäre ja Englisch! Ich hätte von der betreffenden Webseite auch eine deutsche Version vorliegen, aber leider nicht online – wenn Sie mir sagen, wie ich Sie erreichen kann, schicke ich Sie Ihnen gerne per Mail… (so, und jetzt kommt irgendeine Ausrede in der Art “meine Mailadresse gebe ich nicht raus” oder so – wetten?).

    Meine Meinung habe ich bereits geäußert: Herr Professor hat den Spieß umgedreht. Um vom Verfall der Urknalltheorie abzulenken, greift er die Dunkle Materie an.

    Und damit haben Sie schön gezeigt, dass Sie (1) keine Ahnung von den Massen an Belegen für die Theorie haben (und inzwischen auch gezeigt, dass Sie kein Interesse haben, die Belege überhaupt kennen zu lernen), und (2) den Artikel von Florian (seit wann ist der “Professor”?) nicht kapiert haben.

    P.S.: Aber vielleicht sollen Sie Ihre Vorstellung über die “Unfehlbarkeit” von den Professoren doch überdenken: https://www.walter-orlov.wg.am/buch/Luege_als_Beweis.pdf

    Ein Video vom P.M.-Magazin? Na, da wundert’s mich aber auch nicht, dass da nichts vernünftiges drin vorkommt. Versuchen Sie’s doch mal mit einer vernünftigen Fachzeitschrift wie z. B. Science oder Nature!

  44. #44 Bjoern
    16. Juni 2010

    @Orlov:

    Erwischt 🙂 Ich mag einfach nicht, wenn ich weggeschubst werde: “Lies dies und das und lass uns lieber in Ruhe!”

    Aha. Sie erwarten also, dass wir Ihnen jede Einzelheit mit dem Löffel einflössen, und wenn wir das nicht tun, sondern Ihnen einfach sagen, wo Sie’s ausführlich nachlesen kannst, spielen Sie die beleidigte Leberwurst? Der Sinn eines FAQ ist Ihnen aber schon klar, oder…?

  45. #45 Florian Freistetter
    16. Juni 2010

    @Orlov: Ich sage ihnen, was sie hier tun können: über dunkle Materie diskutieren. Und ich sage ihnen, was sie NICHT tun können: eine “Die Wissenschaftler lügen alle!! Der Urknall ist falsch!!! Ich allein habe eine viel bessere Theorie!!!!”-Diskussion führen. Der Urknall ist hier NICHT das Thema; die Urknall-Theorie hängt NICHT ursächlich von der Existenz der dunklen Materie ab; die Arbeit von Kroupa et al. widerlegt NICHT die Urknalltheorie. Wenn sie ihre tolle “Theorie” der Öffentlichkeit vorstellen wollen, dann schreiben sie ein paper und reichen es bei einer Fachzeitschrift ein. Aber hier ist der falsche Platz dafür und ich behalte mir vor, weitere Kommentare dieser Art zu löschen.

  46. #46 Walter Orlov
    16. Juni 2010

    Schon gut, keine Aufregung, ich bin artig, ich verschwinde.

  47. #47 nihil jie
    16. Juni 2010

    aber was mich bei der ganzen geschichte um die DM am meisten freut ist die tatsache, dass das konzept der wissenschaft hier ganz gut greift und gut für alle sichbar ist. es zeigt nämlich ganz deutlich, dass hier nicht um dogmen geht sondern um fakten, zahlenwerte usw… 🙂
    ich möchte dieses prinziep in einer demokratischen gesellschaft echt nicht missen… es hat in den vergangenen zeiten zu viel blut gekostet (wortwörtlich viel blut) um sich das zu erkämpfen.

  48. #48 mac
    17. Juni 2010

    Es fällt mir schwer nachzuvollziehen, wie man einerseits dieses Argument:

    Zitat Anfang
    Dabei haben sie Beobachtungen gemacht, die bisherigen Vorstellungen über die dunkle Materie widersprechen. Zum Beispiel sollten gewissen Galaxien umso heller sein, je mehr dunkle Materie sie enthalten. Denn man geht davon aus, dass sich ursprünglich zuerst die dunkle Materie zu “Klumpen” zusammengefunden hat die dann die normale Materie angezogen haben. Je mehr dunkle Materie desto mehr normale, helle Materie sollte es also geben. Das konnte aber bei Beobachtungen nicht bestätigt werden.
    Zitat Ende

    als Indiz gegen das Vorhandensein von DM ansehen kann, andererseits aber MOND unter den selben Voraussetzungen den Vorzug geben kann.

    In meinem Verständnis ist die Existenz kleiner Strukturen (Zwerggalaxien) mit (teilweise extrem) unterschiedlichem DM-Anteil ein wesentlich gewichtigeres Argument gegen MOND als gegen DM.

  49. #49 Marcel
    17. Juni 2010

    @ mac: Zunaecht eine Bitte: Koenntest du mir vielleicht ausfuehrlicher erlaeutern, warum du “die Existenz kleiner Strukturen (Zwerggalaxien) mit (teilweise extrem) unterschiedlichem DM-Anteil” fuer ein Argument haeltst, dass gegen MOND gewichtiger sei?

    Wir unterscheiden im Paper zwischen der Untersuchung der fuenf neuen Problemen fuer die vorherrschende Dunkle-Materie-Theorie und dem Ausblick auf moegliche Alternativen. Dass alle DM-Modelle eine Relation zwischen der Masse eines DM-Halos und der von ihm gehaltenen leuchtenden Materie vorhersagen, ist eigentlich selbstverstaendlicht (Energieerhaltung). Ueberprueft man das aber an den Beobachtungen, so wird man ueberrscht: Die Satellitengalaxien haben zwar Leuchtkraefte, die ueber vier Groessenordnungen erstrecken, man findet fuer alle aber fast die gleiche dynamische Masse (siehe das Nature Paper von Stigari, 2008). In so fern sollte es klar sein, dass dies ein ernstes Problem des DM-Modells ist.

    Welche Vorhersage MOND in diesem Zusammenhang fuer die Satelliten macht, kann ich nicht so leicht beantworten. Denn man muss in erster Linie beruecksichtigen, dass es sich dann um voellig andere Objekte handeln muesste: Gezeiten-Zwerggalaxien. Die Satelliten muessen dann auch nicht zwingend in einem Gleichgewichtszustand sein.

  50. #50 Karl Mistelberger
    17. Juni 2010

    Aber ich hätte auch kein Problem damit, von einer modifizierten Gravitationstheorie überzeugt zu werden.

    Ein neuer Ansatz löst ein Problem, hilft aber gar nichts bei bekannten Problemen und überzeugt deshalb überhaupt nicht: https://scienceblogs.com/startswithabang/2009/10/is_it_evil_to_sell_mond_over_d.php

  51. #51 Marcel
    17. Juni 2010

    @ Karl Mistelberger:
    das stimmt so nicht. Der von Ihnen verlinkte Artikel ist leider kein Glanzstueck in der Diskussion um MOND. Lesen Sie einmal die Kommentare unter ihm, Sie werden feststellen, dass vieles nicht korrekt dargestellt wird. Besonders die letzten Kommentare aeussern sich dazu, was MOND doch alles erklaert.
    Man muss ausserdem bedenken: Selbst wenn noch gar keine anderen Erklaerungsansaetze wie MOND existieren wuerde, die Probleme der vorherrschenden Dunkle-Materie-Kosmologie sind eklatant und wuerden auch ohne alternatives Modell bestehen bleiben.
    Den von ihnen verlinkten Artikel haben wir uebrigens zum Teil schon diskutiert (unten in den Kommentaren): https://www.scienceblogs.de/astrodicticum-simplex/2010/06/wissenschaft-esoterik-egal-welt-der-wunder-und-happinez.php

  52. #52 mac
    18. Juni 2010

    Hallo Marcel,

    zunächst mal will ich hier nicht den Eindruck erwecken, als wäre ich ein Fachmann auf diesem Gebiet. Ich bin Laie, der sich in ein Teilgebiet dieser Thematik aus Neugier hobby-mäßig eingearbeitet hat. Ich betrachte also eine Diskussion mit Dir als Chance Neues zu lernen und das bisher gelernte zu überprüfen.

    zu Deiner Frage

    Zitat Anfang
    Koenntest du mir vielleicht ausfuehrlicher erlaeutern, warum du …
    Zitat Ende

    Nimmt man MOND als Ursache für die der DM zugeschriebenen Wirkung an, muß man akzeptieren, daß man in einer Struktur mit r = 8 kPc (innerhalb der Sonnenbahn) keine nennenswerte Abweichung der Dynamik vom erwarteten Wert mißt, bei der Dynamik der Sterne der Hipparchos-Messungen keine Hinweise auf DM findet, aber bei Kleingalaxien einen DM-Anteil findet, der um mehr als eine Größenordnung vom ‚globalen‘ Verhältnis von ca. 5/1 abweicht. Ich kann mir bisher jedenfalls, auch mit Eurer Untersuchung, keine Not vorstellen, die ausreicht um sowas zu schlucken.

    Dein Argument:

    Zitat Anfang
    Dass alle DM-Modelle eine Relation zwischen der Masse eines DM-Halos und der von ihm gehaltenen leuchtenden Materie vorhersagen, ist eigentlich selbstverstaendlicht (Energieerhaltung).
    Zitat Ende

    Erscheint mir, zumindest als strenge Abhängigkeit, nicht so zwingend. Wenn DM kinetische Energie nur über eine (gravitative) Impulsübertragung an BM los werden kann, dann müssen die chaotischen Prozesse bei den Verschmelzungen zu lokal unterschiedlichen Verhältnissen zwischen DM und BM führen. Ich sehe dieses Argument gestützt durch z.B. die Interpretation des Bullet Clusters (in der Abbildung von Florian hier auf dieser Seite).

    Akzeptiert man die Auffassung, daß sich große Galaxien durch Verschmelzung kleinerer Strukturen gebildet haben, dann erscheint mir in diesem Zusammenhang auch normal, daß es auch bei solchen Strukturen ebenso Unterschiede im Verhältnis DM/BM gibt, wenn auch nicht mehr so krass wie bei den Zwerggalaxien. (Z.B. NGC1132 https://www.astronews.com/news/artikel/2008/02/0802-007.shtml )

    Gerade bei solchen Beobachtungen sehe ich keine glaubwürdige Möglichkeit auf DM, allein mit einer modifizierten Gravitationstheorie verzichten zu können.

    Ob es wenigstens bei den Zwerggalaxien den Ausweg aus diesem Dilemma für MOND über nicht leuchtende baryonische Materie gibt, kann ich mir zwar vorstellen, aber ohne tieferen Einstieg komme ich da auch nicht über die Aussagen von z.B E. D’Onghia https://arxiv.org/abs/0907.2442 die Du ja auch schon indirekt erwähnt hast, hinaus.

    Wenn man MOND favorisiert, dann fallen ja die teilweise extremen Unterschiede im Verhältnis BM/DM ohne einen stetigen Bezug zur Ausdehnung der Strukturen nicht weg. Mit MOND allein, kann man die aber noch viel schlechter erklären, als mit DM allein.

    Herzliche Grüße

    MAC

  53. #53 Bullet
    18. Juni 2010

    Nebenbei, mac: Zitate lassen sich cool mit
    (spitze Klammer auf) blockquote (spitze Klammer zu) Zitatzitatzitat (spitze Klammer auf) /blockquote (spitze Klammer zu)
    farblich absetzen. Das erhöht die Lesbarkeit ungemein. (Aber es dürfen innerhalb des Zitates keine Leerzeilen vorkommen. Dann wird das Tag leider automatisch geschlossen.)

    Leider ist das noch nicht zu dBremer vorgedrungen.

  54. #54 mac
    18. Juni 2010

    @Bullet

    Zitate lassen sich cool mit
    (spitze Klammer auf) blockquote (spitze Klammer zu) Zitatzitatzitat (spitze Klammer auf) /blockquote (spitze Klammer zu)
    farblich absetzen.

    Danke für den Tip! 🙂

  55. #55 Marcel
    18. Juni 2010

    Hallo mac,

    jetzt wo das halbe Institut beim Fussball gucken ist, finde ich endlich die Zeit dir zu antworten.

    Es freu mich, dass du so viel Interesse an der Thematik hast, dass du dich sogar selbst eingearbeitet hast.

    Bei MOND ist nicht der Abstand entscheident, man kann die 8 kpc von der Milchstrasse also nicht auf kleinere Strukturen anwenden. Aenderungen der Gravitation treten bei MOND unterhalb einer gewissen Beschleunigung auf (a_0 genannt, liegt bei etwa 10 hoch -10 m/s^2). Daher sagt man auch, dass die Gravitation hier im Limit kleiner Felder modifiziert wird.

    Die Abbildung 8 in unserem Paper zeigt ganz schoen, welcher Bereich von Masse-zu-Leuchtkraft-Verhaeltnissen zu erwarten ist, wenn MOND tatsaechlich gueltig ist, wir bei der Massenbestimmung von Galaxien aber nur Newtons Gesetze annehmen. Du wirst sehen, bis auf drei fallen alle Zwerggalaxien in den erwarteten Bereich. Die Ausnahmen und moegliche Erklaerngen besprechen wir ebenfalls.

    Was die Energieerhaltung betrifft: Hier geht es nicht um die von dir angesprochenen grossen Galaxien, sondern nur um die kleinsten DM Halos (von vielleicht 10 hoch 9 Sonnenmassen bevor sie der Milchstrasse nahe kamen). Deren Potentialtopf ist nicht sehr tief, weshalb das baryonische Gas, dass dort hinein faellt, sehr leicht wieder entfernt werden kann. Das Gas wird aufgeheizt, durch entstehende Sterne/ionisierende Strahlung, und durch seine Expansion aus dem Halo getrieben. Dies geschieht um so leichter, je kleiner der DM Halo und damit je geringer seine Gravitationskraft ist. Somit steht kleineren Halos viel weniger Gas zu Verfuegung, es entstehen weniger Sterne und der Satellit wird somit weniger hell sein.
    Uebrigens haben nicht wir uns diesen Prozessen zugewandt. Sie wurden auch nicht betrachtet, um die DM zu widerlegen. Das Gegenteil ist der Fall: Um zu erklaeren, warum die Modelrechnungen tausende von DM Subhalos um die Milchstrasse vorhersagen, wir aber nur 20 bis 30 Satellitengalaxien sehen, haben die Kosmologen diese Prozesse angewandt. Der Wunsch war, das Gas aus moeglichst vielen Halos heraus zu bekommen, damit diese Dunkel bleiben und nicht als Satellitengalaxien sichtbar sind. Beim dem Versuch, eben gerade das Problem der “Fehlenden Satelliten” zu loesen, haben sie dieses neue Problem fuer die DM aufgetan.

  56. #56 Karl Mistelberger
    18. Juni 2010

    Den von ihnen verlinkten Artikel haben wir uebrigens zum Teil schon diskutiert

    Beim Stöbern fiel mir auf, dass mir als Abonnent der ersten Stunde von Spektrum der Wissenschaft für August weiteres Ungemach ins Haus steht. Immer wieder schockiert mich die Redaktion mit “sensationellen” Artikeln, z.B. mit dem von Keppler und Röckmann: https://www.spektrum.de/artikel/873312 Darauf angesprochen sind die Leute seit damals etwas wortkarg. Aber wahrscheinlich werde ich das Abonnement auch in diesem Sommer nicht kündigen.

  57. #57 mac
    18. Juni 2010

    Hallo Marcel,

    erst mal vielen Dank für Deine Antwort, trotz Fußball!

    Mir war es neu, daß die hier von Dir angesprochene modifizierte Gravitationstheorie nicht vom Abstand, sondern von der Stärke der Beschleunigung aus geht, daher brauche ich etwas Zeit, bis ich mir die dazu gehörenden Gegebenheiten plausibel machen kann. Für unsere Sonne, im Abstand von 8 kPc zum GZ komme ich schon mal auf gut 2E-10 m/s^2. Damit wäre Deine Aussage zu den 8kPc noch plausibel. Jetzt muß ich allerdings noch versuchen Daten zur Dichte von Kugelsternhaufen und Kleingalaxien zu finden.

    Ein Prüfstein für diese Grenzgravitation könnten Objekte des Sonnensystems sein, die weiter weg von der Sonne, als 8000 AE sind. Das scheint aber zur Zeit noch nicht gut meßbar zu sein, wenn ich mir die meßtechnischen Schwierigkeiten bei der Pioneer Anomalie als Vergleich dazu anschaue, fürchte ich, daß das noch lange so bleiben wird.

    Während ich über die Konsequenzen einer solchen Gravitation nachgedacht habe, ist mir noch folgendes Problem? aufgefallen:
    Wenn das was Du mir dazu (quantitativ) geschrieben hast stimmt, dann sollten doch Doppelsterne im Abstand von einigen tausend AE dieser modulierten Gravitation ebenso folgen müssen und dann hätte ich zumindest im ersten Anlauf Probleme, mir stabile Umlaufbahnen vorzustellen, weil im (genügend entfernten) Apogäum eine Beschleunigung auftritt, die nicht mehr zur (Gegen-) Beschleunigung des Perigäums paßt. Ob das zur Trennung oder zu Kreisbahnen führt (ich tippe auf Kreisbahn) kann ich jetzt ohne Simulation nicht einschätzen, aber beides paßt, wenn ich da richtig informiert bin, nicht zu den beobachteten Doppelsternen.

    Herzliche Grüße

    MAC

  58. #58 peter pan
    20. Juni 2010

    Hallo lieber Herr Orlov,

    habe mich sehr gefreut, Sie hier zu sehen. Wir kennen uns flüchtig aus ein paar Mails. 7 Jahre oder 15? So ungefähr. jedenfalls sind sie ein Wissenschaftler, den ich gerne als Referenz verwende. Wegen der Qualität. Bei den hiesigen Pseudowissenschaftlern würde dies mir nicht einmal im Traum einfallen.

    Ohne daß ich das aktuelle Themma gelesen habe (man muß nicht die Meinung aller Wendehälse kennen), stimme ich mit Ihnen bestimmt überein. Obwohl ich die Postings nur im 3 Minuten Schnelldurchgang gelesen habe.

    Ihr letztes Statement hat mich bereits davon überzeugt, daß die hiesigen Pseudowissenschaftler wieder einmal die Oberhand in der virtuellen und realen Pseudowissenschaft errungen haben. Ist schwer, das richig auszudrücken, bei diesen Pseudomeistern. Daher ist es gut, mit dieser Spezies nicht weiter zu diskutieren. Es ist einfach sinnlos.

    Sie sind Wissenschaftler und haben es wirklich nicht nötig, sich von solchen Schattengestalten Denkvorschriften anhören zu müssen.

    Alles Gute und beste Erkenntnisse!

  59. #59 Marcel
    20. Juni 2010

    @ mac: Die Sache ist leider noch etwas schwieriger: die insgesamt wirkende Beschleunigung legt die Grenze fest. Man muss zusätzlich zu den Doppelsternen die Umgebung, die Galaxie in der sie sind, berücksichtigen. Also das externe Feld. Im Randbereich von Galaxien könnte es aber vielleicht wirklich einen Einfluss geben. Ich habe jetzt kein Paper im Kopf das sich damit befasst, aber ich werde mich mal umhören ob jemand eines kennt.

  60. #60 S.S.T.
    20. Juni 2010

    @peter pan
    ‘Es genügt nicht keine Meinung zu haben, man muss auch unfähig sein sie auszusprechen.’

  61. #61 mac
    21. Juni 2010

    Hallo Marcel,

    Zitat Marcel 20.6.10 10:14
    Im Randbereich von Galaxien könnte es aber vielleicht wirklich einen Einfluss geben. Ich habe jetzt kein Paper im Kopf das sich damit befasst, aber ich werde mich mal umhören ob jemand eines kennt.

    Ja, das könnte ein Prüfstein sein. Ich kann es allerdings nur in meiner Vorstellung nicht gut genug prüfen. Da wäre jetzt (für mich) eine Simulation nötig, um eine quantitative Vorstellung zu gewinnen. Die ist in erster Näherung gar nicht so schwierig (geht mit jedem PC). Zwei Zentralmassen im Abstand von Milchstraße und Andromeda, ein einzelner Stern im Umlauf um die Zentralmasse von ‚Andromeda‘, außerhalb ‚Deiner‘ 1E-10 m/s^2 Feldstärke. ‚Andromeda‘ und ‚Milchstraße‘ mit der passenden Geschwindigkeit aufeinander zu fallen lassen (Startentfernung so, wie sie vor einigen 10^9 Jahren war, und schauen wie weit der einzelne Stern seine Kreisbahn verläßt, mit MOND und mit Newton.

    Ich hoffe eigentlich, daß Du die dafür nötigen Werkzeuge schon parat hast. Ich müßte sie erst noch entwickeln, was ich mit dem bisschen Freizeit nicht so schnell hinkriegen werde. Ich werde aber dieser Frage auf jeden Fall auch selber nachgehen.

    Herzliche Grüße

    MAC

  62. #62 delta-v
    21. Juni 2010

    Man würde dem wissenschaftlichen Fortschritt einen großen Gefallen tun, wenn man sich von der fixen Idee lösen würde, alles im Kosmos wäre von Gravitation beherrscht. Niemand weiß wie Gravitation funktioniert, es ist daher nicht verwunderlich, daß die gängigen Theorien regelmäßig von neuen Beobachtungsdaten Lügen gestraft werden. Oder um es in Abwandlung eines oben gemachten Spruches zu sagen: “Es reicht nicht unwissend zu sein, man braucht auch die Mathematik, um sein Unwissen in eine wichtig klingende Theorie fassen zu können.”

    Alle Kräfte, die benötigt werden, um die Beobachtungen zu verstehen, stehen in Form der jeweiligen EM-Felder zur Verfügung, man muß lediglich von dem längst widerlegten Dogma lassen, interstellare oder intergalaktische Materie wäre elektrisch neutral und Ladungstrennung könne im All nicht stattfinden.

  63. #63 cydonia
    21. Juni 2010

    EM-Felder! So’n Quatsch, es sind WM-Felder, die EM war vor zwei Jahren!
    Aber die WM-Felder sind genauso grün wie die EM-Felder, und das ist doch eigentlich ganz schön.

  64. #64 Bernd
    28. Juni 2010

    Also ich les diese Artikel hier immer ganz gerne – bin ein interessierter Laie und kann daher mit vielen Kommentaren nicht wirklich etwas anfangen – den letzten von cydonia hab ich aber definitiv vollständig kapiert 😉

    Wahrscheinlich werden wir wirklich erst in 10 Jahren wissen ob “dunkle Materie” wirklich existiert – für mich stellt sich das Problem aus dem was ich darüber gelesen habe so dar, dass eben gewisse Beobachtungen derzeit durch das Postulat einer dunklem Materie erklärbar wären. Und aus der Vergangenheit wissen wir, dass nicht alles was “theoretisch” eingeführt wurde immer auch so notwenig war, aber andererseits vieles sich auch als richtig herausgestellt hat.
    Also bleibt nix anderes als mal wieder abzuwarten – erst wenn ein tatsächlicher Nachweis für die Existenz oder Nichtexistenz gelingt wird diese Frage geklärt sein (und dafür auch ein Nobelpreis vergeben).

  65. #65 mac
    13. Juli 2010

    Hallo Marcel,

    ich habe über Deine Erklärung zur modifizierten Gravitation nochmal nachgedacht (aber immer noch nicht simuliert 🙁 ).

    Du (Ihr?) begründest das Fehlen von beobachtbarer DM innerhalb der Sonnenbahn um das galaktische Zentrum mit einem, auch noch am Ort der Sonne immer noch zu starkem Gravitationsfeld. Eine von Newton abweichende Gravitationswirkung soll in diesem Modell erst bei Beschleunigungen unterhalb 1E-10 m/s^2 auftreten.

    Wie Du gesehen hast, war für mich dieser Feldstärkenansatz neu/überraschend und ich habe daher vor lauter Vereinfachungen beim Nachdenken über eine Simulation nicht sofort gesehen, wo da (zumindest in meiner derzeitigen Vorstellung) der Haken ist:

    Einen solchen Ort mit einer resultierenden Beschleunigung kleiner 1E-10 m/s^2 gibt es, zumindest bei den von hier aus beobachtbaren Meßobjekten, gar nicht.

    Nehmen wir einen Schneeball mit einer Masse von 27 mg und einem Durchmesser von 8 mm. Für die Materie an seiner Oberfläche ist diese Grenzfeldstärke bereits überschritten. Jeder mm Radius der dazu kommt, macht es noch schlimmer. Das aber ist, (Feldstärkentechnisch ) im Prinzip genau das Gleiche, wie die Überschreitung dieser Feldstärke am Ort der Sonnenbahn um das GZ.

    So weit so schlecht, das ist aber auch so trivial, daß Ihr dafür, vermute ich, einen Ausweg erdacht habt. Den müßte ich für eine Simulation kennen, denn mir fällt wahrscheinlich nicht der dafür einfachste Ausweg ein.

    Herzliche Grüße

    MAC

  66. #66 Marcel
    16. Juli 2010

    Hallo mac,

    schön dass du am Thema dran bleibst. Das mit MOND Simulationen ist schon etwas kompliziert, derzeit gibt es keinen (verfügbaren) N-Body Code. Zweikoerperprobleme hingegen sind natürlich machbar. Aber wie du schon abgeschätzt hast müssten das sehr weite Doppelsternsysteme sein. Diese gehen aber leicht kaputt, womöglich würde dafür sogar das Gezeitenfeld der Milchstrasse ausreichen. Und mir fällt bisher auch kein beobachtbaer Effekt ein. Sicher, die Umlaufzeiten wären kürzer, aber vermutlich immer noch zu lang. Für zwei Objekte die immer im MOND-Bereich bleiben braucht man übrigens nicht einmal eine Simulation, man muss bloss die Relation von Kraft und Beschleunigung verändern.

    Zu deinem Einwand, dass intern immer eine größere Beschleunigung herrscht: Die Frage hat mich anfangs auch stutzig gemacht. Das scheint ganz natürlich zu sein, so haben sich glücklicherweise auch zwei ganz schlaue MOND-Köpfe schon früher damit beschäftigt: Beckenstein & Milgrom (1984): https://adsabs.harvard.edu/abs/1984ApJ…286….7B
    Sie konnten zeigen, dass die Bewegung des Massenzentrums in einem externen Feld MOND folgt. Die üblicherweise gegebene Beschreibung von MOND ist leider sehr vereinfacht. Die einfachste Form von MOND erhält, glaube ich, nicht einmal den Impuls. Es gibt aber natürlich ausgereiftere Versionen, zum Beispiel AQUAL (für AQUAdratic Lagrangian). Aber da bin ich leider auch kein Experte.

    Ich versuche mir den MOND-Effekt mit der Raumkrümmung anschaulich zu machen, vielleicht hilft dir das auch. Im Bereich geringer Krümmung (=kleine Beschleunigung) krümmt sich der Raum nach MOND etwas stärker als erwartet. Dies erklärt dann die größere Beschleunigung, auch auf zusammengesetzte Systeme, die um sich herum noch einen eigenen Potentialtopf formen. Umgekehrt erklärt dieses Bild den externen Feldeffekt: Ist die Krümmung des Raums am Ort eines Systems ohnehin schon stark, so kann auch intern kein MOND-Effekt auftreten. Das Bild ist sicher nicht optimal, aber es ist spannend damit herum zu spielen und gibt eine anschaulichere Vorstellung und Erklärung, hoffe ich.

    Schönen Gruß, Marcel

  67. #67 mac
    19. Juli 2010

    Hallo Marcel,

    hab’ gerade Deine Antwort gelesen. Vielen Dank dafür. Muß ich erst mal drüber nachdenken, was im Moment nicht gerade günstig ist. Ich werde aber so, oder so darauf zurück kommen.

    Herzliche Grüße

    MAC

  68. #68 mac
    30. Juli 2010

    Hallo Marcel,

    Bei einer Frage komme ich bis jetzt nicht zu einer einleuchtenden Vorstellung.

    Mein Stand (auch mit Deiner Hilfe) bis hier ist:
    Eine Masse (Proton, Stern, Galaxis) krümmt den Raum von ihrem Schwerpunkt aus bis ihre Gravitation auf ca. 1E-10 m/s^2 abgesunken ist, newtondominiert und ab da geht diese Krümmung irgendwie in einen mONDdominierten Verlauf über. Dieses ‚Verhalten‘ soll (bei MOND) eine Eigenschaft des Raumes sein, mit der er auf eine Krümmung reagiert, nicht aber abhängig von der Entfernung zur Masse.

    Ein Doppelsternsystem mit einem Abstand der Sterne von einigen hundert AE soll untereinander dieser ‚Gesetzmäßigkeit‘ nicht folgen dürfen, wenn es sich seinerseits in einem Feld > 1E-10 m/s^2 befindet und dieser Raum daher nicht mehr durch MOND zu krümmen ist.

    Im Prinzip würde also zwar der hiesige Raum mit unserem Sonnensystems von jedem einzelnen Stern, (und nahezu von) jedem einzelnen Proton des Universums MOND dominiert gekrümmt. Das geschieht z.B. in der Wechselwirkung Galaktisches Zentrum / Sonnensystem aber nicht, weil der Gradient der Krümmung zum GZ hin (im Abstand des Sonnensystems) immer noch größer ist, als 1E-10 m/s^2.

    Das soll aber wohl geschehen, in der Wechselwirkung Andromeda / Raum der Milchstraße.

    Also der ‚Trichter‘ den sich Andromeda eingerichtet hat, kann den hiesigen Raum mONDdominiert krümmen, Sagittarius A und Proxima Centauri dagegen nicht.

    Die Logik dieses Verhaltens erschließt sich mir zumindest nicht aus mir selbst heraus. Wenn das so ist, dann müßte ja der, die Krümmung erleidende Raum ‚wissen‘, von ‚wem‘, oder von wo er gekrümmt wird und Fallunterscheidungen machen.

    Ich weiß nicht ob das deutlich wird, was ich meine? (Ich verheddere mich ja selber immer wieder, wenn ich es aus anderen Perspektiven betrachte).

    Noch ein etwas anderer Blickwinkel, bei dem ich versuche mich von den unwillkürlich entstehenden Grenzlinien zu lösen, die mit dem Trichterbild und den Namen Andromeda, Milchstraße, GZ heraufbeschworen werden :

    Leerer Raum, nur zwei Sterne sind da. Ausreichend Abstand für MOND-Anziehung. Es kommt ein dritter Stern dazu, auch genügend weit weg für MOND. Es kommen jetzt so viele Sterne dazu, daß die Bedingung für MOND deutlich überschritten wird und Newton gilt.

    Bringe ich jetzt einen weiteren Stern hinzu, der wie alle anderen für sich allein mOND wirken würde, wirkt er jetzt aber ‚nur‘ noch newton. Wenn ich jetzt den Begriff Andromeda durch den Begriff Stern ersetze und einen weiteren Stern dort plaziere, wo Andromeda wäre, und diese Galaxis in meinem Modell auch gar nicht existieren würde, dann darf dieser Stern jetzt aber, für mich bisher nicht nachvollziehbar, auf meinen Bezugsstern mit MOND wirken, obwohl alle anderen das nicht dürfen.

    Das ist für mich nicht plausibel.

    Hier hänge ich fest.

    Herzliche Grüße

    MAC

  69. #69 Aveneer
    31. Juli 2010

    Auch wenn ich mich zu den Laien zählen muss und eine Diskussion mit mir sicher zu den weniger spannenden Momenten zählen wird, so hätte ich doch zwei Fragen die mich in Zusammenhang mit der MOND-Therorie interessieren würden.

    Ich habe nicht im Ansatz das mathematische Verständnis um die Folgen einer Feldabhängigen Änderung des Gravitationsgesetzes zu erahnen. Aber ich habe die ART so verstanden, dass die Gravitation allein geometrische Ursachen hat. Wodurch die Konstanz von c erklärt wird. Eine Änderung der geometrischen Gesetze bei kleinen Gravitationsfeldern erzeugt für mich daher ein Bild, dass hier die Konstanz von c nicht mehr eingehalten werden würde?

    Kann man die ART bei kleinen Feldern wirklich so einfach modifizieren ohne das ganze Modell infrage zu stellen?

    Und dann komme ich zu einer Frage die mir unter den Nägeln brennt 😉

    Zitat Marcel:
    [blogquote] Für zwei Objekte die immer im MOND-Bereich bleiben braucht man übrigens nicht einmal eine Simulation, man muss bloss die Relation von Kraft und Beschleunigung verändern.[/blogquote] bekomme das Zitat nicht hin :((

    Während die Kraft an einem Ort die auf eine Masse wirkt durch das Newtonsche Gesetz / RT erklärt wird, hängt die Beschleunigung eines Objektes an einem Ort x ja von der Kraft und der Trägenmasse des Objektes am Ort x ab. Die Trägemasse ist nicht identisch mit der Ruhemasse sie wird in der RT nur als äquivalent beschrieben/postuliert. Ohne dass es einen wirkliche Ursache dafür gibt. Bei Ernst Mach ist die Trägemasse abhängig vom Abstand der Massen (Machschen Prinzip).

    Lange Rede kurzer „Sinn“:
    Was spricht dagegen, die Äquvivalentz von Ruhemasse und Trägermasse bei einer Feldstärke <1E-10 m/s^2 infrage zu stellen? Sprich bei einer Feldstärke <1E-10 m/s^2 nimmt die Trägemasse stärker ab, als gedacht. Bei einer gegeben Kraft an einem Ort X wäre die Beschleunigung größer?

    Ich entschuldige mich schon einmal, falls das alles völliger Humbug ist

    Gruß
    Aveneer

  70. #70 Aveneer
    31. Juli 2010

    Also das Kleinerzeichen hat den Text gelöscht ? Sorry

    Lange Rede kurzer „Sinn“:
    Was spricht dagegen, die Äquvivalentz von Ruhemasse und Trägermasse bei einer Feldstärke kleiner 1E-10 m/s^2 infrage zu stellen? Sprich bei einer Feldstärke kleiner 1E-10 m/s^2 nimmt die Trägemasse stärker ab, als gedacht. Bei einer gegeben Kraft an einem Ort X wäre die Beschleunigung größer?

    Also ich sehe schon ich muss das Bloggen irgendwo anders erst einmal üben! Peinlich

  71. #71 Marcel
    3. August 2010

    Hallo MAC und Aveneer,

    Ich will mal versuchen auf eure Frage zu antworten. Zunaechst auf Aveneer, ich denke das ist einfacher zu erklären:

    Es ist nicht notwendigerweise der Fall, dass MOND der Allgemeinen Relativitätstheorie (ART) widerspricht. Sie kommt ja bei hohen Geschwindigkeiten (v -> c) zum tragen, MOND hingegen bei geringen Beschleunigungen (a -> 0). Daher behandeln sie gegensätzliche Extreme.
    Die Annahme der Konstanz der Lichtgeschwindigkeit stammt hingegen aus der Speziellen Relativitätstheorie (SRT, die ja gerade auf c = konstant basiert). Sie wird von MOND nicht in Frage gestellt. In der ART gibt es sogar so etwas wie Abweichungen von der Konstanz der Lichtgeschwindigkeit (Shapiro-Verzögerung).

    Zur zweiten Frage: Die Impuls-Interpretation von MOND bricht ja gerade die Symmetrie der Equivalenz von schwerer und träger Masse. Ich denke, es spricht nichts dagegen, die Äquivalenz in diesem Bereich anzuzweifeln. Denn die Experimente, welche die Äquivalenz bisher bestätigen, werden ja auf der Erde durchgeführt, also alle im Bereich von so hohen Beschleunigungen, dass MOND nicht gilt.

    So, dann du dir, MAC.

    Ich fürchte da habe ich wohl mehr Verwirrung angerichtet als geholfen. Darum will ich vorweg schicken: eine eindeutige Interpretation von MOND gibt es noch nicht. Die Theorie ist, eben aufgrund der nicht-lokalen Einfluesse, auch schwerer anschaulich zu fassen, da wir es gewohnt sind im klassischen Bild zu denken. Ich will trotzdem versuchen, einige Punkte klarer zu machen.

    Das Argument mit den zu hohen Beschleunigungen der Atome eines Sterns ist etwas, auf dass wohl jeder stösst, der anfängt sich mit MOND zu beschäftigen. Die Frage ist im Endeffekt: Können wir Sterne als Punktmassen behandeln? Das von mir oben zitierte Paper zeigt, dass man ein System auch in MOND als Massenzentrum behandeln kann. Es ist, denke ich, so ähnlich wie bei der Quantenphysik: ab einer gewissen Größe können wir Systeme klassisch rechnen, obwohl wir wissen dass die internen Vorgänge anders sind.

    Eine zentrale Frage ist auch: Was ist der Unterschied zwischen internen und externen Beschleunigungen?

    Interne Beschleunigungen sind, bis zu einer gewissen Entfernung, grösser als a_0, daher ist das Objekt im inneren (z.B. Planetensystem) newtonisch. Ab einem gewissen Radius aber ist die effektive Beschleunigung des Systems kleiner, ab dort kann es dann wie ein Punktteilchen in modifizierter oder newtonscher Dynamik behandelt werden, je nachdem, wie stark das zugrundeliegened äußere Feld ist.

    Die Skalen von externe Beschleunigungen sind größer als das Objekt. Ist die externe Beschleunigung gering (z.B. Am Rang einer Galaxie), so verhält sich das Objekt wie ein Punktteilchen in MOND, ist die externe Beschleunigung aber groß (z.B. Im Zentrum einer Galaxie), so folgt es Newton.

    Der Bereich zwischen diesen Zonen ist, wie so oft, kompliziert und es gibt noch keine einheitliche Behandlung für ihn, verschiedene Übergangsfunktionen sind gebräuchlich. Achso, ich habe da auch noch ein Paper gefunden dass den Einfluss von MOND auf das Sonnensystem untersucht: https://arxiv.org/abs/0906.2937 . Wie gesagt, durch das externe Feld ist MOND eigentlich nicht gültig, aber je nach Form dieser Übergangsfunktion könnten minimale Effekte auftreten.

    Nun zu deinem Beispiel: Auch Sag A und Proxima Centauri würden den hiesigen Raum nach MOND krümmen, wenn sie alleine wären. Der Raum ist aber, aufgrund der gesamten Masseverteilung, bereits zu stark gekrümmt, so dass Newton gilt. Betrachtet man aber das Gesamtsystem Milchstrasse, so befindet es sich unter MOND-dominiertem Einfluss von Andromeda. Die interne Dynamik der Milchstrasse hat ja keinen Einfluss auf ihre Schwerpunktsbewegung.

    Der Raum braucht also keine Fallunterscheidungen. Entweder er ist “stark” gekrümmt und es gilt Newton, oder er ist nur schwach gekrümmt, dann weicht MOND von Newton ab. In diesem Bild ist die externe Beschleunigung so etwas wie eine “Hintergrundkrümmung” des Raumes, auf die dann die lokalen, stärkeren und daher Newton´schen Krümmungen aufsetzen. Damit lässt sich das System intern klassisch beschreiben, folgt als ganzes aber der modifizierten Dynamik.

    Schliesslich zu deinem Gedankenexperiment mit den Sternen: Auch eine grosse Anzahl an Sternen kann durch MOND beschrieben werden, wenn die Dichte der Sterne gering genug ist, die Beschleunigungen also kleiner als a_0 sind. Das ist zum Beispiel in Satellitengalaxien der Fall. Kommen sich dort dann zwei der Sterne sehr nahe, wird ihre gegenseitige Beschleunigung größer. So kann es dazu kommen, dass sie einander zeitweise newtonisch anziehen.
    Ist die Sternendicht aber sehr gross, so ist die Beschleunigung größer als a_0 und die Sterne folgen intern Newtons Dynamik. Ab einem gewissen Abstand (wo die Beschleunigung wieder kleiner a_0 wird) wirkt das System aber wiederum gemäß MOND.
    Gleiches gilt für einzelne Sterne. Jeder Stern wirkt gleich, was zählt ist die Gesamtbeschleunigung. Stellt du einen einsamen Stern irgendwo hin so wirkt er bis etwa 0.1 Lichtjahre laut Newton, weiter aussen laut MOND. Ist er aber von anderen Sternen umgeben die eine größere Gesamtbeschleunigung verursachen, so kann MOND dort nicht zum tragen kommen. Erst weiter draussen, wo die Beschleunigung unter a_0 gefallen ist, setzt die MOND Beschreibung wieder ein.

    Ich hoffe das hilft dir ein klareres Bild zu bekommen.

    Schönen Gruß,
    Marcel

  72. #72 Marcel
    3. August 2010

    Vielleicht interessiert manche von Euch: der allgemeinverständliche Artikel zu unserer Arbeit ist in der aktuellen Ausgabe der “Spektrum der Wissenschaft” erschienen. Er lässt sich auch kostenlos herunter laden. Ausserdem hat Spektrum uns gebeten einen Blog zum Thema zu eröffnen, um eine Diskussion zu ermöglichen. Dieser findet sich hier: https://www.scilogs.eu/en/blog/the-dark-matter-crisis

    @Aveneer: Ich wollte dir auch per Mail auf deine Kontaktformular-Anfrage antworten, aber scheinbar gibt es ein Problem mit deiner Emailadresse, meine Antwort ist zurück gekommen.

  73. #73 Aveneer
    3. August 2010

    Hallo Marcel,
    danke für deine Antwort.

    In der ART gibt es sogar so etwas wie Abweichungen von der Konstanz der Lichtgeschwindigkeit (Shapiro-Verzögerung).

    Das ist mir bekannt – aber auch diese Abweichung resultiert aus der Geometrie der Raumzeit. Eine Änderung dessen bewirkt aber eine weitere Abweichung? Aber das ist nur ein Bauchgefühl, ich werde deine Antwort nicht infrage stellen.

    Die Impuls-Interpretation von MOND bricht ja gerade die Symmetrie der Equivalenz von schwerer und träger Masse.

    Ich frage mich jedoch, warum alle Welt dann von einer Modifizierung der Gravitation spricht? Man muss doch nicht die Raumzeit stärker krümmen, wenn die Masse „leichter“ beschleunigt werden kann. Es ist ja nicht so, dass nur du dieses Bild der veränderten Raumzeitkrümmung verwendest. Alle sprechen von einer „MOdified Newtonian Dynamics“ die eine (imho) unnötige Abweichung von 1/r2 postuliert?
    Wäre es nicht sinnvoller von „MOdified Galileoian Dynamics“ zu sprechen?

    Siehst du einen Zusammenhang zum Machschen Prinzip der Trägheit? Wird das Diskutiert?

    Achso, ich habe da auch noch ein Paper gefunden dass den Einfluss von MOND auf das Sonnensystem untersucht:

    Wenn man unser Sonnesystem aus der Milchstraße nehmen würde und ins „Nichts“ setzen würde – würde die MOND für die Pioneer-Anomalie ausreichen? Sprich, durch die geringere Trägheit würde sie vom G-Feld stärker zurückgezogen als „normal“?

    Gruß
    Aveneer

  74. #74 wrentzsch
    30. August 2010

    Wie kommt ihr auf die Idee, dass der Urknall schon beendet ist?

  75. #75 Florian Freistetter
    30. August 2010

    @wrentzsch: Ähmm… weil der Urknall per definition der Anfang ist?

  76. #76 Ralf Paul
    Potsdam
    28. April 2013

    Ist Dunkle Materie notwendig?

    Mathematisch ist das Verhältnis von sichtbare und „Dunkler Materie“ aktuell mit 5% zu 95% angegeben, was einem einfachen Bruch 5/95=1/19 entspricht.
    Damit lässt sich die energetische Gesamtmenge auf das maximal 20fache steigern und ich bezweifle, dass eine so geringe Steigerung für ein unendliches Universum reicht.

    Physikalisch macht es genaugenommen auch keinen Sinn da hier eine Hebelfunktion von 1/19 beschrieben wird d.h. ein Hebel für Alles?
    Ein ziemlich kleiner Werkzeugkasten für ein ganzes Universum, dem ja jedes Schweizer Taschenmesser überlegen wäre.
    Vielleicht wäre es hier noch mal ganz sinnvoll bei Archimedes nachzulesen den der wusste wie das mit dem Hebel geht.

    Newton würde wahrscheinlich sagen, dass 1/19 zwar noch weit von einem Schwarzen Loch entfernt ist und eher einem Planeten / Mond Verhältnis entspricht.
    Auf Grund der vorliegenden Erkenntnisse würde man da aber keinen Urlaub machen wollen selbst wenn der Planet in einer habitablen Zone läge.
    Sicher würde er auch darüber nörgeln, dass man bei diesen Energiemengen noch einen Unterschied zwischen „Dunkler Materie und Energie“ macht, nur weil es schick aussieht?
    Alles genaugenommen Grundschulwissen und nachprüfbar.

    Mein Liebling bleibt ja Einstein, trotz seiner sechs in Geometrie.
    Falls E=mc² noch gültig ist, kann es die DM unter Einbeziehung der Rotationsgeschwindigkeit als zusätzlichen Multiplikator locker schlagen ohne auch nur ein Naturgesetz zu verletzen oder zu verbiegen.
    Denn E in diesem Fall wäre E dynamisch (wahrscheinlich besser) oder kurz Ed = Masse(m)*Umlaufgeschwindigkeit^2(c²)*Rotationsgeschwindigkeit (cr oder c1) schon bei einer Rotationsgeschwindigkeit von 21Kmh schon überlegen.
    Da Einstein ja auf die Dominanz der Geschwindigkeit bei der Beschreibung der Energie hin weißt, müssten Umlaufgeschwindigkeit ( c ) und Rotationsgeschwindigkeit ( c1 ) mindestens gleichwertig sein, also mindestens die zweite Potenz haben, was wiederum zu Ed=mc²c1² führen würde, wo dann schon eine Rotationsgeschwindigkeit von 5Kmh schon locker ausreicht.
    In einem dreidimensionalen System sollten wir aber eher von einer dritten Potenz für die Rotationsgeschwindigkeit ausgehen was wiederum zu Ed=mc²c1³ führt und einer Überlegenheit von Einstein über die „Dunkle Materie“ ab 3Kmh Rotationsgeschwindigkeit.
    Oh Wunder und Alles mit einfachen Grundschulwissen und schon ist die „Dunkle Seite der Macht“ geschlagen?
    Es soll jedenfalls Objekte geben, die sich sogar schneller drehen.
    Sicher wird es dazu weiterhin eine geteilte Meinung geben, aber wer sich nicht der „Dunklen Seite der Macht“ ergeben will, dem sei Trost gegeben durch die Tatsache, das sich diese Aussagen nicht nur mathematisch beweisen lassen, sondern auch experimentell und es gibt sogar praktische Anwendungen.
    Inspiration können Sie sich in der Ballistik holen (der gezogene Lauf) oder Sie greifen ein altes Experiment auf(68 oder 69 glaube ich) „Die Energieabgabe eines Teilchens beim Durchschreiten einer Phasengrenzfläche“ hier wurde das 1. Mal nachgewiesen, dass ein Teichen deutlich mehr Energie abgab als es nach E=mc² haben dürfte.
    Leider zog man daraus die falschen Schlussfolgerungen, denn man vermutete das es doch etwas schnelleres wie das Licht gibt, was ja gar nicht notwendig ist und uns nur in Schwierigkeiten mit der Speziellen Relativitätstheorie bringt und da es schon genug Unsinn gibt, müssen wir ja nicht noch die Zeit verbiegen.

    Mit freundlichem Gruß
    Ralf Paul

  77. #77 Dietmar
    28. April 2013

    Oha: Wieder ein “Einstein-Widerleger”, der, wie könnte es anders sein, nichts richtig recherchiert:

    Mein Liebling bleibt ja Einstein, trotz seiner sechs in Geometrie.

    1. Quelle bitte!

    2. Was bedeutet diese Zensur?

    Und mit Deiner Pseudo-Physik beschäftige ich mich nicht.

    Ich bin jetzt beruflich weg für ´ne Weile. Grüße in die Runde!

  78. #78 JaJoHa
    28. April 2013

    @Ralf Paul
    Kleiner Tipp: Formeln mit Latex tippen, liest sich viel angenehmer: Ein Dollarzeichen dahinter latex und ein Leerzeichen und dann die Formel, beendet mit einen weiteren Dollarzeichen 🙂
    Die Formel für Energie ist übrigens korrekter E=\gamma m_0 c^2_0 mit Vakuumlichtgeschwindigkeit c_0 und \gamma=\frac{1}{sqrt{1-\beta^2}}. Dabei ist \beta=\frac{v}{c}.
    Eine Kreisbahn würde man damit in folgender Art verarbeiten:
    Bahn nach der Zeit ableiten , r ist der Ortsvektor (KS ist dabei egal) \vec{v}=\frac{d\vec{r}}{dt}
    Das v² für “killt” die Vektoren und macht daraus Skalare.
    Funktioniert unabhängig von der Art der Bahnkurve, sofern stetig und (fast überall) differenzierbar.
    Und die Beschreibung des Experimentes klingt nach Übergangsstrahlung, damit funktionieren TRT. Da wird nur ein Bruchteil der kinetischen Energie in Strahlung umgewandelt. https://rkb.home.cern.ch/rkb/PH14pp/node194.html
    Da werden 10-30 keV bei GeV Teilchen abgestrahlt.
    Zum Thema “Raum verbiegen”
    Die Gravity Probes haben genau das nachgewiesen (ART Effekte durch die Erde)

  79. #79 JaJoHa
    28. April 2013

    Nachtrag
    Ich hab vergessen, zu sagen was m_0 ist. Das ist die Ruhemasse des betrachteten Objekts, also die Masse, die ich im IS (Inertialsystem) des Teilchens messen würde.

  80. #80 Adent
    28. April 2013

    @Ralf Paul
    Seltsam, das sie noch keine Nobelpreis bekommen haben, ach ich vergaß wegen der Weltverschwörung. Na dann viel Spaß beim rumphantasieren…

  81. #81 JaJoHa
    28. April 2013

    @Adent
    Ist schon merkwürdig, die wollen fast immer an die Formel für Energie ran und vergessen dabei parktisch immer die Masse korrekt aufzulösen in Ruhemasse und Lorentzfaktor 😉

  82. #82 Adent
    28. April 2013

    @JaJoHa
    Einfaches Schulwissen halt, hat er ja selbst geschrieben 😉

  83. #83 JaJoHa
    28. April 2013

    @Adent
    Ja, das ist so als ob ich dir mit Schulwissen aus Bio erklären wollte, wie du einen T-Rex aus Hühnern zu klonen hast 😉
    Das Problem ist fast immer, das die Leute nicht die Formel inklusive der nötigen Mathematik und den Zusammenhängen haben, sondern nur E=mc²

  84. #84 Mirko
    2. März 2016

    Hallo FF,

    Der Thread hier hat schon viel Unsinn gesehen. Ich hab eigentlich eine Frage zum ursprünglichen Thema, aber hat ja Stand 2010. Wie ist der Stand heute aus Deiner Sicht? Der Bonner Prof Pavel Kroupa scheint felsenfest der Ansicht, dass gewisse Beobachtungen Dunkle Materie ausschließen, während zB am CERN danach gesucht wird. Ist er ein Spinner? Falsch verstanden? Oder ist das eine ernsthafte Alternative? Oder ist sie durch jüngste Beobachtungen einfach nur (un-) wahrscheinlicher geworden?

    Danke!

  85. #85 Florian Freistetter
    2. März 2016

    @Mirko: Ich bin kein Experte auf diesem Gebiet. Aber mWn gibt es ausreichend Beobachtungen, die dunkle Materie zwingend nötig machen. Das heißt nicht, dass es auch ne andere Gravitationstheorie a la MOND geben kann. Aber selbst die käme nicht ohne dunkle Materie aus. Und am CERN sucht man nicht direkt nach “dunkler Materie”. Sondern nach hypothetischen Elementarteilchen, die aber – sofern vorhanden – die Bestandteile der dunklen Materie sein könnte.

  86. #86 Captain E.
    2. März 2016

    Tja, solange Professor Kroupa nicht glasklar beweisen kann, dass es Dunkle Materie nicht geben kann oder zumindest ein paar gute Belege dafür angeben kann, muss man ihn leider als “Spinner” ansehen. In der Vergangenheit waren bekanntlich manche Spinner in Wahrheit Genies, aber die meisten waren eben doch nur dieses: Spinner.

  87. #87 Alderamin
    2. März 2016

    @Mirko

    Eine modifizierte Gravitation ist einerseits nicht mit der gut belegten Allgemeinen Relativitätstheorie vereinbar und andererseits erklärt sie weder den Bullet Cluster (wo dunkle und sichtbare Materie sich getrennt haben), noch andere Gravitationslinseneffekte zwischen Galaxien, nicht das Spektrum der Hintergrundstrahlung (Punkt 7) und nicht das relative Verhältnis der Elemente, die beim Urknall entstanden. Das ist einfach mal eine schlechtere Erklärung als Dunkle Materie. Der DM vorzuwerfen, man habe ihre Bestand-Teilchen noch nicht gefunden, ist auch kein gutes Argument, denn eine Begründung für eine Abweichung vom Gravitationsgesetz gibt es ja auch nicht. Man fittet einfach die Formel an die Beobachtung (Rotationskurven der Spiralgalaxien). Das geht immer irgendwie und bringt keine neue Erkenntnis oder falsifizierbare Vorhersagen.

  88. #88 StefanL
    3. März 2016

    Na ja, “die modifizierte Gravitation” gibt es ja so nicht. Noch wird meistens darunter MOND verstanden und die reicht kaum zur Beschreibung der bennannten Phänomene. Erfolgreicher ist da schon TeVeS (Tensor-Vektor-Skalar)- oder f( R)-Gravity. Und die STVG taugt u.a. wohl auch zur Erklärung von CMB und dem Bullet Cluster.
    Alles in allem ist das (meiste) nicht ein “fitting der Formeln” vergleichbar den Epizyklen ggüb. Kepler’schen Gesetzen sondern ein Modifizieren/Verallgemeinern der strukturellen Beschreibung von “Variablen”, hier spezifisch eben von denen die in den ART Gleichungen autftreten.

    Das “Rumgeeiere” bei der ( immer noch hypothetischen) Dunklen Materie ( bspw. https:// http://www.weltderphysik.de/gebiet/astro/news/2015/dunkle-materie-und-dunkle-energie-bleiben-raetselhaft/ ) , Axionen oder Teilchen die nur der Gravitation und der starken Wechselwirkung unterliegen und/oder WIMPS sogar in mehreren Varianten und SUSY Modelle deren Testergebnisse bestimmte vormals favorisierte Modelle wohl ausschließen, vergrößert ja auch nicht gerade die Verläßlichkeit derartiger Theorien.

    Einigkeit besteht dabei bei all diesen Erklärungsansätzen darüber, daß die Beschreibung der Gravitation gemäß dem Energie-Impuls(Spannungs)-Tensor mit der baryonischen Materie allein ( + Dunkler Materie?) wie gehabt (ohne modifizierte T_μν ?) so eher nicht der Weisheit letzter Schluß ist.
    Und natürlich hat das letzte Wort das Experiment. Aber selbst wenn die Suche nach “neuen” Teilchen nicht völlig aufgegeben wird, stellt sich die Frage nach wieviel Fehlversuchen eines Nachweises ggfs. doch bestimmte Modifikationen als (i.w.) zielführender zu werten sind.

    Und um es nicht unerwähnt zu lassen: (stringtheoretische) Modelle mit parallel Universen resp. Branen-/ekpyrotische Modellierung (vgl. auch die verfrüht kommunizierten “Ergebnisse” von BICEP2) sind ebenfalls (noch) nicht ausgeschlossene Erklärungsansätze. ( https:// de.wikipedia.org/wiki/Ekpyrotisches_Universum )

  89. #89 Captain E.
    3. März 2016

    Nun, Fakt ist, dass es Teilchen gibt, die ziemlich gut zu dem passen, wie die Dunkle Materie sein müsste. Das sind die Neutrinos. Alleine reichen sie zwar nicht aus zur Erklärung der beobachteten Phänomene, aber wenn es solche Teilchen gibt (drei verschiedene Neutrinotypen!), dann spricht zunächst einmal nichts grundsätzlich dagegen, dass weitere Teilchen mit passenden Eigenschaften existieren.

  90. #90 Alderamin
    3. März 2016

    @StefanL

    Aber selbst wenn die Suche nach “neuen” Teilchen nicht völlig aufgegeben wird, stellt sich die Frage nach wieviel Fehlversuchen eines Nachweises ggfs. doch bestimmte Modifikationen als (i.w.) zielführender zu werten sind.

    Hypothetische Teilchen, die nur und ausschließlich über Gravitation wechselwirken, sind ausgesprochen schwierig nachzuweisen; dass sie auch schwach wechselwirken oder sich paarweise vernichten könnten, folgt ja nur der Logik, den verlorenen Schlüssel dort zu suchen, wo der Lichtschein der Straßenlaterne hin fällt. Beides muss ja nicht notwendigerweise der Fall sein. Tatsache ist, dass die DM-Hypothese die oben genannten Beobachtungen am besten erklärt.

    Aber am CERN wurden neulich von beiden Detektoren Hinweise auf ein neues Teilchen jenseits des Standardmodells gefunden. Bis jetzt reicht die Signifikanz noch nicht (obwohl die Kurve rein optisch gesehen ziemlich eindeutig nach einem solchen Teilchen aussieht), aber im Laufe des Jahres wird man sicher mehr erfahren. Und da könnte noch ein ganzer Rattenschwanz dran hängen. Solange dieser Bereich noch nicht ausgelotet ist (und jenseits des Standardmodells muss es noch neue Physik geben, schon wegen der CP-Verletzung) ist es zu früh, die Dunkle Materie schon abzuschreiben.

  91. #91 StefanL
    3. März 2016

    @Captain E.
    Neutrinos sind aber wohl HDM und nicht CDM.
    Wie soll es mit (hypothetischen!) sterilen Neutrinos aussehen? Neutrinos nicht der Neutrino Osszillation unterworfen?
    Unbestritten sind im Laufe der Geschichte immer wieder Teilchen aus theoretischen Vorhersagen erfolgt und haben sich experimentell bestätigen lassen – wenn auch, wie gerade im Falle von (HDM-)Neutrinos und ihrer Oszillationseigenschaft, nicht ganz so einfach wie erstmal postuliert.
    Einige der aktuellen (SUSY-)Theorien und damit ihre impliziten Teilchen(LSPs) sind aber wohl nicht zutreffend &rightarrow; https://cds.cern.ch/record/2117955/files/arXiv:1512.08002.pdf

  92. #92 Captain E.
    3. März 2016

    Nun fang hier mal nicht an, Haare zu spalten, StefanL. Wenn es die besagte Dunkle Materie geben sollte, dann hätte die nun einmal genau die Eigenschaften, die wir heutzutage den Neutrinos zuschreiben: Sie haben offensichtlich eine Ruhemasse ungleich Null und wechselwirken mit Massen und zudem unterliegen sie der schwachen Wechselwirkung. Die Neutrinos sind nun als heiß und schnell, aber warum soll es nicht noch weitere geben, die kalt und langsam sind? Denkbar ist das auf alle Fälle. Oder man findet irgendwann einen Abkühlungsprozess für Neutrinos, an den man noch nie zuvor gedacht hatte. Aber wenn die beiden großen Detektoren am LHC bereits jetzt Hinweise liefern, dann dauert es wohl wirklich nicht mehr lange, bis die Frage nach der ominösen dunklen Materie ganz oder teilweise beantwortet sein wird.

  93. #93 StefanL
    3. März 2016

    @Alderamin
    Klar, jenseits der aktuellen Messmöglichkeiten bleibt immer noch “Raum”. Für die aktuellen 13TeV gilt wie für die alten 8 TeV Ergebnisse bis auf weiteres (Zitat aus dem von Dir verlinkten Artikel:)
    The new ATLAS and CMS results do not show any significant excesses that could indicate the presence of particles predicted by alternative models such as supersymmetry. The two experiments have therefore established new limits for the masses of these hypothetical new particles.
    Bleibt spannend was neue ‘runs’ ergeben.
    Die Frage, falls ein Partikel (e.g. das 750GeV Signal ist kein weiteres Higgs) nachgewiesen wird, bleibt ob es ausreichend für eine vollständige(re) Erklärung ist oder nicht. Deswegen schrieb ich auch “…(i.w.) zielführend “.
    Mal ganz naiv gefragt: Wenn die DM-Teilchen entsprechend schwer sind, gibt es ja auch weniger – und damit natürlich auch (wahrscheinlich) weniger annhilierende Kollisionen… Gibt es über diesen Gedanken( sofern das nicht völliger Blödsinn ist) irgendwelche Abschätzungen über beobachtete Strahlung hin zu der (Massen-)Größenordnung der (nach wie vor hypothetischen) DM-Teilchenklasse? Etwa so Simulationskurven in der Art von “Teilchenmasse & Annhilationswahrscheinlichkeit deckt Gravitationseffekt & beobachtete (Annhilations-)Strahlung”?

    Tatsache ist, dass die DM-Hypothese die oben genannten Beobachtungen am besten erklärt.

    Wie willst Du das gegenwärtig begründen? Wo ist STVG tatsächlich schlechter als DM?
    Es stimmt zwar, dass Gravitation (zumindest nach unserem Verständnis) stets mit Materie(Masse/Energie) gekoppelt ist und sich daher gravitationelle (gravitative?) Fragen mit Materie angehen lassen. Das bedeutet aber doch keineswegs, dass wir den “Koppelungsmechanismus” tatsächlich vollständig beschrieben, geschweige denn (vollständig) verstanden haben.
    Niels hatte im Was können und wozu braucht man Gravitationswellen?-thread das paper Mass and Angular Momentum in General Relativity verlinkt (Btw, schon ein Satz aus dem dortigen abstract ist zitierwürdig: This article is not intended to be a rigorous and exhaustive review of the subject, but rather an invitation to the topic for non-experts. 😉 ). Insbesondere ist da die (hintergrundunabhängige) Hawking-Energie (Gl. (60) ) aufgeführt. Eine Beschreibung gravitativ wirksamer “Energie” nicht unwesentlich abhängig von der betrachteten (topologischen) Oberfläche entsprechender Raum(zeit)-bereiche sowie der metrischen Struktur. D.h. (zumindest interpretiere ich das so) ebenfalls ein (theoretischer) Hinweis auf strukturell-formal abhängige Gravitationswirkung schon aus der “Was-ist–gravitativ-wirksame-Materie”-Frage selbst heraus.

    @Captain E.
    Haarespalten? Nun, wie soll es sich denn bei “abgekühlten” Neutrinos mit der Oszillation verhalten (Massen- und Verweildauerdifferenzen)? Und wie soll das zu DM-Halos passen?
    …und ansonsten immer langsam mit den jungen Pferden – siehe obiges Zitat aus dem CERN-Artikel selbst…

  94. #94 Mirko
    3. März 2016

    Jetzt hat sich leider niemand mit den .. Indizien.. auseinander gesetzt, die der Kölner Prof gegen DM ins Feld führt. Ich vermeide “Beweise”, obwohl er das selbst so sieht. Und ich geh mal davon aus, dass er grundsätzlich mehr Ahnung vom Thema als ich hat und wohl auch mehr als die meisten Foristen hier (sorry). Warum wird er trotzdem ignoriert? Ist er eher ein Däniken oder haben alle anderen Scheuklappen, so dass nicht sein kann was nicht sein darf?

  95. #95 Krypto
    3. März 2016

    @Mirko:
    Meinst Du den aus Bonn:
    https://astro.uni-bonn.de/~pavel/
    Allein schon die Tatsache, dass er dort arbeitet, zeigt doch, dass er
    1) kein Däniken ist
    2) er seine andere Sicht ohne weiteres, insbesondere ohne direkte Auswirkungen auf seine Anstellung, verkünden kann.
    Es ist sogar außerordentlich wichtig in der Wissenschaft, dass alles hinterfragt, angezweifelt und geprüft wird. Stichwort ist: “Falsifikation”.
    Die überwiegende Mehrzahl von Fachleuten ist allerdings nicht seiner Meinung.
    Dass er ignoriert wird, behauptet er wohl kaum, das scheint mir eher Deine Meinung zu sein.
    That´s all.

    so dass nicht sein kann was nicht sein darf?

    Das erinnert mich an einen bayrischen Mathematiklehrer mit chronischer Selbstüberschätzung 😉

  96. #96 PDP10
    3. März 2016

    @Mirko:

    “Jetzt hat sich leider niemand mit den .. Indizien.. auseinander gesetzt, die der Kölner Prof gegen DM ins Feld führt. […] Warum wird er trotzdem ignoriert? Ist er eher ein Däniken oder haben alle anderen Scheuklappen, so dass nicht sein kann was nicht sein darf?

    Das sich da grad niemand mit auseinander gesetzt hat, liegt wohl eher an dem kleinen “Großstreit” der da oben um Wortklauberei geführt wird … :-).

    Aber nein, Prof. Kroupa ist ganz sicher kein Spinner und übrigens ist er Prof an der Uni Bonn – nicht Köln.

    Ich schliesse mich allerdings Florian an, wenn er schreibt, dass Kroupas Behauptung, dass es viele Indizien gibt, die die DM wiederlegen, übertrieben ist.

    Wir wissen was wir messen. Das ist das Eine.

    Kroupa und seine Kollegen versuchen nun eine Gravitationstheorie aufzustellen, die diese Messungen erklären ohne anzunehmen, dass da jede Menge Materie ist, die man im wahrsten Sinne des Wortes nicht sieht.
    Das ist gar keine so dumme Idee. Wenn man es genau nimmt, suchen wir ja jetzt schon über 80 Jahre nach dem Zeug (seit den ersten Beobachtungen von Zwicki der Bewegung von Galaxienhaufen, die mit der sichtbaren Materie alleine nicht erklärbar wären).

    Leider funktioniert Kroupas MOND Theorie wohl auf anderen Gebieten noch nicht sehr gut.

    Aber, wie gesagt, Spinnerei ist das nicht. Sondern das was Wissenschaftler immer machen, wenn sie nicht weiterkommen: Eine neue Hypothese aufstellen und sehen wie sie sich so schlägt.
    ZB. was die Vorhersage von Resultaten von Experimenten angeht oder dem Ausrechnen von dem was man schon beobachtet hat usw.

    Es ist nur so, dass alles was wir bisher wissen der dunklen Materie im Vergleich zu Kroupas Theorie deutlich bessere Chancen einräumt, als die Dortmunder am Samstag gegen Bayern haben werden.

    Aber ich vermute mal, die nächsten paar Jahre werden da Antworten bringen.
    Interessante Zeiten in denen wir leben! :-).

  97. #97 Captain E.
    4. März 2016

    @StefanL:

    Haarespalten? Nun, wie soll es sich denn bei “abgekühlten” Neutrinos mit der Oszillation verhalten (Massen- und Verweildauerdifferenzen)? Und wie soll das zu DM-Halos passen?
    …und ansonsten immer langsam mit den jungen Pferden – siehe obiges Zitat aus dem CERN-Artikel selbst…

    Tja, daran solltest du dich selber halten. Stand der Dinge ist nun einmal, dass die meisten Wissenschaftler die Existenz schwach und gravitativ wechselwirkender Materie als am wahrscheinlichsten ansehen. Fakt ist, dass derartige Materie bereits entdeckt wurde, sogar drei verschiedene Teilchen. Wenn es drei Teilchen gibt, die die aufgestellten Bedingungen prinzipiell erfüllen, warum soll es nicht auch noch ein viertes, fünftes oder sechstes Teilchen geben?

    Das mit dem “Abkühlen der Neutrinos” ist nur mal etwas, was ich aus dem Handgelenk geschüttelt habe. Die Beurteilung, ob es da etwas geben kann, überlasse ich gerne den Experten. Fakt ist natürlich, dass das gemeine Neutrino Ruhemasse haben muss, da es mit etwas niedrigerer Geschwindigkeit als der Lichtgeschwindigkeit unterwegs ist. Wenn es also nicht lichtschnell fliegen kann, ist jede denkbare Geschwindigkeit relativ zu was auch immer vorstellbar, inklusive einem relativen Stillstand. Wenn die Dunkle Materie, die die Bildung der sichtbaren Materie erst bewirkt haben soll, tatsächlich kalte Neutrinos und nicht etwas völlig neues sein sollte, dann müssten sie schon seit kurz nach dem Urknall so langsam unterwegs sein. So ganz verstanden ist der aber nun bei weitem nicht. Aber wie obe bereits erwähnt: Vielleicht findet man bald ein paar neue Kandidaten für die Dunkle Materie.

  98. #98 Florian Freistetter
    4. März 2016

    @PDP10: “Aber nein, Prof. Kroupa ist ganz sicher kein Spinner und übrigens ist er Prof an der Uni Bonn – nicht Köln.”

    Ich glaube, einer seiner Mitarbeiter hat sogar mal nen Gastbeitrag bei mir im Blog geschrieben…

  99. #99 Florian Freistetter
    4. März 2016

    @Mirko: “Jetzt hat sich leider niemand mit den .. Indizien.. auseinander gesetzt, die der Kölner Prof gegen DM ins Feld führt. “

    Ähm. Schau mal die wissenschaftliche Literatur durch (zb hier: https://arxiv.org). Wissenschaftliche Fachartikel zu MOND et al. wurden und werden regelmäßig publiziert. MOND ist ein völlig normaler Teil des wissenschaftlichen Diskurses. Es ist halt nur so, dass die Mehrheit der Astronomen in der dunklen Materie den besseren Erklärungsansatz sieht. Weder sind MOND-Forscher “Dänikens”; noch gibt es da “Scheuklappen”.

  100. #100 Alderamin
    4. März 2016

    @StefanL

    The new ATLAS and CMS results do not show any significant excesses that could indicate the presence of particles predicted by alternative models such as supersymmetry.

    SuSy muss ja auch nicht stimmen. Jedenfalls ist da anscheinend noch was, was nicht im Standardmodell steckt. Den Theoretikern wird ohne Zweifel etwas einfallen.

    Gibt es über diesen Gedanken( sofern das nicht völliger Blödsinn ist) irgendwelche Abschätzungen über beobachtete Strahlung hin zu der (Massen-)Größenordnung der (nach wie vor hypothetischen) DM-Teilchenklasse?

    Kurze Google-Suche: ja, gibt es.

    Wo ist STVG tatsächlich schlechter als DM?

    Ich kenne STVG nicht, aber DM erklärt in jedem Fall auch die Expansionsrate, die zur Bildung der beobachteten Menge an Helium und Lithium bei der Nukleosynthese vorhanden war, und da war die Materie dicht wie im Inneren eines Sterns und das gewöhnliche Gravitationsgesetz für große Feldstärken sollte da greifen (die MOND-Varianten sehen ja eher Abweichungen bei geringen Feldstärken). Und die gleiche DM-Menge relativ zu den Baryonen passt auch zu den Rotationskurven und Geschwindigkeiten der Galaxien in Galaxienhaufen. Und dann muss eine modifizierte Gravitation erklären, warum in manchen Fällen Gravitationswirkung zwischen Galaxien ohne nachweisbare baryonische Materie auftritt. Da könnten natürlich irgendwelche Sternleichen herum hängen, aber warum sollten sie das gerade dort tun?

  101. #101 StefanL
    5. März 2016

    @Alderamin
    Danke für den link zu einer Formel der Annihilationsratenschätzung (bei WIMPs). Neben ein paar weiteren Parametern/Bedingungen (Kollisionsschätzung…) naheliegend berücksichtigend die (Ruhe-)Masse eines Partikels sowie die Geschwindigkeit/Temperatur. Bezogen auf den aktuellen(?) Kenntnisstand (und hier setzt dann meine Frage bzgl. Simulation und beobachteten Phänomenen an) ist die beobachtete mögliche Annihilationsstrahlung ja äußerst gering. Das heißt dann wohl entweder die Partikel sind sehr langsam (wie passt das dann zur Filamentbildung? Gebe da ggfs. der Bullet (oder derTrainwreck) Cluster eine Abschätzung (v ggüb. CMB)?), doch so gut wie keine Annihilation (Konsequenzen für die Teicheneigenschaften?) oder sie sind einigermaßen schwer ( und hier vielleicht eine Abschätzung bspw. abgeleitet aus den Annahmen des Milchstraßen-DM-Halos?). Zumindest letzteres gäbe dann ja auch u.U. einen Hinweis auf die ggfs. zu erwartenden LHC Signale (bzw. in welchen Bereichen die gegenwärtigen DM-Modelle Vorhersagen treffen).

    Und zur STVG ( J.W. Moffat et al. arbeiten sich daran ab):
    Rotationskurven – arxiv.org/abs/astro-ph/0506370
    Tully-Fisher – arxiv.org/abs/0712.1796
    Massenprofile, GalaxienCluster – arxiv.org/abs/astro-ph/0507222 , arxiv.org/abs/0708.1935
    Bullet Cluster – arxiv.org/abs/astro-ph/0702146
    CMB und Expansion (primordiale Nukleosynthese) – arxiv.org/abs/0710.0364

    Ist für die primordiale Nukleosynthese darüber(Expansion) hinaus DM, bspw. als “Katalysator”, notwendig so wäre das dann wohl in wesentlich geringerem Maße notwendig als bei “nur-DM”-Modellen und würde DM entsprechend selten machen.

  102. #102 StefanL
    5. März 2016

    @Captain E.

    daran solltest du dich selber halten.

    Wo habe ich
    The new ATLAS and CMS results do not show any significant excesses that could indicate the presence of particles predicted by alternative models such as supersymmetry.
    missverstanden? (Du willst doch sicherlich nicht Haare spalten und hier ‘alternative models such as supersymmetry’ nur auf SUSYs bezogen haben…)

    Stand der Dinge ist nun einmal, dass die meisten Wissenschaftler die Existenz schwach und gravitativ wechselwirkender Materie als am wahrscheinlichsten ansehen.

    Dieses Argument verstehe ich nicht. Was möchtest Du damit sagen?

    Fakt ist, dass derartige Materie bereits entdeckt wurde, sogar drei verschiedene Teilchen. Wenn es drei Teilchen gibt, die die aufgestellten Bedingungen prinzipiell erfüllen, warum soll es nicht auch noch ein viertes, fünftes oder sechstes Teilchen geben?

    Vielleicht wegen der Unitarität der PMNS-matrix?

    Wenn es also nicht lichtschnell fliegen kann, ist jede denkbare Geschwindigkeit relativ zu was auch immer vorstellbar, inklusive einem relativen Stillstand.

    Selbst mit c fliegend würde ein parallel mit c fliegendes etwas stillstehen…und zwei diametral ausgesandte Neutrinos bewegen sich relativ zueinander mit annährend 2c…Kannst Du mir das was Du damit aussdrücken willst genauer erläutern? Oder soll das nur ein subtiler Hinweis auf (hypothetische) Majorana-Teilchen oder Chiraitätsbrüche sein?

    Vielleicht findet man bald ein paar neue Kandidaten für die Dunkle Materie.

    Sofern tatsächlich entsprechende Partikel, i.e. mit hinreichenden Eigenschaften, entdeckt werden (ein weiteres Higgs ist eher keine so gute DM-Erklärung).
    Irgendwie faßt das “ein paar neue Kandidaten” ein/das (gegenwärtige)Dilemma der DM-Theorie(n) zusammen. Es wimmelt ja nur so vor hypothetischen DM-Teilchen (denen die entsprechend benötigten Eigenschaften zugewiesen werden) – nahezu vergleichbar dem Teilchenzoo vor dem Standardmodell.

    Psychologisch ist das ja auch irgendwie erklärbar: Etwas faßbares( Teilchen) wird präferiert ggüb. einer abstrakten formalistischen Erklärung. Ist ähnlich dem Planeten Vulkan innerhalb der Merkurbahn vs. ART.
    Aber bitte nicht missverstehen: dies ist kein Argument in die eine oder die andere Richtung.

  103. #103 PDP10
    5. März 2016

    @Krypto:

    Das erinnert mich an einen bayrischen Mathematiklehrer mit chronischer Selbstüberschätzung

    Ach jetzt weiss ich, wen du meinst .. mann hat das gedauert mit dem Groschen :-).

  104. #104 Krypto
    5. März 2016

    @PDP10:
    ROFL, das ist wortwörtlich von ihm aus nem unterirdischen tp-Artikel übernommen 😉

  105. #105 Alderamin
    7. März 2016

    @StefanL

    Das heißt dann wohl entweder die Partikel sind sehr langsam (wie passt das dann zur Filamentbildung? Gebe da ggfs. der Bullet (oder derTrainwreck) Cluster eine Abschätzung (v ggüb. CMB)?)

    Die Teilchen müssen langsam sein, um Filamente bilden zu können (deswegen heißt das Modell ja auch “Lambda-Cold Dark Matter”) ansonsten würden sie die Fluchtgeschwindigkeit von Filamenten, Galaxienhaufen und Galaxien übertreffen und ihnen enfliehen. Deswegen funktionieren gewöhnliche Neutrinos nicht als DM.

    doch so gut wie keine Annihilation (Konsequenzen für die Teicheneigenschaften?)

    Ist wohl eine Frage des Wirkungsquerschnitts. Wie nahe müssen sich solche Teilchen kommen? Wie oft begegnen sie sich? Die mittlere Dichte in der Milchstraße soll so bei einer Protonenmasse pro drei Kubikzentimeter liegen. Wenn die Teilchen einige GeV haben (Proton knapp 1 GeV), dann liegen aus mikrokosmischer Sicht Welten zwischen ihnen.

    oder sie sind einigermaßen schwer ( und hier vielleicht eine Abschätzung bspw. abgeleitet aus den Annahmen des Milchstraßen-DM-Halos?)

    Über die Masse der Teilchen lässt sich nur spekulieren, einzig die Dichte kann man ermitteln. Viele leichte Teilchen oder wenige massive pro Volumeneinheit ergeben die selbe Dichte.

    Und zur STVG ( J.W. Moffat et al. arbeiten sich daran ab):
    Rotationskurven – arxiv.org/abs/astro-ph/0506370
    Tully-Fisher – arxiv.org/abs/0712.1796
    Massenprofile, GalaxienCluster – arxiv.org/abs/astro-ph/0507222 , arxiv.org/abs/0708.1935
    Bullet Cluster – arxiv.org/abs/astro-ph/0702146
    CMB und Expansion (primordiale Nukleosynthese) – arxiv.org/abs/0710.0364

    Danke für die Links.

    Ist für die primordiale Nukleosynthese darüber(Expansion) hinaus DM, bspw. als “Katalysator”, notwendig so wäre das dann wohl in wesentlich geringerem Maße notwendig als bei “nur-DM”-Modellen und würde DM entsprechend selten machen.

    Wie gesagt, die Teilchendichte, die zur Nukleosynthese passt, passt auch zur beobachteten Gesamtmenge, mit der man die Rotationskurven der Galaxien erklären kann. Wenn da eine modifzierte Gravitation mitspielte und weniger DM nötig wäre, dann würde die Menge nicht mehr zur Nukleosynthese passen. (“Katalysator” klingt ein bisschen danach, als ob die Teilchen an der Nukleosynthese selbst teilgenommen hätten, aber ihre Dichte hat lediglich die Expansion des Universums gebremst, so dass die zur Fusion der heute im unveränderten Gas gefundenen Anteile von Deuterium, Helium, Lithium etc. nötigen Bedingungen lange genug anhalten konnten).

  106. #106 Randerscheinung
    Nürnberg
    3. September 2018

    Ich würde gern mal eine Frage zu dem Thema loswerden:
    Wenn die DM eine mögliche Erklärung für die Bewegung der Massen in großen Entfernungen zum Massezentrum sein kann(*1), wäre dann nicht eine der gravitativen Raumkrümmung entgegengesetzte Raumkrümmung (*2) eine der DM absolut äquivalente Erklärung??? Wieso erwähnt das keiner?

    *1: was sie ja ist
    *2: für kleine a, also weit weg vom Massezentrum = Rand des Universums

    Dabei habe ich eine bestimmte Vorstellung von der Raumkrümmung:
    gravitativ=dichter werden der Raumzeit
    der gravitativen entgegengesetzt = dünner werden der RZ
    Das stimmt aber nicht mit dem gängigen Bild des Gummituchs überein… Wobei das ja so wie so der gängigen Annahme widerspricht, dass die RZ nur innere, keine äußere Krümmung (in höherer Dimension) besitzt…

  107. #107 PDP10
    3. September 2018

    @Randerscheinung:

    wäre dann nicht eine der gravitativen Raumkrümmung entgegengesetzte Raumkrümmung (*2) eine der DM absolut äquivalente Erklärung??? Wieso erwähnt das keiner?

    Das wäre sie möglicherweise wenn man das mal durchrechnen könnte und dann auf die gleichen Ergebnisse käme (was ich ehrlich gesagt bezweifel).

    Die Frage der Fragen wäre allerdings: Wo sollte diese “entgegengesetzte Raumkrümmung” her kommen?

    Masse krümmt den Raum. (sehr vereinfacht gesagt …).

    Was sollte also den Raum “entgegengesetzt” krümmen?

  108. #108 Randerscheinung
    Nürnberg
    3. September 2018

    Ok, jetzt wirs es spekulativ ;-). Möchte vorher noch mal betonen, dass die Ursprungsfrage nur darum ging, ob nicht solch eine umgekehrte Raumkrümmung äquivalent der DM wäre…

    Nun zur Spekulation:
    Da dies den Bereich kleiner Beschleunigungen betrifft, handelt es sich um Bereiche sehr weit entfernt vom Massezentrum. Man könnte annehmen, dass Raum quasi an Materie gekoppelt ist und in hinreichender Entfernung von dieser Materie dünner wird. Ok, das klingt ein bisschen ungewohnt, aber für mich unterscheiden sich die Vorstellungen “der Raum ist unendlich” und “der Raum verschwindet in genügender Entfernung zum Massezentrum” nicht im Grad ihrer Absurdität.

  109. #109 PDP10
    4. September 2018

    @Randerscheinung:

    Das beantwortet aber die Frage nicht.

    Was sollte also den Raum “entgegengesetzt” krümmen?

    Wenn der Raum “dünner” wäre, wieso sollte er “entgegengesetzt” gekrümmt sein?

  110. #110 Randerscheinung
    4. September 2018

    “Dünner werdend” wäre auf jeden Fall eine Raumkrümmung. Ob die Einsteinsche (gravitative) Raumkrümmung äquivalent zu “dichter werdend” ist bin ich mir allerdings nicht sicher.
    Aber es heißt, dass die gravitative Raumkrümmung nur eine innere Krümmung ist, ohne äußere (in höheren Dimensionen) und da sie symmetrisch um ein kugelförmiges Massezentrum herum auftritt wäre auch hier “dichter werden des Raumes” eine Möglichkeit. Mir fällt eigentlich keine andere Möglichkeit ein ohne weitere Dimensionen zu bemühen. Was soll denn eine Raumkrümmung, die nur innere, nicht äußere, RK ist sonst sein als eine Veränderung der Dichte?

  111. #111 Bullet
    4. September 2018

    wait, wait… die Dunkle Materie ist ein Hilfskonstrukt, das erklärt, wieso mehr Massenwirkung da ist als (leuchtende) Masse beobachtbar. D.h. Dunkle Materie fungiert als Erklärung für stärkere Krümmung. Damit ist doch bereits der Gedanke

    wäre dann nicht eine der gravitativen Raumkrümmung entgegengesetzte Raumkrümmung (*2) eine der DM absolut äquivalente Erklärung???

    vom Tisch.

  112. #112 Randerscheinung
    4. September 2018

    stärkere Krümmung innen ist aber äquivalent zu entgegengesetzter Krümmung außen

    Man könnte statt dem (@Bullet: mir gefällt der Begriff) “Hilfskonstrukt” Dunkle Materie diesseits der “Probemasse” (*1) doch auch ebenso als Hilfskonstrukt annehmen, dass es eine entgegengesetzte Raumkrümmung jenseits der Probemasse gibt. Dies sollten meiner Meinung nach absolut äquivalente Erklärungsmöglichkeiten sein. Das ist doch eigentlich ganz simpel! Einzig muss man dann die “Unendlichkeit des leeren Raums” und die “Gleichförmigkeit des leeren Raums” über Bord werfen. Gibt es da wirklich gar nichts zu dem Thema, keine Überlegungen in diese Richtung?

    (*1) die sich abweichend von den Erwartungen aus “heller Materie” und ART bewegt

  113. #113 Bullet
    4. September 2018

    stärkere Krümmung innen ist aber äquivalent zu entgegengesetzter Krümmung außen

    Wus? Wie kommst’n auf dit schmale Brett?

  114. #114 Randerscheinung
    4. September 2018

    Nur rein theoretisch! Ich denke nur, es sollte von der Mathematik her egal sein ob der Raum im Inneren dichter oder der äußere dünner ist. Das ist äquivalent. Was wir sehen ist eine Beschleunigung, die von einer größeren Raumkrümmung im Inneren (=dunkler Materie) herrühren könnte. Es sollte äquivalent sein, diese Beschleunigung mit einer entgegengesetzten Raumkrümmung im Äußeren Bereich (kleine a) zu erklären. Mehr sag ich ja gar nicht.
    Oder ist schon dieser Gedanke aus irgendeinem Grund falsch? Wieso?

  115. #115 Alderamin
    4. September 2018

    @Randerscheinung

    In positiv gekrümmtem Raum ist die Winkelsumme im Dreieck > 180° und der Kreisumfang < 2πr. In negativ gekrümmtem Raum gilt entsprechendes mit umgekehrten Ungleichheitszeichen. “Innen” und “außen” gibt’s da nicht; es ist keine höhere Dimension nötig, in die sich der Raum (oder besser die Raumzeit) krümmen könnte.

    Und selbst wenn dem so wäre: eine Kugel wie die Erde ist ohnehin gemäß obiger Definition innen wie außen positiv gekrümmt, eine Sattelfläche innen wie außen negativ.

    Die dunkle Materie krümmt den Raum eindeutig positiv, wie jede andere Masse auch. Sie ist in Galaxien verdichtet und tritt manchmal getrennt von der sichtbaren Materie auf (Bullet Cluster). Sie muss sich aus einer anfänglich homogenen Verteilung zu Filamenten und Galaxienhalos verdichtet haben, um konform mit der kosmischen Hintergrundstrahlung und der heutigen Materieverteilung sein. Und sie muss die Expansion des Universums in den ersten paar Minuten stark genug gebremst haben, dass Druck und Temperatur zur Entstehung von 25% Helium und einem Bruchteil Deuterium, Helium-3, Lithium und Beryllium reichten. Das wären so die Mindestanforderungen an eine alternative Theorie der Dunklen Materie…

  116. #116 alex
    4. September 2018

    @Randerscheinung(#110)
    Ich bin mir nicht sicher, ob ich verstehe was genau du mit der “Dichte des Raums” meinst. Aber das klingt ein bisschen wie in der metrischen Version der Nordström-Gravitationstheorie. Dort ist die Metrik überall ein skalares Vielfaches der Minkowski-Metrik (mit einem von Ort und Zeit abhängigen Skalierungsfaktor).

    Aber diese Theorie ist experimentell widerlegt. Und in der ART gibt es deutlich mehr Freiheitsgrade für Krümmung. “Veränderung der Dichte” ist dort nicht die einzige Art in der die Raumzeit gekrümmt sein kann.

  117. #117 Randerscheinung
    4. September 2018

    Hallo Alderadmin,

    vielen Dank für Deine schon mal sehr hilfreiche Antwort!

    Ich meinte mit innen “näher am Massezentrum als die Probemasse” und mit außen “weiter vom Massezentrum entfernt als die Probemasse”.
    Außerdem gibt es noch äußere und innere Krümmung. Innere kommt ohne höhere Dimensionen aus und reicht für die Beschreibung der Raumkrümmung lt. ART

    Mein Gedanke ist lediglich, dass “mehr Materie innen” äquivalent sein sollte zu ” weniger dichter Raum außen”.

    Frage 1:
    Der Raum um zB. unsere Sonne ist nach der Winkelsummen-Definition positiv gekrümmt, oder?

    Frage 2:
    Was bedeutet denn innere Krümmung ohne äußere Krümmung? In meiner Vorstellung ist das (wenn punktsymmetrisch) eine Dichte-Veränderung. Positive Krümmung = höhere Dichte. Ist diese Vorstellung richtig oder falsch?

    Das Problem bei Deinen Beispielen (Kugel und Sattel): sie haben beide sowohl innere als auch äußere Krümmung. Was wäre nun also eine innere Krümmung ohne äußere Krümmung? Wenn punktsymmetrisch dann zwingend eine Dichteveränderung, oder?

  118. #118 Bullet
    4. September 2018

    Ich denke nur, es sollte von der Mathematik her egal sein ob der Raum im Inneren dichter oder der äußere dünner ist. Das ist äquivalent.

    Nein, ist es nicht.
    Gegenbeispiel, Wenn aus einem Raum mit 3 Leuten 5 Leute rausgehen, müssen erst 2 Leute wieder reinkommen, damit der Raum leer wird. Ist das mathematisch korrekt?

  119. #119 Randerscheinung
    4. September 2018

    @alex #116:
    danke, sehr interessant. So wie Du das beschreibst entspricht das tatsächlich so ungefähr meiner Vorstellung (Krümmung = Metrikänderung des flachen 4D-Raums). Aber wie wurde diese Theorie widerlegt? Und was bedeutet das? Und gilt dabei immer noch, was ich mal irgendwo gelesen habe, dass die Raumkrümmung der ART nur eine “innere” Krümmung ist, und ohne höhere Dimensionen auskommt?
    Was meinst Du mit “mehr Freiheitsgrade für Krümmung”? Das würde doch in Richtung “äußere Krümmung” gehen, also höhere Dimensionen, oder?

    Die einfachste Form der Raumkrümmung wäre doch tatsächlich ein von Ort und Zeit abhängiges skalares Vielfaches der Minkowski-Metrik… Es wäre wirklich interessant zu wissen, wie das widerlegt wurde. Für alle Größenordnungen widerlegt? Echt? Ich bin gespannt!

    Für den ursprünglichen Gedanken, dass die dunkle Materie ersetzt werden könnte durch einen entgegengesetzten Krümmungs-Effekt weit weg vom Massezentrum ist aber das Wesen der Krümmung nicht von Belang, als Dichteveränderung fände ich es eben plausibel (Rand des Universums).

  120. #120 Randerscheinung
    4. September 2018

    @Bullet: nette Methapher, beschreibt aber nur, das Du eine entgegengesetzte Krümmung nicht für möglich hältst weil Du sie Dir nicht vorstellen kannst.
    War nicht die Antimaterie anfangs ebenso seltsam?

  121. #121 alex
    4. September 2018

    @Randerscheinung

    Krümmung = Metrikänderung des flachen 4D-Raums

    Das ist sowohl bei Nordström als auch bei Einstein so (zumindest lokal). Der Unterschied ist, dass bei Nordström die Metrik immer ein skalares Vielfaches der Minkowski-Metrik ist. Bei Einstein muss das nicht der Fall sein. Und in vielen interessanten Fällen (z.B. der Schwarzschild-Metrik) ist das auch nicht so.

    Aber wie wurde diese Theorie widerlegt?

    Man hat ausgerechnet, welche Vorhersagen diese Theorie macht. Z.B. sagt sie, dass Licht im Schwerefeld der Sonne nicht abgelenkt wird. Oder sie produziert einen anderen Wert für die Perihel-Drehung des Merkur als die ART. Und wenn man das dann in der Natur misst, passen die Resultate zur ART aber nicht zur Nordström-Theorie.

    Und gilt dabei immer noch, was ich mal irgendwo gelesen habe, dass die Raumkrümmung der ART nur eine “innere” Krümmung ist, und ohne höhere Dimensionen auskommt?

    Ja.

    Was meinst Du mit “mehr Freiheitsgrade für Krümmung”?

    Bei Nordström ist die Metrik immer ein skalares Vielfaches der Minkowski-Metrik, bei Einstein nicht. Vereinfacht gesprochen könnte bei Einstein die Zeit in Bezug auf den Raum anders gekrümmt sein als die verschiedenen Raumrichtungen zueinander. Bei Nordström geht das nicht.

    Das würde doch in Richtung “äußere Krümmung” gehen, also höhere Dimensionen, oder?

    Nein. Problematisch an der Sache ist vermutlich, dass man sich das eigentlich nur für zweidimensionale Objekte vorstellen kann (also gekrümmte Flächen). Und (wenn ich mich richtig erinnere) sind die beiden Theorien in zwei Dimensionen äquivalent. (Weil es da nur einen Freiheitsgrad der inneren Krümmung gibt, die Gauß-Krümmung.) Es ist also möglich, dass die Anschauung hier in die Irre führt.

    Man sollte vielleicht noch dazu sagen, dass es für Körper die sich langsam durch schwache Gravitationsfelder bewegen (was bei den Rotationskurven von Galaxien sicher der Fall ist) gar nicht auf die Krümmung des Raumes ankommt. Sondern nur auf die “Krümmung der Zeit in Bezug auf den Raum”. Technischer gesprochen: Die Ortsabhängigkeit der 00-Komponente der Metrik. Argumentationen die nur die Krümmung des Raums betrachten ohne die Zeit miteinzubeziehen, führen also vermutlich nicht zum Ziel.

  122. #122 noch'n Flo
    Schoggiland
    4. September 2018

    @ Bulli:

    Ist das mathematisch korrekt?

    Nope, da alle Grössen der Gleichung Elemente von N sein müssen.

  123. #123 Randerscheinung
    4. September 2018

    @alex: sehr schön, vielen Dank für diese aufschlussreiche Erläuterung!
    Ok, also Dichte wäre Nordström, eine Vereinfachung, ein skalarer Faktor der Minkowski-Metrik statt unabhängige Krümmung in allen Raumrichtungen und der Zeit. Okok.
    Mit diesem Wissen jetzt noch mal zur Anfangsfrage:

    ==> Wäre es nicht möglich, dass man die Beschleunigung, die mit der dunklen Materie erklärbar ist, durch eine der gravitativen Raumzeitkrümmung entgegengesetzte Krümmung erklärt, die in großem Abstand zum Massezentrum auftritt? Warum redet darüber niemand? Oder habe ich einen Denkfehler?

    Ok, “entgegengesetzt” ist natürlich nicht so leicht zu definieren… Aber das man solch ein Gesetz finden könnte (da ja das ganze mit der dunklen Materie auch funktioniert) scheint mir zumindest möglich…

    Und des Weiteren fände ich es irgendwie schlüssig, wenn diese Krümmung dann zur Auflösung des Raumes in genügendem Abstand zum Massezentrum führen würde. Aber das wäre dann Spekulation oder Interpretation.

  124. #124 alex
    4. September 2018

    @Randerscheinung

    Wäre es nicht möglich, dass man die Beschleunigung, die mit der dunklen Materie erklärbar ist, durch eine der gravitativen Raumzeitkrümmung entgegengesetzte Krümmung erklärt, die in großem Abstand zum Massezentrum auftritt?

    Das kann ich aus dem Stegreif nicht sagen. Das müsste man mal an einem Beispiel durchrechnen (vielleicht am besten zunächst einmal nichtrelativistisch, weil das ist einfacher). Der mathematische Weg von der Massenverteilung zur Geschwindigkeit von Probekörpern ist ja nicht ganz trivial. Die Massenverteilung bestimmt die (Ricci-)Krümmung, die bestimmt die Zusammenhangskomponenten, die bestimmen die Beschleunigung, und die bestimmt die Geschwindigkeit. Und in jedem Schritt dieser Kette muss man (gewöhnliche oder partielle) Differentialgleichungen lösen. Ob da am Ende dunkle Materie an einer Stelle die selben Auswirkungen hat wie negative Masse (?) an einer anderen Stelle oder nicht, ist für mich nicht offensichtlich klar.

    Oder anders ausgedrückt: An einem festen Punkt kannst du sicherlich die anziehende Kraft von dunkler Materie innen durch eine abstoßende Kraft von außen ersetzen. Aber man beobachtet diesen Effekt ja nicht nur an einem Punkt sonden z.B. überall in einer Galaxie. Und ob das dann auch geht, ist für mich nicht offensichtlich.

    Eine Situation die andeutet, dass das möglicherweise nicht geht, ist der kugelsymmetrische Fall. Dort ist es sowohl in der klassischen Newtonschen Theorie als auch in der ART so, dass für die Bewegung eines Probekörpers nur die Masse relevant ist, die sich innerhalb des aktuellen Radius befindet. Die Kräfte von außerhalb heben sich alle gegenseitig auf. Also zumindest wenn die Massenverteilung kugelsymmetrisch ist, funktioniert deine Idee nicht. Es sei denn du nimmst grundlegende Änderungen an der ART vor, aber dann sind wir wieder im Bereich der MOND-Theorien.

    Und des Weiteren fände ich es irgendwie schlüssig, wenn diese Krümmung dann zur Auflösung des Raumes in genügendem Abstand zum Massezentrum führen würde.

    Ich verstehe nicht wirklich was das bedeutet.

  125. #125 Randerscheinung
    4. September 2018

    Hmm… wieso ist im kugelsymmetrischen Fall alles Äußere nicht relevant? Ich steh grade auf dem Schlauch…

  126. #126 Frantischek
    4. September 2018

    Stell dir vor du hast einen Planeten mit 1000km Durchmesser.
    Du bist am Boden eines 250km tiefen Brunnens.
    “Unter” dir befinden sich 750km Gestein, über dir 250km.
    Von dem Gestein unter deinen Füßen befinden sich 500km näher am Kern als du.
    Die 250km über dir heben die 250 unter dir die weiter vom Kern entfernt sind als du auf.
    Du spürst nur 500km.

    Hoffe das ist verständlich und richtig.

  127. #127 alex
    4. September 2018

    @Randerscheinung:
    In der Newtonschen Gravitationstheorie ist das ein klassischer Satz, heißt soweit ich weiß das “Newtonsche Schalentheorem”.
    Es ist nicht besonders schwer das nachzurechnen. Man braucht eigentlich nur ein bisschen Trigonometrie und mehrdimensionale Integration. Ich weiß nicht, ob es dafür eine einfache anschauliche Begründung gibt. Es ist jedenfalls wichtig, dass die Kraft mit 1/r^2 abfällt. Für andere Entfernungsabhängigkeiten gilt das nicht notwendigerweise.

  128. #128 alex
    4. September 2018

    @Frantischek
    Ganz richtig ist das nicht. Die 250km über dir sind dir ja viel näher als die 250km unter dir “hinter dem Kern”, und deshalb ist deren Kraft auf dich auch stärker. Aber es gibt ja auch noch Masse auf der Seite. Und davon ist mehr unter dir als über dir. Und zusammen führt das dazu, dass sich alles gegenseitig aufhebt. Jedenfalls für eine 1/r^2-Kraft.

  129. #129 Randerscheinung
    5. September 2018

    Vielen Dank für die Erklärungen!
    Newtons Schalentheorem ist sehr interessant für meine Anfangsfrage:
    Die Gravitationskraft innerhalb einer homogenen Massekugel steigt mit 1/r an!
    Dies ist der Zusammenhang in der MOND – Theorie.
    Damit nun zurück zur Probemasse weit weg vom Massezentrum:
    Wenn sich nun also außen (für kleine a) ein “entgegengesetztes Phänomen” (*1) aufbauen würde, käme man mit der gleichen Mathematik auf MOND.

    (*1): Was das genau ist muss man ja dafür erst mal noch nicht festlegen. Ich finde ein dünner werden des Raums (vielleicht damit Zeitdehnung) irgendwie plausibel. Wie oben schon erläutert. Aber das sind alles nur Metaphern…

    Ich kann mir nicht vorstellen, dass es Berechnungen dazu nicht schon gibt…

  130. #130 Alderamin
    5. September 2018

    @Randerscheinung

    Die Gravitationskraft innerhalb einer homogenen Massekugel steigt mit 1/r an!
    Dies ist der Zusammenhang in der MOND – Theorie.

    Das ist (ein) Grund, warum man Dunkle-Materie-Halos um Galaxien annimmt, denn in einer Galaxienscheibe gilt dieser Zusammenhang gerade nicht, deswegen muss MOND Newton MOdifizieren, damit es wieder passt.

    Ein anderer Grund für die Annahme der haloförmigen Verteilung ist die Ablenkung von Licht um Galaxien herum, die man messen kann. Und noch einer die Bewegung von Kugelsternhaufen bei verschiedenen Radien, die eine Galaxie umkreisen. Für die Verteilung der Masse in Abhängigkeit vom Radius gibt es ein Standardmodell, das nach Navarro, Frenk und White. Mehr dazu hier.

  131. #131 Bullet
    5. September 2018

    @Randerscheinung:

    @Bullet: nette Methapher, beschreibt aber nur, das Du eine entgegengesetzte Krümmung nicht für möglich hältst weil Du sie Dir nicht vorstellen kannst.
    War nicht die Antimaterie anfangs ebenso seltsam?

    Ich stelle zuerst fest, daß du auf meine Frage gar nicht eingehst. Nicht gelesen oder nicht verstanden?
    Ich stelle weiterhin fest, daß der Duktus deiner Antwort unangenehm nahe an der typischer Cranks ist: ausweichen, Torpfosten verschieben, ignorieren. Zufall?
    Drittens wäre die korrekte Antwort die gewesen, die noch’n Flo in #122 gegeben hat:

    Nope, da alle Grössen der Gleichung Elemente von N sein müssen.

    Das ist keine Metapher. Der Witz mit den -2 Leuten besteht in der fehlerhaften Anwendung von Zahlenmengen. Und genau das ist der Fehler, den du hier gemacht hast, als du behauptetest, “es sollte von der Mathematik her egal sein ob der Raum im Inneren dichter oder der äußere dünner ist. Das ist äquivalent.”
    Nein (zum zweiten Mal), ist es nicht.
    Außerdem wäre eine “entgegengesetzte Krümmung” auch eine Krümmung, also nichts neues.
    Der Spruch mit der Antimaterie ist ähnlich daneben, denn an Antimaterie ist nichts, aber auch gar nichts seltsam. Ich weiß nicht einmal, wie du auf einen solch absurden Gedanken kommen kannst, wenn du nur einen klitzekleinen Klecks Ahnung von der Natur der Antimaterie hast. Der Umkehrschluß: hast du nicht.
    Was soll also das Theater?

  132. #132 alex
    5. September 2018

    @Randerscheinung:

    Die Gravitationskraft innerhalb einer homogenen Massekugel steigt mit 1/r an!

    Nein, sie steigt mit r an. Aber ich sehe nicht wirklich, warum das hier relevant ist.

    Dies ist der Zusammenhang in der MOND – Theorie.

    Das verstehe ich nicht.

    Wenn sich nun also außen (für kleine a) ein “entgegengesetztes Phänomen” (*1) aufbauen würde, käme man mit der gleichen Mathematik auf MOND.

    Ein zu was entgegengesetzes Phänomen? Das Schalentheorem zeigt doch, dass zumindest im kugelsymmetrischen Fall man dunkle Materie innen nicht durch etwas ersetzen kann das von außen eine abstoßend wirkende (und dem selben Entfernungsgesetz unterliegende) Kraft ausübt.

    Was das genau ist muss man ja dafür erst mal noch nicht festlegen.

    Man muss aber schon festlegen, wie es sich physikalisch verhält. Denn sonst kann man ja nicht berechnen, welche Konsequenzen das hätte.

    Ich finde ein dünner werden des Raums (vielleicht damit Zeitdehnung) irgendwie plausibel.

    Ich habe leider keine Ahnung, was ein dünner werden des Raums sein soll. Deshalb kann ich das nicht besonders plausibel finden.

    Ich denke es ist sinnvoll, das ganze zunächst klassisch zu betrachten, d.h. innerhalb oder als Modifikation der Newtonschen Mechanik und nicht der ART. Man hat es ja mit kleinen Geschwindigkeiten und schwachen Gravitationsfeldern zu tun, daher sollte das möglich sein. Und die Newtonsche Mechanik ist anschaulicher und die Mathematik dafür ist viel einfacher. Dann muss man auch nicht mit vage definierten Metaphern argumentieren. Und wenn man eine einigermaßen gut funktionierende klassische Beschreibung gefunden hat, kann man sich immer noch überlegen, wie man das in die ART einbaut (so wurde das ja auch bei MOND gemacht).

    Es würde vielleicht helfen, wenn du erklären könntest, was genau dein Ziel ist. Es ist ja längst bekannt, dass man die Rotationskurven von Galaxien mit MOND erklären kann. Wenn deine Idee also etwas erfordert, das von außen eine nach innen wirkende Kraft ausübt und zusätzlich noch eine Modifikation der bekannten Naturgesetze notwendig ist, dann ist das a priori weniger plausibel als dunkle Materie und MOND. Weil du ja zwei neue Dinge einführen musst.

    @Bullet:

    Außerdem wäre eine “entgegengesetzte Krümmung” auch eine Krümmung, also nichts neues.

    Ich verstehe zwar nicht wirklich was Randerscheinung genau meint, aber möglicherweise erfordert seine/ihre Idee Materie mit negativer Dichte. Das wäre schon etwas neues.

    Der Spruch mit der Antimaterie ist ähnlich daneben, denn an Antimaterie ist nichts, aber auch gar nichts seltsam.

    Naja, als Antimaterie das erste mal theoretisch vorhergesagt und dann danach experimentell beobachtet wurde, war das schon seltsam. Nicht so seltsam wie Materie mit negativer Dichte, aber schon seltsam. Aber das hat ja mit dem Thema hier nichts zu tun.

  133. #133 Alderamin
    5. September 2018

    @alex

    Nein, sie steigt mit r an.

    *Stirnklatsch* – natürlich steigt sie an…!

    Ich verstehe zwar nicht wirklich was Randerscheinung genau meint, aber möglicherweise erfordert seine/ihre Idee Materie mit negativer Dichte. Das wäre schon etwas neues.

    Oder negativen Druck, dann wäre es so was wie Dunkle Energie, die aber genau das Gegenteil von DM bewirkt.

  134. #134 Randerscheinung
    5. September 2018

    @ Bullet,
    insofern Metapher, weil Du mit einer Analogie die Absurdität meiner Überlegungen herausstellen willst.
    Die Mathematik an sich (3-5+2=0) ist schon mal nicht falsch. Sie kann in dem Fall nicht angewendet werden, weil es keine “Antimenschen” gibt. (Wie Du ja auch sagst: fehlerhafte Anwendung von Zahlenmengen) Deshalb meine Analogie mit der Antimaterie: Manchmal wäre diese Mathematik eben doch anwendbar: 3 Teilchen sind drin, 5 Antiteilchen kommen dazu, 2 Teilchen müssen hinein und dann ist der Raum wieder leer.
    (Dabei fällt mir ein: Der Bus hält leer an einer Haltestelle, 5 Leute steigen ein, an der nächsten steigen 6 Leute aus – um welche Haltestelle handelt es sich?)

    So, und nun zum Ursprungsproblem:
    Es wäre Deiner Meinung nach NICHT äquivalent ob der Raum im Inneren dichter oder im Äußeren dünner ist? Wieso nicht? Bin echt gespannt auf Deine Antwort!

    Die Skalar Tensor Vektor Gravitation schafft einige korrekte Berechnungen von Gravitationslinsen bis Bullet-Cluster… auch sehr interessant. Was spricht eigentlich da dagegen?

  135. #135 Randerscheinung
    5. September 2018

    @alex:

    Die Gravitationskraft innerhalb einer homogenen Massekugel steigt mit 1/r an!

    Nein, sie steigt mit r an. Aber ich sehe nicht wirklich, warum das hier relevant ist.

    Ok, stimmt, mein Fehler: Die Gravitationskraft innerhalb der homogenen Massekugel ist proportional zu r. Dieser Zusammenhang kommt durch M1~r^3 und F~1/r^2 aus dem Gravitationsgesetz. Gekürzt bleibt F~r. Anders ist es außerhalb, da ist der Zusammenhang 1/r^2. In der MOND-Theorie wird ab einer gewissen Schwäche der Felder ein 1/r-Zusammenhang angenommen anstatt des 1/r^2.

    Wenn sich nun also außen (für kleine a) ein “entgegengesetztes Phänomen” (*1) aufbauen würde, käme man mit der gleichen Mathematik auf MOND.

    Ein zu was entgegengesetzes Phänomen? Das Schalentheorem zeigt doch, dass zumindest im kugelsymmetrischen Fall man dunkle Materie innen nicht durch etwas ersetzen kann das von außen eine abstoßend wirkende (und dem selben Entfernungsgesetz unterliegende) Kraft ausübt.

    Bei meiner Vorstellung vom “Dünnerwerden des Raums” handelt es sich ja nicht um “eine abstoßende Kraft”.
    Was haltet ihr davon:
    Masse = Quelle des Raums
    genügende Entfernung = Senke des Raums

    Bisherige sonstige Vorschläge:
    Materie mit negativer Dichte; außen
    Dunkle Energie (Materie mit negativem Druck); außen
    Dunkle Materie (in Halos); innen
    Ich finde, sie alle treffen nicht so hundertprozentig das, was ich mit “Dünnerwerden des Raums” meine.

    Ich denke es ist sinnvoll, das ganze zunächst klassisch zu betrachten, d.h. innerhalb oder als Modifikation der Newtonschen Mechanik und nicht der ART.

    Hm… Ich weiß bloß nicht, wie ich mein Konzept vom “Dünnerwerden des Raums” in die newtonsche Mechanik transferieren soll… Wenn man wie oben mit dem Schalenmodell betrachtet, dass aus 1/r^2 1/r wird und sich fragt wieso, dann kommt man ja direkt auf die dunkle Materie. Newton gilt doch nur für den Raum mit überall konstantem metrischem Tensor, oder? Ich wüßte nicht, wie man einen veränderlichen metrischen Tensor nach Newton überführen könnte?

    Es würde vielleicht helfen, wenn du erklären könntest, was genau dein Ziel ist. Es ist ja längst bekannt, dass man die Rotationskurven von Galaxien mit MOND erklären kann. Wenn deine Idee also etwas erfordert, das von außen eine nach innen wirkende Kraft ausübt und zusätzlich noch eine Modifikation der bekannten Naturgesetze notwendig ist, dann ist das a priori weniger plausibel als dunkle Materie und MOND. Weil du ja zwei neue Dinge einführen musst.

    Ein Ziel wäre, einen Grund für die Anwendbarkeit von MOND zu finden. Ich würde ja nicht “etwas neues” einführen, sondern neue Grenzen von etwas Bewährtem (der Raumzeit) einführen.

  136. #136 alex
    5. September 2018

    @Randerscheinung:

    Bei meiner Vorstellung vom “Dünnerwerden des Raums” handelt es sich ja nicht um “eine abstoßende Kraft”.

    Dann wäre es schön, wenn du erklären könntest, um was es sich stattdessen handelt. Ich verstehe es nämlich nicht.

    Masse = Quelle des Raums
    genügende Entfernung = Senke des Raums

    Ich keine Ahnung was das bedeutet. Was sind Quellen und Senken des Raums? Lässt sich das im üblichen ART-Theorierahmen (Differentialgeometrie auf pseudo-riemannschen Mannigfaltigkeiten) beschreiben? Wenn ja, wie?

    Wenn man wie oben mit dem Schalenmodell betrachtet, dass aus 1/r^2 1/r wird und sich fragt wieso, dann kommt man ja direkt auf die dunkle Materie.

    Auch das verstehe ich nicht.

    Newton gilt doch nur für den Raum mit überall konstantem metrischem Tensor, oder? Ich wüßte nicht, wie man einen veränderlichen metrischen Tensor nach Newton überführen könnte?

    Für langsame Geschwindigkeiten und schwache Gravitationsfelder (aber nicht notwendigerweise konstante Metrik) geht die ART in die Newtonsche Theorie über. Konkret äußert sich das dann so, dass es ein Koordinatensystem gibt, in dem die Metrik nur schwach von Minkowski abweicht, wobei die Abweichung in der 00-Komponente proportional zum Newtonschen Gravitationspotential ist (und die Abweichungen in den anderen Komponenten für die Bewegung von Probekörpern irrelevant sind).

    Ein Ziel wäre, einen Grund für die Anwendbarkeit von MOND zu finden.

    Auch hier bin ich mir nicht sicher, ob ich verstehe was du meinst. MOND ist deshalb auf Rotationskurven von Galaxien anwendbar, weil sie genau dafür konstruiert wurde.

    Ich würde ja nicht “etwas neues” einführen, sondern neue Grenzen von etwas Bewährtem (der Raumzeit) einführen.

    Ich möchte mich nicht unnötig wiederholen, aber auch das verstehe ich nicht. Wenn du Dinge wie die Rotationskurven von Galaxien ohne dunkle Materie erklären willst, dann musst du bekannte Naturgesetze abändern. Wenn du also nicht einfach eine bekannte Theorie wiederholen willst (und in dem Fall wüsste ich nicht, was der Zweck dieser Diskussion ist), musst du etwas neues einführen.

  137. #137 Randerscheinung
    5. September 2018

    Ich hatte einfach den Gedanken, dass man statt der DM auch ein Dünnerwerden des Raums annehmen könnte und auf die gleichen Gesetzmäßigkeiten käme. Mehr nicht. Ich habe leider noch nicht wirklich verstanden, warum das nicht gehen soll…

  138. #138 Bullet
    5. September 2018

    @Randerscheinung:

    Manchmal wäre diese Mathematik eben doch anwendbar: 3 Teilchen sind drin, 5 Antiteilchen kommen dazu, 2 Teilchen müssen hinein und dann ist der Raum wieder leer.

    Ja, das dachte ich mir. Meinst du das echt ernst? Drei Teilchen plus fünf Antiteilchen sind acht Teilchen. Kann dir jede Waage bestätigen.

    Es wäre Deiner Meinung nach NICHT äquivalent ob der Raum im Inneren dichter oder im Äußeren dünner ist? Wieso nicht? Bin echt gespannt auf Deine Antwort!

    Unabhängig davon, was du mit “Dichte” im konkreten Fall meinst: kleiner als Null kann eine Dichte nicht werden. Und wenn ich mal noch eine Analogie heranziehen darf: auf einer endlosen Fläche steht ein kegelförmiger Berg. Die Höhe über Grund ist das, was man als “Dichte” messen kann. Wenn du jetzt auf dem Kegelmantel stehst und die gegenüber dem Kegelmantel um n ° geneigte Ebene siehst, kannst du natürlich auf die Idee kommen, es sei egal, ob der Kegelmantel gegenüber der Ebene oder die Ebene gegenüber dem Kegelmantel geneigt ist. Ein Ball, den du in der Tasche hast, interessiert sich nicht dafür und zeigt auf beiden Flächen verschiedene Verhaltensweisen. Und das unabhängig davon, ob da nu ein Berg steht oder ein Loch ist.

  139. #139 alex
    5. September 2018

    @Randerscheinung:
    Die Hauptschwiedrigkeit die ich dabei sehe ist, dass ich nicht verstehe was genau du mit “Dünnerwerden des Raums” meinst. Wie gesagt, es wäre schön, wenn du das erklären könntest. Nach Möglichkeit mit etablierten mathematischen Begriffen. Denn mit anderen Metaphern wie Quellen oder Senken des Raums kann ich leider genausowenig anfangen.

  140. #140 Randerscheinung
    5. September 2018

    @Bullet:
    Würden sich die Teilchen nicht zerstrahlen? Jedenfalls könnte man ja auch die Analogie benutzen: Auf meinem Konto sind 3 €, ich bezahle für 5 und muss dann wieder 2€ einzahlen, um 0 zu erhalten (netterweise gibt mir meine Bank einen DispoKredit kostenlos für die kurze Zeit)
    Dichte kleiner als Null geht nicht, klar. Aber man könnte die Dichte irgendwo im Raum als 1 definieren und dann wird sie am Rand kleiner als 1.
    Kegel: Mein “Dünnerwerden des Raums” wäre in diesem Bild die Frage: ist es nicht egal für die Kugel, ob die geneigte Ebene an einem Berg oder einem Loch ist?
    Der Punkt, an dem sich unsere Vorstellungen offenbar unterscheiden ist der, dass Du annimmst, der leere Raum ohne Masse könnte existieren und wäre unendlich und vor allem ungekrümmt. Nach meiner Idee würde er ohne Masse nicht existieren.

  141. #141 Bullet
    5. September 2018

    Würden sich die Teilchen nicht zerstrahlen? Jedenfalls könnte man ja auch die Analogie benutzen: Auf meinem Konto sind 3 €, ich bezahle für 5 und muss dann wieder 2€ einzahlen, um 0 zu erhalten

    a) “zerstrahlen: ja, vielleicht. Aber du scheinst zu vergessen, daß “zerstrahlen” nicht “weg” bedeutet. Denn die Energie bleibt dieselbe, also ein Photonenäquivalent von acht Teilchen. Daß ich das jemandem erklären muß, der wie ganz selbstverständlich “Die Skalar Tensor Vektor Gravitation schafft einige korrekte Berechnungen von Gravitationslinsen bis Bullet-Cluster” in die Runde wirft, finde ich befremdlich. Ich weiß nicht einmal, was ein Tensor ist, aber ich verstehe genug Physik, um zu wissen, daß du da gerade echten Quark von dir gegeben hast. Das paßt nicht zusammen.
    b) deine Analogie geht wieder nicht von N aus (vgl. noch’n Flos Kommentar #122), weil du offensichtlich keine Ahnung hast, was “Antimaterie” ist.

    Dichte kleiner als Null geht nicht, klar. Aber man könnte die Dichte irgendwo im Raum als 1 definieren und dann wird sie am Rand kleiner als 1.

    Oder du ziehst einen Zaun um dich und definierst dich als “draußen”, um damit schlagartig alle Löwen der Welt eingefangen zu haben.
    Kann man machen, wirkt aber nicht gerade als Einladung zum nächsten MENSA-Treffen.

  142. #142 Randerscheinung
    5. September 2018

    Die Hauptschwiedrigkeit die ich dabei sehe ist, dass ich nicht verstehe was genau du mit “Dünnerwerden des Raums” meinst. Wie gesagt, es wäre schön, wenn du das erklären könntest. Nach Möglichkeit mit etablierten mathematischen Begriffen. Denn mit anderen Metaphern wie Quellen oder Senken des Raums kann ich leider genausowenig anfangen.

    MOND benutzt ja ab einem gewissen genügend kleinen a statt des 1/r^2 -Gravitationsgesetzes 1/r. An dieser Wechselebene (diesseits davon benutzt MOND 1/r^2, jenseits 1/r) definiere ich jetzt mal die Raumdichte als 1. In größerem Abstand zum Massezentrum wird der Raum “dünner”, d.h. die Raumdichte kleiner als 1. (weiter oben hatte ich übrigens mal innen und außen auf die Probemasse bezogen, mit dem Schalentheorem von Newton sollte man doch aber lieber diese Wechselebene nehmen) Das sähe für das Gravitationspotential genauso aus wie dunkle Materie, die sich zwischen der Probemasse und der Wechselebene befindet. Finde ich. Das meine ich mit “äquivalent”. Es sähe genauso aus. Was ist daran falsch?

  143. #143 Randerscheinung
    5. September 2018

    Angenommen, die Raumdichte ist an einem Ort 0.5. Wir schauen von der Erde aus drauf und nehmen an, sie wäre 1 (Weil wir wie Bullet denken, der leere Raum ist überall flach). Wenn sich eine Probemasse dort um einen Meter pro Sekunde bewegt sieht es für uns (die wir annehmen, dort wäre die Raumdichte 1) so aus, als hätte sich die Probemasse um 2 Meter pro Sekunde bewegt. Wir überschätzen also die Geschwindigkeit, weil wir annehmen, die Raumdichte wäre überall gleich.

  144. #144 alex
    5. September 2018

    @Randerscheinung:
    Was genau bedeutet für dich “Raumdichte”? Das ist kein Begriff der in der ART oder der Differentialgeometrie verwendet wird.

  145. #145 Randerscheinung
    5. September 2018

    Lt. Relativitätstheorie ist die Raumzeit nahe dem Massezentrum stärker gekrümmt als weiter weg. Stärker gekrümmt = Raumdichte >1.
    Versteht mich nicht falsch, das muss nicht unbedingt symmetrisch in allen Raumrichtungen und der Zeit sein oder sich durch einen skalaren Faktor aus dem Minkowski-Raum ableiten wie bei Nordström.
    Die Raumdichte sehe ich als Maß für die Stärke (und Art) der Raumkrümmung. Nahe dem Massezentrum ist sie höher als weiter weg, an der “Wechselebene” (s.oben/MOND) =1 und jenseits der Wechselebene kleiner 1.
    Das ist irgendwie immer noch nur eine Metapher, das tut mir Leid, nichts richtig Mathematisches. Aber ich kann mir ja auch nicht vorstellen, dass ich der erste bin, der diese Idee hat. Dafür ist sie eigentlich zu simpel.

    Weiter könnte man dieses Bild ausbauen und begründen: Dass zum Raum die Masse dazugehört. Dass die Wechselebene markiert, wo genau ein “Raumquant” pro Volumeneinheit der Dichte 1 existiert. Jenseits davon muss dann die Volumeneinheit größer werden, die Dichte geringer, weil die Grenze der kleinstmöglichen Gravitation erreicht ist unterhalb der der Raum eben dünner werden muss.

    Ich hätte gedacht, dass schnell jemand sagt, dass das doch schon dort oder dort behandelt und widerlegt worden ist.

  146. #146 Randerscheinung
    5. September 2018

    Versuch eines Ansatzes einer mathematischen Beschreibung:
    Nicht Raumdichte, sondern der metrische Tensor aus der ART ist das entscheidende Maß.

    @Bullet: Ein Tensor ist ein Element des V hoch n; eine Variable mit mindestens 2 Indizes, der metrische Tensor hat genau zwei Indizes und gibt an, wie sich Abstände in der gekrümmten Raumzeit berechnen lassen.

    Was auch immer eine homogene Massekugel mit dem metrischen Tensor macht, jenseits der Wechselebene sollte das Umgekehrte passieren.

  147. #147 alex
    5. September 2018

    @Randerscheinung:

    Lt. Relativitätstheorie ist die Raumzeit nahe dem Massezentrum stärker gekrümmt als weiter weg.

    Naja, so ganz richtig ist das nicht.

    Stärker gekrümmt = Raumdichte >1.

    Das ist eine ziemlich eigenartige Definition.

    Die Raumdichte sehe ich als Maß für die Stärke (und Art) der Raumkrümmung. Nahe dem Massezentrum ist sie höher als weiter weg, an der “Wechselebene” (s.oben/MOND) =1 und jenseits der Wechselebene kleiner 1.

    Nur falls das nicht klar sein sollte: Das Äquivalent der ART zur Gravitationskraft bei Newton ist nicht die Krümmung. Sondern der Zusammenhang. Und “Raumdichte” wäre doch eher direkt die Metrik (oder deren Determinante oder ähnliches). Metrik, Zusammenhang, und Krümmung sind zunächst einmal drei unterschiedliche Dinge.

    Weiter könnte man dieses Bild ausbauen und begründen: Dass zum Raum die Masse dazugehört. Dass die Wechselebene markiert, wo genau ein “Raumquant” pro Volumeneinheit der Dichte 1 existiert. Jenseits davon muss dann die Volumeneinheit größer werden, die Dichte geringer, weil die Grenze der kleinstmöglichen Gravitation erreicht ist unterhalb der der Raum eben dünner werden muss.

    Da wären wir wieder bei Aussagen die ich beim besten Willen nicht verstehe.

    Ein Tensor ist ein Element des V hoch n; eine Variable mit mindestens 2 Indizes, …

    Naja …

    Was auch immer eine homogene Massekugel mit dem metrischen Tensor macht, jenseits der Wechselebene sollte das Umgekehrte passieren.

    Und wie soll das konkret aussehen? Es fällt mir schwer mir vorzustellen was genau “das Umgekehrte” bedeuten soll. Die Metrik die man in der ART für eine homogene Kugel bekommt, ist ja bekannt. Sie steht z.B. im Wikipedia-Artikel über die Schwarzschild-Metrik. (Ich verlinke das jetzt nicht, weil Kommentare mit Links oft im Spam landen.)

    Wenn du mit deiner Idee eine Metrik produzierst, die genauso aussieht (modulo Diffeomorphismen) wie die in der gewöhnlichen ART mit dunkler Materie, dann kommen natürlich die selben Ergebnisse heraus. Aber ich sehe bislang nicht, warum das der Fall sein sollte. Und dann bleibt noch die Frage, warum genau diese Metrik herauskommen soll. Die Grundgleichung der ART ist ja nicht die Schwarzschild-Metrik. Sondern die Einsteinsche Feldgleichung. D.h. es wäre notwendig, eine Modifikation der Feldgleichung zu finden, so dass deren Lösungen deinen Ideen entsprechen. Und es ist ziemlich schwierig, die Feldgleichung abzuändern, ohne irgend welche grundsätzlichen Prinzipien zu verletzen.

    Wenn du hingegen eine Metrik produzierst, die anders aussieht als die der ART mit dunkler Materie, warum sollten dann die Bewegungen von Probekörpern gleich sein?

  148. #148 Randerscheinung
    5. September 2018

    Wenn du mit deiner Idee eine Metrik produzierst, die genauso aussieht (modulo Diffeomorphismen) wie die in der gewöhnlichen ART mit dunkler Materie, dann kommen natürlich die selben Ergebnisse heraus.

    immerhin…
    Tja, wenn ich das schaffen würde… Aber so fit bin ich natürlich nicht. Ich habe geglaubt, das es bei meinen Metaphern schon sofort bei jemandem klingelt…

    Es gibt ja so einige Anwärter, zB die STVG (Skalar-Tensor-Vektor Gravitation oder MOG, modified gravity) von Moffat, zumindest seine Versprechungen in einigen arxiv- Papers klingen ganz gut, ob er die dann halten kann weiß ich nicht. Aber auch MOG nimmt, wenn ich das richtig verstanden habe, eine flache Raumzeit an wenn keine Masse da ist.

    Schreiben denn die Feldgleichungen vor, dass die leere Raumzeit flach ist? Oder woher kommt dieses Postulat?

    Noch mal zurück zum Anfang: Ich denke immer noch, dass es zu äquivalenten Bewegungen führt, ob man nun Materie hinzufügt oder Raum wegnimmt.

  149. #149 alex
    5. September 2018

    Schreiben denn die Feldgleichungen vor, dass die leere Raumzeit flach ist? Oder woher kommt dieses Postulat?

    Wenn keine Materie vorhanden ist, ist eine flache Raumzeit eine mögliche Lösung der Feldgleichungen. (Jedenfalls wenn die kosmologische Konstante gleich null ist oder vernachlässigt werden kann.) Es ist aber nicht die einzige Lösung. Es können auch z.B. Gravitationswellen vorhanden sein. Dort wo die sind ist die Raumzeit dann nicht flach.

    Technischer ausgedrückt: Wenn keine Materie (und keine elektromagnetischen Felder etc.) vorhanden sind, ist der Energie-Impuls-Tensor gleich null. Dann sagen die Feldgleichungen (ohne kosmologische Konstante), dass der Ricci-Tensor gleich null sein muss. Eine Möglichkeit das zu erreichen ist mit verschwindendem Riemann-Krümmungstensor (denn der Ricci-Tensor ist eine Kontraktion des Riemann-Tensors), d.h. mit flacher Raumzeit.

    Ich denke immer noch, dass es zu äquivalenten Bewegungen führt, ob man nun Materie hinzufügt oder Raum wegnimmt.

    Und ich sehe immer noch nicht warum das so sein sollte. Und wie gesagt, ist im nichtrelativistischen Limes der Raum gar nicht so relevant; für Bewegungen kommt es auf die Ortsabhängigkeit der 00-Komponente der Metrik an.

  150. #150 Randerscheinung
    5. September 2018

    Ok, und wie sehen die Lösungen der Feldgleichungen für einen Massepunkt aus?

  151. #152 Randerscheinung
    6. September 2018

    Eine (triviale) Lösung der Feldgleichungen für “keine Masse” (Energie-Impuls-Tensor=0) wäre doch auch “kein Raum”, nichts. Ich hoffe, da stimmst Du mir zu?
    Und dazu fügen wir dann einen Massepunkt hinzu. Was passiert dann?

  152. #153 alex
    6. September 2018

    Hm, ob keine Raumzeit wirklich eine Lösung ist, hängt wohl davon ab wie genau man “Lösung” definiert. Das ist ein bisschen so wie dass alle Elemente der leeren Menge jede beliebige Eigenschaft erfüllen. Ob das physikalisch sinnvoll ist, wage ich zu bezweifeln.

    Aber mal angenommen man verwendet einen Lösungs-Begriff der das einschließt. Zu was soll man einen Massepunkt hinzufügen wenn nichts da ist? Ein Massepunkt ist letztlich eine bestimmte Konfiguration des Energie-Impuls-Tensorfeldes. Das Tensorfeld braucht eine Mannigfaltigkeit auf der es definiert ist. Und das heißt eine Raumzeit.

  153. #154 Randerscheinung
    6. September 2018

    Hm…
    Oder kann man eine Lösung der Feldgleichungen für einen Massepunkt finden bei der man explizit davon ausgeht, dass ohne den Massepunkt gar keine Raumzeit da wäre?

  154. #155 alex
    6. September 2018

    Soweit ich weiß ist es nicht möglich den “Zustand” bei dem keine Raumzeit existiert in der ART zu beschreiben. Jedenfalls habe ich keine Ahnung wie das gehen sollte. In Quantengravitationstheorien mag das anders sein.

    In der ART ist es ja auch nicht so dass die Raumzeit nur “wegen” der Materie existiert. Die Raumzeit ist ein dynamisches Objekt mit eigenen Freiheitsgraden (was man z.B. an den Gravitationswellen sieht).

  155. #156 Randerscheinung
    6. September 2018

    Sprich eine Mathematisierung der Aussage
    Masse = Quelle des Raums

    Man könnte z.B. Lösungen für den Massepunkt aufstellen und dann fordern, dass für m gegen 0 die Raumzeit verschwindet.

  156. #157 Randerscheinung
    6. September 2018

    Antwort auf #155:
    Hm, die Raumzeit wird in der bisherigen Interpretation der ART immer unabhängig gesehen. Aber das muss doch nicht zwingend so sein? Die Raumzeit kann doch alle möglichen Formen annehmen, unendlich oder nicht oder gekrümmt oder so…
    Ok, also als einzige Neuerung würde ich dann wie in #156 annehmen, dass ohne Masse keine Raumzeit existiert. Der Raum geht bei m-> 0 gegen 0 und die Zeit wird in diesem “Mikrouniversum” vielleicht unendlich, oder vll auch 0.
    Jetzt wäre nur die Frage aller Fragen wie man die entsprechenden Gleichungen aufstellt…

  157. #158 alex
    6. September 2018

    Man könnte z.B. Lösungen für den Massepunkt aufstellen und dann fordern, dass für m gegen 0 die Raumzeit verschwindet.

    Ich denke nicht dass das innerhalb der ART (oder ähnlichen Theorien) möglich ist. Die Existenz der Raumzeit ist dort eine Grundvoraussetzung. Auch wüsste ich nicht, wie man stetig von “eine Mannigfaltigkeit existiert” zu “keine Mannigfaltigkeit existiert” kommen könnte. Und ohne das macht der Limes keinen Sinn.

  158. #159 Randerscheinung
    6. September 2018

    Also die Lösung würde ich aus der inneren und äußeren Schwarzschild-Metrik und dann ab einem gewissen Abstand einem Zusatzterm zusammensetzen. Der Zusatzterm enthält eine “umgekehrte Krümmung” und müsste gewährleisten, dass das Ding bei m->0 immer kleiner wird.

  159. #160 alex
    6. September 2018

    Hm, die Raumzeit wird in der bisherigen Interpretation der ART immer unabhängig gesehen.

    Das ist keine Interpretation. Jedenfalls nicht in dem Sinn wie es Interpretationen der Quantenmechanik gibt. Dass es in der Raumzeit Freiheitsgrade gibt zusätzlich zu denen der Materie, ist prinzipiell experimentell testbar.

    Aber das muss doch nicht zwingend so sein? Die Raumzeit kann doch alle möglichen Formen annehmen, unendlich oder nicht oder gekrümmt oder so…

    Ich verstehe nicht was das eine mit dem anderen zu tun hat.

    Der Raum geht bei m-> 0 gegen 0 und die Zeit wird in diesem “Mikrouniversum” vielleicht unendlich, oder vll auch 0.

    Jetzt bin ich mir wieder einmal nicht sicher ob ich verstehe was genau du meinst. Es gibt zumindest mathematisch einen Unterschied zwischen “es existiert keine Raumzeit” und “die Raumzeit hat einen Durchmesser/Volumen/… von 0”. Beides ist problematisch, aber ersteres ist noch viel problematischer.

  160. #161 Randerscheinung
    6. September 2018

    Sind Mannigfaltigkeiten denn immer unendlich?
    Ich würde denken, nicht? Oder wenn doch könnte man sie ja in der Unendlichkeit immer schwächer werden lassen…

    Würde es helfen zB. Das Volumen des Raums zu berechnen und gegen 0 gehen zu lassen? Über irgendwas muss man integrieren…

  161. #162 alex
    6. September 2018

    Sind Mannigfaltigkeiten denn immer unendlich?

    “Unendlich” in welchem Sinn? Mannigfaltigkeiten haben immer unendlich viele Punkte. Manche haben ein “Ende”, d.h. einen Rand, andere nicht. Ob sie unendlich nach irgend einem Maß (im physikalischen, nicht notwendigerweise mathemathischen Sinn) sind, ist nicht immer wohldefiniert; es gibt Mannigfaltigkeiten ohne Metrik. Bei denen mit Metrik können Größen wie Volumen, Durchmesser, etc. unendlich sein, müssen es aber nicht.

  162. #163 Randerscheinung
    6. September 2018

    Dass es in der Raumzeit Freiheitsgrade gibt zusätzlich zu denen der Materie, ist prinzipiell experimentell testbar.

    Okok, aber das macht ja erst mal eigentlich nichts. Gravitationswellen sind ja auch nicht vollständig unabhängig von Materie, wurden ja von bewegender Materie erzeugt.

    Es gibt zumindest mathematisch einen Unterschied zwischen “es existiert keine Raumzeit” und “die Raumzeit hat einen Durchmesser/Volumen/… von 0”. Beides ist problematisch, aber ersteres ist noch viel problematischer.

    Ich denke, Durchmesser bzw. Volumen von null sollte reichen.

  163. #164 Randerscheinung
    6. September 2018

    Was hältst Du von #159 ?

  164. #165 alex
    6. September 2018

    @Randerscheinung:
    Es mag durchaus möglich sein, eine Metrik anzugeben, die sich in der Umgebung eines Punktes wie Schwarzschild verhält, für die der (Außen-)Raum jedoch insgesamt endliches Volumen hat. Aber ob sich Probekörper darin dann so bewegen wie du dir das vorstellst, ist eine andere Frage.

  165. #166 Randerscheinung
    7. September 2018

    Ok, ich sag ja nur, dass es äquivalent wäre ob ich dunkle Materie hinzufüge oder Raum wegnehme. Alles, was mit der dunklen Materie funktioniert funktioniert auch mit der umgekehrten Krümmung. Ebenso funktioniert alles, was mit MOND funktioniert auch mit der umgekehrten Krümmung.
    Meine Idee hat nur ein paar Vorteile:
    Als das Universum noch weniger ausgedehnt war gab es weniger davon.
    Man könnte vielleicht erklären, warum alte Galaxien keine DM haben
    Das alte Rätsel der Unendlichkeit des Universums wäre geklärt.

  166. #167 Randerscheinung
    7. September 2018

    Jetzt mal eine Frage an alle: Gibt es diese Idee nicht schon irgendwo?
    Ist sie vielleicht irgendwo widerlegt?

    Nochmal eine Zusammenfassung:
    Energie-Impuls-Tensor als Quelle des Raums
    Limesbetrachtung m->0 führt zu umgekehrter Krümmung
    Diese erklärt DM-Effekte ohne DM

  167. #168 Frantischek
    7. September 2018

    “ich sag ja nur, dass es äquivalent wäre ob ich dunkle Materie hinzufüge oder Raum wegnehme”

    Wenn ich das bis jetzt richtig verstanden habe: Nein.
    Warum?
    Wir haben eine gut funktionierende und ausgiebig getestete Theorie der Gravitaion.

    Die funktioniert. Die Abweichungen entstehen z.b. in den Aussenbereichen der Galaxien.
    Wenn du den Wirkmechanismus umdreht würden Abweichungen entstehen wo größtenteils Newton gilt.
    Tun sie aber nicht.

    Wie immer ohne Gewähr bis sich wer mit Ahnung dazu geäußert hat.

  168. #169 alex
    7. September 2018

    @Randerscheinung:

    Ok, ich sag ja nur, dass es äquivalent wäre ob ich dunkle Materie hinzufüge oder Raum wegnehme.

    Warum? Soweit ich micht erinnere, ist das bisher einfach nur eine Behauptung deinerseits. Kannst du das in irgend einer Form beweisen oder auch nur plausibel machen? Ich sehe jedenfalls nicht, warum das der Fall sein sollte.

  169. #170 Randerscheinung
    7. September 2018

    MOND ist ja auch ausgiebig getestet und nicht widerlegt.

    Plausibel machen: Dichteänderung des Raums ist innere Krümmung (im Gegensatz zur äußeren in höherer Dimension). Schaut euch mal Verzerrbilder an, oder warpfields (die geben bei Bildverzerrungen an, wo wie verzerrt wird). Sowas schaue ich mir beruflich ständig an.
    Das Bild mit dem Gummituchs führt in die falsche Richtung, nämlich die der äußeren Krümmung.

  170. #171 alex
    7. September 2018

    Ich habe den Eindruck dass wir uns im Kreis drehen.

    Zum letzten Mal: “Raumdichte” ist kein Begriff aus der ART.

    Das Gummituch hat sehr wohl innere Krümmung. (Dass dieses Bild dennoch problematisch ist, hat Martin Bäker vor einiger Zeit auf seinen Blog nebenan beschrieben.) Ich bin mir nicht sicher, ob du wirklich verstanden hast, was mit innerer und äußerer Krümmung gemeint ist.

    Und viel wichtiger: Was hat das mit dem Thema zu tun?

  171. #172 Randerscheinung
    7. September 2018

    Statt warpfields besser “kissenförmige Verzeichnung” (meine Vorstellung der gravitativen Krümmung) und “tonnenförmige Verzeichnung” (meine Vorstellung der ungekehrten Krümmung) googlen, das ist weniger Star Trek- lastig und führt direkt zu dem Bild was ich im Kopf habe. Gut, man müsste sich noch die tonnenförmige Verzeichnung außerhalb vorstellen und stärker werdend nach außen hin. Es äquivalent, ob ich DM-Halos habe oder einen dünner werdenden Raum. Kann man sich das daran vorstellen?

  172. #173 Randerscheinung
    7. September 2018

    Raumdichte ist mein Begriff, ich gebs ja klar zu. Das ist der Versuch, die Sache anschaulich zu machen. Besonders die umgekehrte Krümmung wird meiner Meinung nach damit anschaulich.

    Und ja, man müsste es ausrechnen und an vielen Dingen testen.
    Und ja, ich bin kein Experte der ART.

    Aber es kommt mir so logisch vor, dass ich denke, das muss doch eigentlich einer von den ART- Experten schon mal berechnet haben. Vielleicht bin ich auch jetzt nur deswegen drauf gekommen, weil ich vor Jahren mal so was gelesen habe, mich nicht mehr dran erinnern kann und das jetzt “re-invented” habe. Bin mir diesbezüglich wirklich nicht sicher.

  173. #174 alex
    7. September 2018

    Es äquivalent, ob ich DM-Halos habe oder einen dünner werdenden Raum.

    Eigentlich möchte ich mich nicht dauernd wiederholen. Aber: Warum?

  174. #175 Randerscheinung
    7. September 2018

    Ok, alex, ist mein Bild falsch? Kannst Du mir denn klar machen, warum Raum wegnehmen und Materie hinzufügen NICHT äquivalent sein sollte?
    Und klar ist es komplexer mit der Zeit. Es müsste am Rand (a <a0) das Gegenteil mit der Zeit passieren was in der homogenen Massekugel mit der Zeit passiert.

    Nochmal: Eine umgekehrte Raumzeit-Krümmung am Rand (a <a0) sollte äquivalent sein zu Dark matter -Halos.

  175. #176 alex
    7. September 2018

    ist mein Bild falsch?

    Welches Bild?

    Kannst Du mir denn klar machen, warum Raum wegnehmen und Materie hinzufügen NICHT äquivalent sein sollte?

    Ich weiß nicht mit 100%iger Sicherheit dass das falsch ist. Ich sehe nur keinen offensichtlichen Grund weshalb es wahr sein sollte.

  176. #177 Randerscheinung
    7. September 2018

    Ich sag ja auch nicht, dass es wahr sein muss. Nur eben äquivalent. Also ob ich DM einführe oder Raum wegnehme ist für die Probemasse egal.

  177. #178 alex
    7. September 2018

    Mit “es” in “Ich sehe nur keinen offensichtlichen Grund weshalb es wahr sein sollte.” meinte ich deine Aussage

    dass es äquivalent wäre ob ich dunkle Materie hinzufüge oder Raum wegnehme.

    Warum ist diese Aussage deiner Ansicht nach wahr?

  178. #179 Randerscheinung
    7. September 2018

    Ok gehen wir mal ins 2D, als erste Anschauung. Du hast die Bilder über kissenförmige und tonnenförmige Verzeichnung gefunden?
    Jetzt noch eine Vereinfachung der Vorstellung: Statt Massezentrum eine Gerade (parallel zur y-Achse), links davon homogene Masse, rechts Raum. Wir betrachten zB. mal einen Bereich von 10cm mal gesamte x-Achse.
    In meiner Vorstellung würde dann die Masse bewirken, dass immer weniger Kästchen in den 10 cm sind. Weit weg ändert sich din Anzahl der Kästchen aber kaum noch. Ich könnte sie dann “künstlich verringern” (umgekehrte Krümmung). Das mache ich ab z.B. x=100. Das sieht doch für ein Probeteilchen auf x=110 aus als wäre es nahe einer Masse.

  179. #180 alex
    7. September 2018

    Was genau stellen in diesem Bild die Kästchen dar?

  180. #181 Randerscheinung
    7. September 2018

    Ich bin mir nicht sicher… vielleicht den metrischen Tensor?

  181. #182 Randerscheinung
    7. September 2018

    Jedenfalls ist die Anzahl der Kästchen pro Volumen meine Vorstellung von Raumdichte. Und jedenfalls wäre es eine innere Krümmung ohne äußere, wenn sich die Anzahl der Kästchen verringert.

  182. #183 alex
    7. September 2018

    Dann ist es schwer zu sagen, ob an dem Bild etwas nicht stimmt. Ein anderes grundsätzliches Problem ist dass die ART in zwei Dimensionen nicht wirklich Sinn macht (die linke Seite der Feldgleichungen ist da konstant gleich Null), so dass es nicht immer einfach ist, Dinge aus 2d nach 4d zu übertragen. Und wie gesagt ist für den hier betrachteten Fall der “Raum-Anteil” der Metrik nicht wirklich relevant.

    Aber wie gesagt, wenn dein Vorschlag zur selben Metrik führt wie die gewöhnliche ART mit dunkler Materie, dann kommen selbstverständlich auch die selben Bewegungsgleichungen raus. Es stellen sich nur zwei Fragen: Kommt da tatsächlich die selbe Metrik raus? Und warum kommt gerade diese Metrik raus? Einfach zu postulieren, dass es keine dunkle Materie gibt, dass die Metrik aber trotzdem so aussieht als gäbe es dunkle Materie, hilft ja nicht weiter. Im Idealfall hättest du eine Modifikation der Feldgleichungen, aus der sich dieses Ergebnis ergibt.

  183. #184 Randerscheinung
    7. September 2018

    Ich müsste ja die Feldgleichungen nicht mal modifizieren! (Das gefällt mir besonders) Die einzige wirklich neue Aussage die ich einführe ist “Masse = Quelle des Raums”. Daraus folgt dann, dass die bisherigen Vakuumlösungen “nicht anwendbar” sind.

    Ich denke, dass die gleiche Metrik herauskommt, weil MOND (1/r-Zusammenhang statt 1/r^2) so gut passt und in der homogenen Massekugel ein r-Anstieg stattfindet. Also wäre vorstellbar, dass die umgekehrte Krümmung noch ein r oben erzeugt (metrischer Tensor) und damit dann der Zusammenhang zu 1/r führt.
    Naja, aber das müsste man wirklich mal durchrechnen. Könntest Du das?

  184. #185 alex
    7. September 2018

    Ich müsste ja die Feldgleichungen nicht mal modifizieren!

    Doch. Wenn du die Feldgleichungen nicht modifizierst, kommt heraus dass man dunkle Materie braucht.

    Ich denke, dass die gleiche Metrik herauskommt, weil MOND (1/r-Zusammenhang statt 1/r^2) so gut passt und in der homogenen Massekugel ein r-Anstieg stattfindet. Also wäre vorstellbar, dass die umgekehrte Krümmung noch ein r oben erzeugt (metrischer Tensor) und damit dann der Zusammenhang zu 1/r führt.
    Naja, aber das müsste man wirklich mal durchrechnen. Könntest Du das?

    Wieder einmal verstehe ich nicht was du meinst. Daher muss meine Antwort leider “nein” lauten.

  185. #186 Randerscheinung
    7. September 2018

    Man bräuchte 2 Konstanten: a0 (Wann beginnt die umgekehrte Krümmung) und eine, wie stark die umgekehrte Krümmung tatsächlich ist, nennen wir das mal k. Die Konstante a0 ist ja schon mal aus MOND bekannt. Die Konstante k müsste man dann vielleicht anhand der Rotations-Kurven berechnen.

  186. #187 Randerscheinung
    7. September 2018

    Wenn du die Feldgleichungen nicht modifizierst, kommt heraus dass man dunkle Materie braucht.

    Hä, wieso denn das?

    Ich würde doch nur für m->0 annehmen, dass das Volumen verschwindet und deswegen die umgekehrte Krümmung einführen müssen… Mehr nicht. Funktioniert doch mit den herkömmlichen Feldgleichungen?!

  187. #188 Randerscheinung
    7. September 2018

    Wieder einmal verstehe ich nicht was du meinst. Daher muss meine Antwort leider “nein” lauten.

    Eine Metrik aufstellen meine ich, Schwarzschild-Metrik plus Zusatzterm, so dass V=0 bei m->0

  188. #189 alex
    7. September 2018

    Funktioniert doch mit den herkömmlichen Feldgleichungen?!

    Echt? Die Rechnung die das zeigt würde ich gerne sehen.

  189. #190 Randerscheinung
    7. September 2018

    Übrigens steht im Wiki über die Schwarzschild-Metrik der schöne Satz
    “Als Folge dessen hat eine Kugelschale gegebenen Umfangs in Anwesenheit einer zentralen Masse ein höheres Volumen als in Abwesenheit der Masse.”
    Das ist auch eine Analogie zu: Materie erzeugt erhöhte Raumdichte.

  190. #191 Randerscheinung
    8. September 2018

    Ok, kämpfe mich durch die Literatur um die Idee wiederzufinden.
    1. Deffayet “Nonlocal metric formulation of MOND with sufficient lensing”:
    Was bedeutet dort “nonlocal”? Ist das diese Idee, dass es quantenverschränkte Teilchen außerhalb des Universums gibt die irgendwie Einfluss nehmen? Oder was anderes? Es gibt eine ganze Gruppe solche nonlocal metrics…

  191. #192 Randerscheinung
    8. September 2018

    Next: Famaey 2012, “Modified NewtonIan Dynamics (MOND): Observational phenomenology and relativistic extensions”
    Sehr schöner Artikel übrigens!
    Was ist ein “Generalized Einstein Aether”?

  192. #193 alex
    8. September 2018

    “nonlocal” heißt, dass Dinge nicht nur an jeweils einem Punkt der Raumzeit passieren (und sich dann ggf. von dort aus ausbreiten), sonden dass verschiedene Punkte der Raumzeit in den Gleichungen direkt miteinander “verbunden” sind. Klassische physikalische Theorien sind üblicherweise lokal (bei der Quantenmechanik ist das komplizierter), jedenfalls wenn sie “fundamental” sind. In effektiven Theorien kann es Nichtlokalität schon geben (das könnte man dann z.B. so interpretieren, dass sich Störungen so schnell ausbreiten, dass man die Signallaufzeit ignorieren kann).

    Konkret in dem Paper sieht man das in Gleichung 75. Das ist der MOND-Term einer Lagrange-Dichte einer solchen Theorie. Die Größen X und Y darin sind in den Gleichungen 63 und 64 definiert. Dort steht in einer etwas seltsamen Physiker-Notation 1/□ angewendet auf eine Funktion, was heißt, dass man eine Wellengleichung mit dieser Funktion als Inhomogenität (und gewissen Randbedingungen) lösen muss. Und in einer solchen Lösung hängt der Wert an einem Punkt der Raumzeit von den Werten der Inhomogenität an vielen anderen Punkten ab. D.h. X und Y sind nichtlokale Funktionale der Metrik und damit ist auch die Lagrange-Dichte nichtlokal.

    “Einstein Aether” (ein Begriff dem ich bisher noch nicht begegnet bin) heißt, dass man in der Theorie ein zeitartiges normiertes Vektorfeld hat. Physikalisch heißt das, dass man an jedem Punkt der Raumzeit ein ausgezeichnetes Bezugssystem hat, so ähnlich wie man sich das in dar Äther-Theorie im 19. Jahrhundert gedacht hat. Nur ist dieses Vektorfeld jetzt selbst dynamisch. Man geht dann wohl davon aus, dass im Großen dieses Feld das Ruhesystem der kosmischen Hintergrundstrahlung anzeigt. “generalized” heißt hier dann anscheinend, dass man in die Lagrange-Dichte noch eine Nichtlinearität in Form der “MOND-Funktion” (wie in anderen MOND-Theorien) einbaut.

  193. #194 Randerscheinung
    8. September 2018

    Was ist eigentlich die Lagrange-Dichte?

    Ist die ART lokal (weil nicht quantenmechanisch) – oder nichtlokal, weil die Masse über die Metrik Einflüsse auf den gesamten Raum hat?
    Wie ist das eigentlich bei der Bewegung einer Masse durch den Raum- breitet sich die Raumzeit-Krümmung dann mit Lichtgeschwindigkeit aus oder ist sie gleich “da”?

  194. #195 Randerscheinung
    8. September 2018

    Deine Antworten sind übrigens spitze, es ist ganz toll, dass ich diese Fragen hier stellen kann, vielen Dank dafür, das wollte ich noch sagen!

  195. #196 alex
    8. September 2018

    Was ist eigentlich die Lagrange-Dichte?

    Die Lagrange-Dichte ist ein ziemlich grundlegendes Konzept aus der Feldtheorie. (Und wenn du die nicht kennst, wäre ein Anfänger-Buch über theoretische Mechanik vielleicht besser als direkt mit Modifikationen der ART anzufangen.)

    Man kann in der (klassischen) Physik die Gleichungen vieler Theorien aus dem Hamiltonschen Prinzip ableiten. Das besagt, dass es eine Größe namens Wirkung gibt, und dass die physikalischen Objekte sich so verhalten, dass die Wirkung dabei minimal wird. D.h. man muss nur angeben wie man die Wirkung berechnet und bekommt dann mit ein wenig Variationsrechnung die Differentialgleichungen nach denen sich Teilchen, Felder, etc. verhalten. Und in der Feldtheorie ist die Wirkung üblicherweise das Integral einer skalaren Funktion über die ganze Raumzeit. Diese Funktion heißt Lagrange-Dichte.

    Ein Vorteil dieser Vorgehensweise ist, dass man an der Wirkung (oder der Lagrange-Dichte) leichter ablesen kann, ob die Theorie bestimmte Symmetrien erfüllt, also ob bestimmte Erhaltungssätze gelten. Auch kommt man von der Wirkung über Pfadintegrale ziemlich direkt zur Quantenmechanik (wobei es dann noch nicht klar ist, ob die so definierte Theorie überhaupt wohldefiniert ist). Und die Darstellung einer Theorie über ihre Lagrange-Dichte ist oft viel kürzer als über die Differentialgleichungen.

    Ist die ART lokal (weil nicht quantenmechanisch) – oder nichtlokal, weil die Masse über die Metrik Einflüsse auf den gesamten Raum hat?

    Die ART ist lokal. Nicht weil sie nicht quantenmechanisch ist, sondern weil ihre Grundgleichungen (also die Einsteinschen Feldgleichungen und die Geodätengleichung) nur die Größen an jeweils einem Punkt der Raumzeit zueinander in Bezug stellen. In Einsteinschen Feldgleichungen steht links etwas das sich aus der Metrik und den ersten und zweiten Ableitungen der Metrik an einem bestimmten Punkt berechnet, und rechts steht der Energie-Impuls-Tensor an dem selben Punkt.

    Wie ist das eigentlich bei der Bewegung einer Masse durch den Raum- breitet sich die Raumzeit-Krümmung dann mit Lichtgeschwindigkeit aus oder ist sie gleich “da”?

    Sie breitet sich mit Lichtgeschwindigkeit aus; im Grunde sind genau das Gravitationswellen.

  196. #197 Randerscheinung
    8. September 2018

    Ich will ja nicht die ART modifizieren. Meine kleine mini-Idee ist ja nur, dass vieleicht die Vakuum-Lösungen nicht stimmen würden (nicht anwendbar wären) wenn Masse=Quelle des Raums wäre (und Raum nicht ohne Masse existieren würde)… Findest du das gar nicht plausibel?
    Hast du schon mal von irgendwas ähnlichem gehört?
    Warum glaubst du eigentlich, dass man unbedingt die Feldgleichungen modifizieren müsste um eine Metrik aufzustellen, die das macht?

  197. #198 alex
    8. September 2018

    Ich will ja nicht die ART modifizieren.

    Ob du willst oder nicht, das wirst du tun müssen (siehe weiter unten).

    Meine kleine mini-Idee ist ja nur, dass vieleicht die Vakuum-Lösungen nicht stimmen würden (nicht anwendbar wären) wenn Masse=Quelle des Raums wäre (und Raum nicht ohne Masse existieren würde)… Findest du das gar nicht plausibel?

    Ich bin mir immer noch nicht sicher, ob ich verstehe was genau das bedeutet.

    Wenn ich dich richtig verstehe möchtest du, dass sich in der Umgebung (wie auch immer man das genau definiert) einer jeden Massenverteilung ein gewisses endliches Volumen an Raum befindet. Dabei würde die Größe dieses Volumens von der Gesamtmasse abhängen; wie genau und ob auch die Anordnung der Masse eine Rolle spielt, wäre noch zu klären. Und dass es kein Außerhalb dieses Volumen gibt (es sei denn, es gibt auch mehr Masse).

    Unter der Annahme, dass das in etwa deine Idee trifft, folgendes Gedankenexperiment: Man startet mit zwei Massen die nah beieinander sind, so dass ihre Volumina überlappen. Außer diesen beiden soll es keine weiteren Massen geben. Nun beschleunigt man eine davon über die relative Fluchtgeschwindigkeit hinaus. Was passiert nun?

    Werden die Volumina der beiden Massen irgendwann nicht mehr überlappen? Wenn ja, heißt das, dass zu diesem Zeitpunkt das Universum in zwei nicht zusammenhängende Teile zerfällt? Wenn ja, wäre das umkehrbar, wenn ja, wie? (Die geflohene Masse abzubremsen und umzukehren macht jedenfalls in der ART keinen Sinn, da Geschwindigkeit immer nur relativ zu verstehen ist, und es in diesem Teil des Universums keine weitere Masse gibt.) Wenn das nicht umkehrbar ist, wäre das alleine schon ein Grund, weshalb eine Modifikation der Feldgleichungen notwendig ist. Denn wie alle anderen bekannten fundamentalen physikalischen Theorien ist auch die ART zeitumkehrinvariant.

    Falls die beiden Volumina nicht getrennt werden, wie soll das funktionieren? Verformen sie sich zu einer Art Röhre? Oder gibt es eine Art Maximalentfernung der beiden Massen die nicht überschritten werden kann (und die vielleicht nur asymptotisch erreicht wird)? Wenn ja, wie funktioniert das? Und was wäre wenn man drei Massen nimmt und sie zunächst in einer Reihe anordnet, so dass die erste von der zweiten und die zweite von der dritten in etwa in der Maximalentfernung entfernt ist (oder geht das nicht? wenn ja warum?) und dann die mittlere zur ersten verschiebt? Trennt das die dritte Masse ab, oder würde die mittlere die dritte “mitziehen”? Wenn ja, warum?

    Etc. pp.

    Hast du schon mal von irgendwas ähnlichem gehört?

    Nein. Es hat eine gewisse entfernte Ähnlichkeit mit dem Machschen Prinzip.

    Warum glaubst du eigentlich, dass man unbedingt die Feldgleichungen modifizieren müsste um eine Metrik aufzustellen, die das macht?

    Wenn ich dich richtig verstehe, möchtest du (u.a.), dass man im kugelsymmetrischen Fall außen eine Metrik bekommt, die nicht die äußere Schwarzschild-Metrik ist; zumindest für ausreichend große Abstände. Aber mit den Einsteinschen Feldgleichungen erhält man zwingend die Schwarzschild-Metrik, wenn man nur Zeitunabhängigkeit, Kugelsymmetrie, und Vakuum (und kein elektrisches Feld) annimmt. Wenn du also für diesen Fall, also eine kugelsymmetrische und sich zeitlich nicht ändernde Massenverteilung die alleine im Vakuum sitzt, etwas anderes als die Schwarzschild-Metrik willst, musst du die Feldgleichungen ändern.

  198. #199 Randerscheinung
    8. September 2018

    …möchtest du, dass sich in der Umgebung (wie auch immer man das genau definiert) einer jeden Massenverteilung ein gewisses endliches Volumen an Raum befindet.

    Ja genau, ein gewisses endliches Volumen. Das ist der entscheidende Punkt. Aber dieses endliche Volumen wird nicht dadurch endlich, dass es irgendwo aufhört. Sondern dadurch, dass die Raumdichte nach außen hin immer weiter abnimmt – und das meinetwegen bis in die Unendlichkeit.

    Dabei würde die Größe dieses Volumens von der Gesamtmasse abhängen; wie genau und ob auch die Anordnung der Masse eine Rolle spielt, wäre noch zu klären. Und dass es kein Außerhalb dieses Volumen gibt (es sei denn, es gibt auch mehr Masse).

    Genau! D.h. eigentlich würde ich sagen, es gibt auch mit zusätzlicher Masse kein “außerhalb”, sondern die Räume fügen sich dann zusammen. Wenn dazwischen a <a0 herrscht dann ist nur der Raum dort "dünner".

    … so dass ihre Volumina überlappen.

    Ich würde sagen, sie überlappen immer. Wie zwei (entgegengesetzte) Hyperbeln die entlang der x-Achse zueinander verschoben sind, sich immer überlappen, unabhängig vom Abstand ihrer Symmetriepunkte.

    Werden die Volumina der beiden Massen irgendwann nicht mehr überlappen?

    Nein, sie überlappen immer. Das nehme ich auch deshalb an, weil wir kein Ende des Universums sehen, sondern überall homogenes Universum, so weit wir sehen können.

    Falls die beiden Volumina nicht getrennt werden, wie soll das funktionieren? Verformen sie sich zu einer Art Röhre? Oder gibt es eine Art Maximalentfernung der beiden Massen die nicht überschritten werden kann (und die vielleicht nur asymptotisch erreicht wird)?

    Auch eine sehr interessante Frage! Wenn ich die beiden Hyperbeln von oben (deren Summe die tatsächliche Raumdichte sein soll) immer weiter voneinander entferne… Wird der Raum dazwischen (Raumdichte ~ y-Wert der Summenfunktion) immer dünner und trotzdem der Abstand größer. Der tatsächliche Abstand wäre das Integral unter der Kurve. Konvergiert das für “Entfernung der Hyperbeln” gegen unendlich? Ich würde sagen, nein, weil man durch Vergrößerung der Entfernung zwischen den Parabeln immer auch wieder Fläche unter der Kurve hinzufügt. Das heißt, der Abstand zwischen den beiden Massen wird immer größer und der Raum wird immer dünner. Es gibt keine Maximalentfernung.

    Aber mit den Einsteinschen Feldgleichungen erhält man zwingend die Schwarzschild-Metrik, wenn man nur Zeitunabhängigkeit, Kugelsymmetrie, und Vakuum (und kein elektrisches Feld) annimmt.

    Ich würde halt nur das Vakuum weglassen, das kann es nach meiner einzigen neuen Idee (Masse= Quelle des Raums) nicht geben. Kugelsymmetrie, Zeitsymmetrie und Feldgleichungen können alle so bleiben wie gehabt. Einzig die Vakuum-Lösung für den flachen Raum ergibt keinen Sinn dabei.
    Die Schwarzschild-Metrik muss an den Rändern zur Vakuum-Lösung für den flachen Raum werden, weil sie voraussetzt , dass ohne die Masse der flache Raum da wäre. Der Einfluss der Masse geht an den Rändern zurück. Wenn man stattdessen voraussetzt, dass ohne Masse auch kein Raum da wäre muss dieser Raum an den Rändern immer dünner werden.

    Ein bisschen komisch ist die Vorstellung schon, wenn man zum Himmel schaut und sich denkt, dass es zwischen den Galaxien solch “dünnen” Raum geben könnte. Oder sich Gravitationswellen durch dieses “Universum mit variabler Raumdichte” stabil bewegen sollen. Aber dann wieder auch nicht, der Raum wäre ja nicht “instabil” deswegen.
    Es würde auch Auswirkungen auf die dunkle Energie haben. Aber immerhin wurde ja schon ein Zusammenhang zwischen der kosmologischen Konstante und a0 gefunden.
    Ich finde es immer logischer, je länger ich drüber nachdenke. Zumindest, dass es äquivalent ist zur DM.

    Was denkst du?

  199. #200 alex
    9. September 2018

    Aber dieses endliche Volumen wird nicht dadurch endlich, dass es irgendwo aufhört. Sondern dadurch, dass die Raumdichte nach außen hin immer weiter abnimmt – und das meinetwegen bis in die Unendlichkeit.

    Was bedeutet das? Oder anders ausgedrückt: Wie unterscheidet man diese beiden Arten von endlichem Volumen experimentell? Wenn man mal der Einfachheit halber von Kugelsymmetrie ausgeht, ist endliches Volumen nur machbar, wenn es eine obere Schranke für die physikalische Entfernung vom Zentrum gibt.

    Wenn ich die beiden Hyperbeln von oben (deren Summe die tatsächliche Raumdichte sein soll) immer weiter voneinander entferne… Wird der Raum dazwischen (Raumdichte ~ y-Wert der Summenfunktion) immer dünner und trotzdem der Abstand größer. Der tatsächliche Abstand wäre das Integral unter der Kurve. Konvergiert das für “Entfernung der Hyperbeln” gegen unendlich? Ich würde sagen, nein, weil man durch Vergrößerung der Entfernung zwischen den Parabeln immer auch wieder Fläche unter der Kurve hinzufügt. Das heißt, der Abstand zwischen den beiden Massen wird immer größer und der Raum wird immer dünner. Es gibt keine Maximalentfernung.

    Das kann ich nicht nachvollziehen. Was genau geben die Hyperbeln an? Also was genau ist die x-Koordinate und was die y-Koordinate im Graphen?

    Ich würde halt nur das Vakuum weglassen, das kann es nach meiner einzigen neuen Idee (Masse= Quelle des Raums) nicht geben.

    “Vakuum” heißt hier nur, dass der Energie-Impuls-Tensor gleich Null ist. Wenn ich mich richtig erinnere, hast du weiter oben gesagt, dass du keine neue Art Materie/Energie/… einführen willst. Also muss der Energie-Impuls-Tensor doch im Außenraum gleich Null sein. Wenn du willst, kann ich statt “Vakuum” auch “Abwesenheit von Materie” sagen. Dann lautet mein Satz von oben:

    Aber mit den Einsteinschen Feldgleichungen erhält man zwingend die Schwarzschild-Metrik, wenn man nur Zeitunabhängigkeit, Kugelsymmetrie, und Abwesenheit von Materie (und kein elektrisches Feld) annimmt.

    Die Schwarzschild-Metrik muss an den Rändern zur Vakuum-Lösung für den flachen Raum werden, weil sie voraussetzt , dass ohne die Masse der flache Raum da wäre.

    Nein, das ist einfach nicht wahr. Wie ich bereits sagte, ist es eine Grundvoraussetzung der ART, dass die Raumzeit existiert. Wenn du das ändern willst, musst du mindestens die Feldgleichungen der ART ändern. Möglicherweise aber viel mehr. Das hat auch nichts mit der Schwarzschild-Metrik per se zu tun. Ich kann mich nur wiederholen: Wenn man die Feldgleichungen der ART für den kugelsymmetrischen Fall ohne Materie aufschreibt, bekommt man ein System von Differentialgleichungen das nur die äußere Schwarzschild-Metrik als Lösung hat.

    Was denkst du?

    Sorry, aber ich denke das ganze ist nicht besonders gut durchdacht und auf jeden Fall nicht kompatibel mit der ART. Und dass es äquivalent zu MOND oder zur dunklen Materie ist, halte ich für extrem unwahrscheinlich.

  200. #201 Randerscheinung
    9. September 2018

    Wenn man mal der Einfachheit halber von Kugelsymmetrie ausgeht, ist endliches Volumen nur machbar, wenn es eine obere Schranke für die physikalische Entfernung vom Zentrum gibt.

    Ja, klar. Der Raum breitet sich unendlich aus, aber mit immer kleiner werdender Raumdichte. Die Frage nach dem Limes ist, geht die Raumdichte schneller gegen 0 als das Volumen gegen unendlich?

    Was genau geben die Hyperbeln an? Also was genau ist die x-Koordinate und was die y-Koordinate im Graphen?

    x-Koordinate: sowas wie die Entfernung die wir annehmen, weil wir die Vakuumlösungen für richtig halten. Es wäre die Entfernung in Kästchen (siehe Kästchen-Analogie) der Größe 1. Wir denken(CMD), diese Funktion sind nicht zwei Hyperbeln, sondern es bleibt dazwischen immer auf 1…

    Also muss der Energie-Impuls-Tensor doch im Außenraum gleich Null sein

    Das ist der entscheidende Punkt: Es gibt keinen Außenraum. Das ist schwer vorstellbar, aber läuft nur darauf hinaus, dass die Raumdichte schneller gegen null geht als das Volumen.
    Was denkst Du damit?

  201. #202 Randerscheinung
    9. September 2018

    Was genau geben die Hyperbeln an? Also was genau ist die x-Koordinate und was die y-Koordinate im Graphen?

    x-Koordinate: sowas wie die Entfernung die wir annehmen, weil wir die Vakuumlösungen für richtig halten. Es wäre die Entfernung in Kästchen (siehe Kästchen-Analogie) der Größe 1. Wir denken(CMD), diese Funktion sind nicht zwei Hyperbeln, sondern es bleibt dazwischen immer auf 1…

    y-Koordinate: Raumdichte. Das Integral unter der Kurve ist der tatsächliche Abstand.

    Die Schwarzschild-Metrik muss an den Rändern zur Vakuum-Lösung für den flachen Raum werden, weil sie voraussetzt , dass ohne die Masse der flache Raum da wäre.

    Nein, das ist einfach nicht wahr. Wie ich bereits sagte, ist es eine Grundvoraussetzung der ART, dass die Raumzeit existiert.

    Okok, sie würde ja existieren. Das mit dem “nicht existieren” war noch nicht so durchdacht. Nur das Volumen müsste für m->0 gegen 0 gehen. Solange m>0 ist würde die Raumzeit sich immer noch bis in die Unendlichkeit erstrecken. Nur eben die Dichte nimmt immer weiter ab.
    Das ist doch gar nicht so schwer zu verstehen… In meinem Kopf ist das Bild ganz klar…

    auf jeden Fall nicht kompatibel mit der ART. Und dass es äquivalent zu MOND oder zur dunklen Materie ist, halte ich für extrem unwahrscheinlich

    Warum denn? Mir kommt es so logisch vor…

  202. #203 alex
    9. September 2018

    Die Frage nach dem Limes ist, geht die Raumdichte schneller gegen 0 als das Volumen gegen unendlich?

    Ja was denn nun? Willst du endliches Volumen oder unendliches Volumen? Wie ich bereits schrieb, ist es leicht zu zeigen, dass im kugelsymmetrischen Fall endliches Volumen nur mit einer endlichen Schranke für die Entfernung machbar ist. Das ist ziemlich elementare Mathematik (eine simple Abschätzung eines Integrals durch ein anderes). Willst du wirklich dagegen argumentieren?

    die Entfernung die wir annehmen, weil wir die Vakuumlösungen für richtig halten

    Was soll das bedeuten? Wie misst man diese Größe experimentell? Wenn sie nicht experimentell messbar ist, sehe ich nicht welchen Sinn es hat, sich mit ihr zu beschäftigen.

    Das ist der entscheidende Punkt: Es gibt keinen Außenraum.

    Aber es gibt Raum in dem keine Materie ist. Das ist was ich mit “Außenraum” meine. Oder meinst du jetzt, dass das ganze Universum komplett voll von Materie ist?

    y-Koordinate: Raumdichte

    Bitte, ein allerletztes Mal: Definiere was “Raumdichte” sein soll. Oder besser, verwende diesen Begriff nicht mehr.

    Solange m>0 ist würde die Raumzeit sich immer noch bis in die Unendlichkeit erstrecken.

    Was soll dann “Unendlichkeit” heißen? Ja wohl nicht, dass eine gerade physikalische Strecke beliebig großer Länge oder ein beliebig großes physikalisches Volumen darin Platz fände.

    In meinem Kopf ist das Bild ganz klar…

    Vielleicht ist dein Bild davon wie Raumzeitgeometrie in der ART “funktioniert” einfach falsch.

    Warum denn?

    Die Frage kann ich direkt an dich zurück stellen. Du versuchst mit deiner Idee ein bestimmtes Phänomen zu erklären. Dann ist es deine Aufgabe darzulegen, warum deine Idee das zu leisten instande ist. Und das hast du bisher einfach nicht getan. Von Carl Sagan gibt es diesen schönen Spruch: “Extraordinary claims require extraordinary evidence”. Und du machst hier mehrere “extraordinary claims” ohne “extraordinary evidence” zu liefern.

  203. #204 Randerscheinung
    9. September 2018

    Die Frage nach dem Limes ist, geht die Raumdichte schneller gegen 0 als das Volumen gegen unendlich?

    Ja was denn nun? Willst du endliches Volumen oder unendliches Volumen?

    ok, mein Fehler: hier habe ich zweimal das Wort Volumen in verschiedenem Kontext verwendet. Das Raumvolumen soll konvergieren. Das Volumen unter der Kurve der zwei Hyperbeln (Intergral der Funktion) ist der Abstand. Der Abstand geht gegen unendlich, das volumen ist aber trotzdem begrenzt. Ich hätte an dieser Stelle “Die Frage nach dem Limes ist, geht die Raumdichte schneller gegen 0 als der Abstand gegen unendlich? ”

    Wie ich bereits schrieb, ist es leicht zu zeigen, dass im kugelsymmetrischen Fall endliches Volumen nur mit einer endlichen Schranke für die Entfernung machbar ist. Das ist ziemlich elementare Mathematik (eine simple Abschätzung eines Integrals durch ein anderes). Willst du wirklich dagegen argumentieren?

    Naja, im euklidischen Raum gilt das natürlich. Aber gilt es in jedem beliebig gekrümmten Raum auch? Ich würde sagen, nein. Man kann sich Krümmungen vorstellen, für die das nicht zutrifft. Stimmst du dabei mit mir überein?

    Aber es gibt Raum in dem keine Materie ist. Das ist was ich mit “Außenraum” meine. Oder meinst du jetzt, dass das ganze Universum komplett voll von Materie ist?

    Nein, dass ist ja grade die einzig neue Grundannahme meiner Idee: Dass es eben keinen Raum gibt, in dem keine Materie ist (=der von jeglicher Materie abgeschnitten ist!). Das ist ganz wichtig, um die Idee zu verstehen. Natürlich gibt es Raum ohne Materie, zwischen den Sternen, Galaxien usw. Aber dort ist (in meiner Theorie) immer noch Einfluss von Materie vorhanden, es gibt immer eine Bescheunigung a>0. So kann man “Masse = Quelle des Raums” alternativ formulieren: Es gibt keinen Raum, für den a=0 gilt. Im Gegensatz dazu beschreibt die Vakuumlösung der Feldgleichungen doch den Raum in dem a=0 gilt, stimmt das nicht? Das wäre erst mal der einzige Unterschied zum Standardmodell. Alles andere folgt daraus…

    Solange m>0 ist würde die Raumzeit sich immer noch bis in die Unendlichkeit erstrecken.

    Was soll dann “Unendlichkeit” heißen? Ja wohl nicht, dass eine gerade physikalische Strecke beliebig großer Länge oder ein beliebig großes physikalisches Volumen darin Platz fände.

    Eine beliebig große Strecke fände darin Platz. Aber nicht ein beliebig großes Volumen. Ich erkenne ein Problem: Volumen = Strecke ^3…

  204. #205 alex
    9. September 2018

    Naja, im euklidischen Raum gilt das natürlich. Aber gilt es in jedem beliebig gekrümmten Raum auch? Ich würde sagen, nein. Man kann sich Krümmungen vorstellen, für die das nicht zutrifft. Stimmst du dabei mit mir überein?

    Vorstellen kann man sich viel. Wenn ich mich anstrenge, kann ich mir vielleicht auch vorstellen, dass in den natürlichen Zahlen 1 + 1 = 3 gilt. Aber mathematisch möglich ist es trotzdem nicht. Wie gesagt, es ist ziemlich einfach das zu zeigen, deshalb würde ich nicht davon ausgehen, dass mein Argument fehlerhaft ist. Falls du trotzdem anderer Meinung bist, würde ich dich bitten, einen Raum mit dieser Eigenschaft explizit anzugeben. Andernfalls wäre es schön, wenn wir diesen sinnlosen Teil unserer Diskussion hier beenden könnten.

    Natürlich gibt es Raum ohne Materie

    Und genau das ist der “Außenraum” von dem ich schrieb.

    Es gibt keinen Raum, für den a=0 gilt. Im Gegensatz dazu beschreibt die Vakuumlösung der Feldgleichungen doch den Raum in dem a=0 gilt, stimmt das nicht?

    In der ART ist die Beschleunigung die auf einen Probekörper wirkt nicht nur abhängig davon, an welchem Punkt der Raumzeit dieser sich befindet. Daher macht diese Aussage in der ART so keinen Sinn. (Außerdem ist die Beschleunigung kein Skalar und damit vom Koordinatensystem abhängig. Das alleine lässt schon sehr am Sinn deiner Aussage zweifeln.)

    Eine beliebig große Strecke fände darin Platz. Aber nicht ein beliebig großes Volumen.

    Wie ich bereits mehrmals sagte, ist das mit Kugelsymmetrie nicht vereinbar.

  205. #206 Randerscheinung
    9. September 2018

    Ok… Für meinen Versuch eines Ansatzes zu einer Herleitung einer umgekehrten Krümmung bräuchte ich V->0 für m-> 0. Dafür bräuchte man ein endliches Volumen. Das gibt es nicht, wenn man sich unendlich weit voneinander entfernen kann. Verstehe so weit, diese Herleitung funktioniert schon mal so nicht.

    Aber: Was wir sehen ist ja, dass das Universum auf großen Skalen homogen bis zum Ende des Sichtbaren ist. D.h. die Frage nach der Entfernung von zwei Massepunkten bis ins Unendliche ist nur rein hypothetisch. Es gibt weder zwei unendlich weit entfernte Massepunkte noch das Vakuum aus den Vakuumlösungen(!!) in unserem sichtbaren Universum.
    Man könnte also ruhig annehmen, dass sich die zwei Massepunkte aus deinem Gedankenexperiment eben nicht unendlich weit voneinander entfernen dürfen. Dass ihr Abstand also nur asymptotisch gegen die größte Entfernung geht. Ohne einen Widerspruch zum sichtbaren Universum zu haben.

    Jetzt mal ohne Herleitung:
    Könnte nicht eine umgekehrte Krümmung der Raumzeit das gleiche bewirken wie die dunkle Materie? Zu dem Kästchen Modell in 2D hast du immerhin geschrieben, dass man es tatsächlich nicht unterscheiden könnte. Das komische ist für mich ja nur, dass niemand auch nur darüber nachdenkt.

  206. #207 alex
    10. September 2018

    D.h. die Frage nach der Entfernung von zwei Massepunkten bis ins Unendliche ist nur rein hypothetisch.

    Wir treiben hier ja theoretische Physik (oder versuchen es zumindest). Da hat man es notwendigerweise immer mit Vereinfachungen und damit hypothetischen Fragen zu tun.

    Man könnte also ruhig annehmen, dass sich die zwei Massepunkte aus deinem Gedankenexperiment eben nicht unendlich weit voneinander entfernen dürfen.

    Gut. Gibt es einen “Mechanismus” der das erklärt, oder ist das einfach ein Postulat? Denn eine solche Maximalentfernung hätte ja auch viele weitere Auswirkungen. Z.B. auf die Energie- und Impulserhaltung, Lorentzinvarianz, Konstanz der Lichtgeschwindigkeit, etc.

    Und was passiert dann in der Situation mit den drei Massen in einer Reihe?

    Dass ihr Abstand also nur asymptotisch gegen die größte Entfernung geht. Ohne einen Widerspruch zum sichtbaren Universum zu haben.

    Klar, wenn du die Maximalentfernung bzw. das Volumen pro Masse nur hoch genug ansetzt. Die Frage ist dann, ob man mit den dafür nötigen Werten den zu erklärenden Effekt noch erklären kann (angenommen dass das überhaupt geht, was ich wie gesagt stark bezweifle). Um einen signifikanten Einfluss auf die Dynamik in einer Galaxie zu haben, sollte die Maximalentfernung für eine durchschnittliche Galaxienmasse in etwa von der Größenordnung der Ausdehnung einer Galaxie sein (kleiner natürlich nicht, aber auch nicht sehr viel größer). Um keinen Widerspruch zu den beobachteten Eigenschaften des sichtbaren Universums als ganzem zu haben, sollte die Maximalentfernung für die Masse des sichtbaren Universums mindestens von der Größenordnung der Ausdehnung des sichtbaren Universums sein. Wenn alles einigermaßen linear ist, wäre das doch nur miteinander vereinbar, wenn das Universum einigermaßen dicht mit Galaxien vollgepackt ist. Und das ist bekanntlich nicht der Fall.

    Und dann wäre da noch das Problem mit der Expansion des Universums.

    Zu dem Kästchen Modell in 2D hast du immerhin geschrieben, dass man es tatsächlich nicht unterscheiden könnte.

    Auf was genau beziehst du dich da? Ich kann mich nicht erinnern, das so gesagt zu haben. Am ähnlichsten scheint mir das noch zu meiner Aussage zu sein, dass falls du die selbe Metrik produzierst, du auch die selben Bewegungen für Probekörper bekommst. Aber das ist ein sehr großes “falls”.

    Das komische ist für mich ja nur, dass niemand auch nur darüber nachdenkt.

    Naja, selbst nach fast einer Woche und etwa 100 Kommentaren habe ich immer noch nicht verstanden, was du eigentlich meinst, und warum du meinst, dass das äquivalent zur dunklen Materie sei.

    Dunkle Materie und MOND sind im Vergleich dazu sehr einfach zu erklären. Ein paar Sätze und vielleicht eine handvoll Formeln sollten ausreichen, so dass jeder einigermaßen physikalisch Vorgebildete die grundlegenden Ideen (und warum diese den Effekt erklären) verstehen kann.

    Ich weiß nicht, ob deine Idee so extrem viel komplizierter ist, oder ob ich besonders schwer von Begriff bin, oder …

  207. #208 Randerscheinung
    10. September 2018

    Man könnte also ruhig annehmen, dass sich die zwei Massepunkte aus deinem Gedankenexperiment eben nicht unendlich weit voneinander entfernen dürfen.

    Gut. Gibt es einen “Mechanismus” der das erklärt, oder ist das einfach ein Postulat? Denn eine solche Maximalentfernung hätte ja auch viele weitere Auswirkungen. Z.B. auf die Energie- und Impulserhaltung, Lorentzinvarianz, Konstanz der Lichtgeschwindigkeit, etc.

    Naja, schon irgendwie ein Postulat, aus der Beobachtung des Universums. Wie gesagt sehen wir ja im Universum weder die Vakuumlösungen noch einen Raum in dem es keine Gravitationswirkungen gibt. Könnte man da nicht sagen, dass das Universum dieses Problem eben genau so löst, dass es weder einen ganz leeren Raum im Sinne der Vakuumlösungen noch eine Überschreitung des maximalen Abstands geben darf?

    Dass ihr Abstand also nur asymptotisch gegen die größte Entfernung geht. Ohne einen Widerspruch zum sichtbaren Universum zu haben.

    Klar, wenn du die Maximalentfernung bzw. das Volumen pro Masse nur hoch genug ansetzt. Die Frage ist dann, ob man mit den dafür nötigen Werten den zu erklärenden Effekt noch erklären kann (angenommen dass das überhaupt geht, was ich wie gesagt stark bezweifle). Um einen signifikanten Einfluss auf die Dynamik in einer Galaxie zu haben, sollte die Maximalentfernung für eine durchschnittliche Galaxienmasse in etwa von der Größenordnung der Ausdehnung einer Galaxie sein (kleiner natürlich nicht, aber auch nicht sehr viel größer).

    Hm… naja, das wären irgendwie zwei Effekte: Einerseits betrachtet man den Bereich, in dem die “umgekehrte Krümmung” losgeht, dort gibt es einen Effekt auf die Dynamik der Galaxie (der äußeren Sterne), der wie MOND aussieht. Das wäre so Größenordnung der Galaxie. Andererseits müsste es eben wegen dem immer dünner werdenden Raum und der Konvergenz “hinter” dem Außenbereich doch immer wieder wieder eine neue Masse geben. Und diese Größenordnung könnte ja beliebig (groß, nur eben endlich) sein, die ist eben so wie sie ist. Auf großen Skalen ist unser Universum ja homogen.

    falls du die selbe Metrik produzierst, du auch die selben Bewegungen für Probekörper bekommst. Aber das ist ein sehr großes “falls”.

    Ok, klar, ein großes “falls”. Aber man könnte versuchen, eine solche Metrik aufzustellen.
    Ich habe nirgends gelesen, dass es jemand versucht hätte und nachgewiesen hat, dass es nicht geht. Oder nicht mit den Beobachtungen übereinstimmt.

    Naja, selbst nach fast einer Woche und etwa 100 Kommentaren habe ich immer noch nicht verstanden, was du eigentlich meinst, und warum du meinst, dass das äquivalent zur dunklen Materie sei.

    Echt? Das Gefühl hatte ich nicht. Ich hatte das Gefühl, das es schon vorwärts geht. Eben weil Du geschrieben hast z.B. dass “falls du die selbe Metrik produzierst, du auch die selben Bewegungen für Probekörper bekommst”. In dem Moment hatte ich das Gefühl, du hattest zumindest das grundlegende Bild verstanden. Dann hast Du gefragt, wo diese Metrik denn herkommen könnte und ich habe mich immer tiefer verstrickt in dem Versuch, sie herzuleiten. Und fehlerhafte Dinge gesagt. Aber es bleibt doch dabei, dass solch eine Metrik äquivalent wäre zur dunklen Materie. Äquivalent heißt, sie erzeugt die gleichen Bewegungen.

    Ich versuche jetzt noch mal eine Definition der Raumdichte: In Wikipedia steht ja (hatte ich schon oben zitiert), dass das Volumen bei gleichem Umfang größer ist, wenn sich innen Masse befindet. Das bedeutet, die Raumdichte ist dort größer als die Raumdichte in der Vakuumlösung der Feldgleichungen, die definiere ich mal als 1. Die Raumdichte mit Masse ist größer 1.
    Also Raumdichte = V1 / V0 * U0 / U1
    dabei wäre
    V1=Volumen im gekrümmten Raum
    V0=Volumen in der Vakuumlösung
    U0=Umfang in der Vakuumlösung
    U1=Unfang im gekrümmten Raum

    Und nun wird eben die Raumdichte weit weg vom Massezentrum kleiner 1. Das ist problematisch mit der Definition oben, weil ja Teile des Umfangs auch im “dünnen Raum” liegen. Ich weiß also auch nicht.

  208. #209 alex
    10. September 2018

    Wie gesagt sehen wir ja im Universum weder die Vakuumlösungen noch einen Raum in dem es keine Gravitationswirkungen gibt.

    Aber wir sehen Raum der extrem leer, also extrem nah am idealen Vakuum ist, und wir sehen dass der Raum auf großen Skalen sehr flach ist.

    Aber man könnte versuchen, eine solche Metrik aufzustellen.

    Die Metrik aufzustellen ist doch kein Problem. Nach Voraussetzung ist das ja genau die Metrik, die man in der ART mit dunkler Materie erhält. Das Problem ist, wie kommt man auf diese Metrik ohne dunkle Materie. Und (s.u.) wie ist diese Metrik mit deinen Ideen vereinbar.

    Echt? Das Gefühl hatte ich nicht. Ich hatte das Gefühl, das es schon vorwärts geht. Eben weil Du geschrieben hast z.B. dass “falls du die selbe Metrik produzierst, du auch die selben Bewegungen für Probekörper bekommst”. In dem Moment hatte ich das Gefühl, du hattest zumindest das grundlegende Bild verstanden.

    Die Metrik die die ART mit dunkler Materie liefert hat keine “umgehehrte Krümmung”, sie hat kein endliches Volumen, und sie hat keine Maximalentfernung. Dafür hat sie (nach den Feldgleichungen, die du ja nicht modifizieren willst) mehr Materie in einer Galaxie als sichtbar ist, also dunkle Materie.

    Du hast zwei Möglichkeiten:

    Entweder du nimmst die selbe Metrik wie in der ART mit dunkler Materie. Dann musst du erklären, warum die Raumzeit gerade diese Metrik hat, die aussieht als wäre da mehr Materie als sichtbar ist. Und du musst die Ideen von umgekehrter Krümmung und endlichem Volumen aufgeben. Dafür bekommst du aber trivialerweise die selben (d.h. die richtigen) Bewegungen von Probekörpern.

    Oder du nimmst eine Metrik, die umgekehrte Krümmung, endliches Volumen, und Maximalentfernung hat. Dann musst du ebenfalls erklären, warum die Raumzeit gerade diese Metrik hat. Und du musst zeigen, dass man die selben (oder jedenfalls ausreichend ähnliche) Bewegungen von Probekörpern bekommt.

    Also Raumdichte = V1 / V0 * U0 / U1
    dabei wäre
    V1=Volumen im gekrümmten Raum
    V0=Volumen in der Vakuumlösung
    U0=Umfang in der Vakuumlösung
    U1=Unfang im gekrümmten Raum

    Volumen und Umfang von was?

  209. #210 Randerscheinung
    10. September 2018

    Volumen und Umfang von was?

    Volumen und zugehöriger Umfang einer Kugel, mit z.B. genormtem Umfang. Das funktioniert für die homogene Massekugel, Scharzschild-Metrik und so, dort wäre dann die Raumdichte mit Masse größer als 1, größer als der flache Raum. Leider funktioniert das nicht so ganz für meine Idee von der umgekehrten Krümmung, weil die “außerhalb” liegen soll und damit ein Stück Umfang auch immer außen liegt, die Kugel verzerrt wäre. Hm… Da es ja einen Maximalabstand geben soll lege ich die gleiche Kugel nun um den halben Maximalabstand herum (“Mittelpunkt des dünnen Raums”, brauche dafür diesbezüglich Kugelsymmetrie). Dann soll jetzt diese Kugel eine Raumdichte kleiner als 1 haben.

    Du hast zwei Möglichkeiten: …

    Das ist eine schöne Aufstellung. Und siehst du, schon fühle ich mich verstanden.

    Wenn Du dachtest, dass ich das ganze Universum erklären will dann hast Du mich sowieso falsch verstanden. Das ist die einzige Idee die ich hatte: Dass es eben auch eine Metrik mit umgekehrter Krümmung geben könnte, die die gleichen Bewegungsgleichungen erzeugt. Na gut, dafür braucht man dann endliches Volumen und Maximalabstand, das hast Du mir aufgezeigt, aber auch das wäre ja vereinbar mit den Beobachtungen.

    Die dunkle Materie kann die Gültigkeit von MOND nicht erklären. Und nicht den Zusammenhang zwischen Lambda und a0. Die umgekehrte Krümmung vielleicht schon.

    Na gut, und es handelt sich hier nur um rein qualitative Betrachtungen. Man müsste das natürlich mal berechnen und für viele Beispiele schauen, ob es immer wieder passt. Klar. Aber warum macht das keiner?

    Aber wir sehen Raum der extrem leer, also extrem nah am idealen Vakuum ist, und wir sehen dass der Raum auf großen Skalen sehr flach ist.

    “Auf großen Skalen” bedeutet hier so Universums-Größenordnungen. Das wäre vereinbar mit meiner Vorstellung der umgekehrten Krümmung: Die Masseverteilung im Universum ist ja (auf großen Skalen) homogen, und die Bereiche mit “dünnem Raum” könnten auch auf großen Skalen betrachtet homogen sein. In der Summe ergäbe sich ein flacher Raum auf Universums-Größenordnung.

  210. #211 alex
    10. September 2018

    Volumen und zugehöriger Umfang einer Kugel, …

    In deiner Formel kommen Größen vor, die in zwei unterschiedlichen Räumen zu messen sind. Also kann es ja nicht eine Kugel sein, es müssen zwei sein. Eine in dem einen Raum, eine im anderen. Was ist der Zusammenhang zwischen diesen Kugeln?

    … mit z.B. genormtem Umfang.

    Wenn der Umfang genormt ist, warum steht dann in deiner Formel U0 / U1? Sollte das nicht immer gleich 1 sein?

    Das funktioniert für die homogene Massekugel, Scharzschild-Metrik und so, dort wäre dann die Raumdichte mit Masse größer als 1, größer als der flache Raum.

    Ich bin mir nicht sicher, ob es in diesen Fällen funktioniert (oder was “funktioniert” dann genau bedeutet). Die Fragen sind doch: Ist das eine wohldefinierte Größe? Und wenn ja, ist das eine sinnvolle Größe, also eine die nützlich für die Beschreibung der Natur ist?

    So wie du sie momentan definiert hast (mit mit Volumina und Umfängen), ist die Raumdichte mathematisch ein sehr kompliziertes Objekt. Sie ist ein nichtlineares und nichtlokales Funktional der Metrik, sie hat ein sehr kompliziertes Transformationsverhalten beim wechseln des Koordinatensystems, und es ist nicht klar, ob sie in Raumzeiten ohne ausgezeichnete Zeitrichtung wohldefiniert ist. Das spricht nicht unbedingt dafür, diese Größe zu verwenden.

    Leider funktioniert das nicht so ganz für meine Idee von der umgekehrten Krümmung, weil die “außerhalb” liegen soll und damit ein Stück Umfang auch immer außen liegt, die Kugel verzerrt wäre. Hm… Da es ja einen Maximalabstand geben soll lege ich die gleiche Kugel nun um den halben Maximalabstand herum (“Mittelpunkt des dünnen Raums”, brauche dafür diesbezüglich Kugelsymmetrie). Dann soll jetzt diese Kugel eine Raumdichte kleiner als 1 haben.

    Das verstehe ich wieder einmal nicht.

    Wenn Du dachtest, dass ich das ganze Universum erklären will dann hast Du mich sowieso falsch verstanden.

    Die Theorie die du ersetzen willst erklärt das ganze Universum (zu einem gewissen Grad). Wenn deine Theorie nicht mindestens genau so gut ist, weshalb sollte man sie dann verwenden? Welchen Vorteil hat sie gegenüber ART mit dunkler Materie?

    Dass es eben auch eine Metrik mit umgekehrter Krümmung geben könnte, die die gleichen Bewegungsgleichungen erzeugt.

    Dann wäre es schön, wenn zu zeigen könntest, dass das der Fall ist. Denn meiner Meinung nach ist es überhaupt nicht klar, dass das wahr ist.

    Na gut, dafür braucht man dann endliches Volumen und Maximalabstand, das hast Du mir aufgezeigt, aber auch das wäre ja vereinbar mit den Beobachtungen.

    Wenn ich mich richtig erinnere, habe ich lediglich gesagt, dass endliches Volumen (und sphärische Symmetrie) nur mit Maximalabstand machbar sind. Dass “umgekehrte Krümmung” zu endlichem Volumen führt, stammt nicht von mir; ich bin mir ziemlich sicher, dass das falsch ist.

    Die dunkle Materie kann die Gültigkeit von MOND nicht erklären.

    Wie ich bereits sagte: MOND wurde genau so konstruiert, dass sie die beobachtete Bewegung von Sternen in Galaxien beschreibt. Da gibt es nichts zu erklären.

    Und nicht den Zusammenhang zwischen Lambda und a0.

    Welchen Zusammenhang?

    Die umgekehrte Krümmung vielleicht schon.

    Bislang hast du noch nicht einmal demonstriert, dass “umgekehrte Krümmung” die Bewegung von Sternen in Galaxien korrekt beschreiben kann. Und die kosmologische Konstante/dunkle Energie hast du bislang in deiner Theorie nicht erwähnt. Warum sollte sie also in irgend einer Form hiermit zu tun haben?

    Aber warum macht das keiner?

    Die meiner Meinung nach einfachste Erklärung wäre, dass es keinen Grund gibt anzunehmen dass es funktioniert.

    Die Masseverteilung im Universum ist ja (auf großen Skalen) homogen, und die Bereiche mit “dünnem Raum” könnten auch auf großen Skalen betrachtet homogen sein. In der Summe ergäbe sich ein flacher Raum auf Universums-Größenordnung.

    Auch diese Schlussfolgerung verstehe ich nicht.

  211. #212 Randerscheinung
    10. September 2018

    Das Volumen einer homogenen Massenkugel mit bestimmtem Umfang ist aufgrund der Raumkrümmung größer als wenn die Massenkugel leer wäre. Ist eine Raumkrümmung möglich, die dazu führt, dass das Volumen der Kugel kleiner wäre als das des leeren Raums?

  212. #213 alex
    10. September 2018

    Das Volumen einer homogenen Massenkugel mit bestimmtem Umfang ist aufgrund der Raumkrümmung größer als wenn die Massenkugel leer wäre.

    Nur damit die Situation klar ist: Das Volumen einer homogenen Kugel positiver Masse mit bestimmtem Umfang ist größer als das Volumen einer Kugel selben Umfangs im Vakuum; also wenn V das Volumen und U der Umfang ist, gilt V > U³/(6 π²). Aber wenn ich mich nicht verrechnet habe, wäre das Volumen noch größer, wenn die Kugel die selbe Masse hätte (gemessen anhand der Metrik im Außenraum) und innen leer wäre, wenn also die gesamte Masse auf eine dünne Schale konzentriert wäre.

    Ist eine Raumkrümmung möglich, die dazu führt, dass das Volumen der Kugel kleiner wäre als das des leeren Raums?

    Ja.

  213. #214 Randerscheinung
    10. September 2018

    Ist eine Raumkrümmung möglich, die dazu führt, dass das Volumen der Kugel kleiner wäre als das des leeren Raums?

    Ja.

    aha, interessant, und wie? In unserem Universum? Ohne äußere Krümmung (d.h. ohne Krümmung in einer höheren Dimension?)

    Die dunkle Materie kann die Gültigkeit von MOND nicht erklären.

    Wie ich bereits sagte: MOND wurde genau so konstruiert, dass sie die beobachtete Bewegung von Sternen in Galaxien beschreibt. Da gibt es nichts zu erklären.

    Findest Du? Aber für mich sieht “für a>a0 gilt F~1/r statt F~1/r^2” eher nach einem Naturgesetz aus als danach, dass da etwas “genau so konstruiert wurde, dass es die Bewegung … beschreibt”! Im Gegensatz dazu sieht mir die dunkle Materie, die genau so angenommen wird, dass sie die Krümmung beschreibt eher konstruiert aus. Findest du nicht? Zum Beispiel ist es doch seltsamt, dass eben grade Halos von dunkler Materie um die Galaxien auftreten und die DM nicht homogener verteilt ist. Und müsste man dann nicht außer der Beschleunigung der äußeren Sterne noch mehr Effekte der DM sehen?

    Und nicht den Zusammenhang zwischen Lambda und a0.

    Welchen Zusammenhang?

    War da nicht irgendwas von Lambda ~ a0^2 oder so?

  214. #215 Alderamin
    10. September 2018

    @Randerscheinung

    Zum Beispiel ist es doch seltsamt, dass eben grade Halos von dunkler Materie um die Galaxien auftreten und die DM nicht homogener verteilt ist.

    Nö, das ist überhaupt nicht erstaunlich, sondern das, was auch in Simulationen von Teilchen herauskommt. Die angenommene DM war anfangs gleichverteilt bis auf geringe Dichteschwankungen und verdichtete sich dann dort, wo die anfängliche Dichte zufällig etwas höher war.

    Nichts anderes tat die sichtbare Materie. Die klumpte nur besser zusammen und bildete Sterne und Planeten. Nach Deiner Logik müsste die sichtbare Materie ebenfalls homogener verteilt sein – aber warum sollte sie, wo es doch Schwerkraft gibt?

    Und müsste man dann nicht außer der Beschleunigung der äußeren Sterne noch mehr Effekte der DM sehen?

    Sieht man doch. Lichtablenkung um dunkle Materie, auch wo wenig sichtbare ist (Bullet Cluster). Bewegung von Kugelsternhaufen um Galaxien. Zusammenhalt von Galaxienhaufen. Flachheit des Universums (zusammen mit der Dunklen Energie), messbar in der Hintergrundstrahlung. Relatives Verhältnis der primordialen Elemente (kommt nur mit DM so raus). Siehe auch #115.

    Mit der DM kann man alle Naturgesetze so lassen, wie man sie kennt. Man braucht nur eine neue Art von Teilchen anzunehmen, die sich nur über Gravitation und evtl. schwache Wechselwirkung nachweisen lässt, dann passt es. Diese Annahme ist nicht so exostisch, man hat im letzten Jahrhundert einen ganzen Teilchenzoo entdeckt, darunter mit den Neutrinos auch Teilchen, die extrem schwer nachweisbar sind.

    Es gibt keine Inkonsistenz bei der Dunklen Materie. Außer, dass man ihre Trägerteilchen noch nicht gefunden hat. Dass wir diese Teilchen noch nicht gefunden haben, heißt nicht, dass es sie nicht gibt. Wir haben auch noch kein Leben auf einem anderen Planeten gefunden. Heißt auch nix.

  215. #216 alex
    10. September 2018

    aha, interessant, und wie?

    Für einen kugelsymmetrischen Raum lässt sich ein Koordinatensystem finden, in dem das Linienelement die Form

    ds² = A( r ) dr² + r^2 dθ^2 + r^2 sin^2 θ dϕ^2

    hat, wobei A eine ausreichend glatte positive Funktion von r ist. Die Fläche mit r = R = const ist eine Kugel (in dem Sinn, dass ihre Punkte genau die sind, die vom “Zentrum” den selben Abstand haben; aber dieser ist im Allgemeinen nicht gleich R). Der Umfang dieser Kugel ist U = 2 π R, und das Volumen ist

    V = 4 \pi \int_0^R \sqrt{A} r^2 dr

    Jetzt kannst du A so wählen, dass V kleiner als U³/(6 π²) ist.

    Ob das als “Raumanteil” einer Raumzeit in der ART möglich ist, ist eine andere Frage. Ohne die Rechnung durchgeführt zu haben, würde ich vermuten, dass dafür “exotische Materie” notwendig ist.

    Aber für mich sieht “für a>a0 gilt F~1/r statt F~1/r^2” eher nach einem Naturgesetz aus als danach, dass da etwas “genau so konstruiert wurde, dass es die Bewegung … beschreibt”!

    Ich weiß nicht auf welche konkrete MOND-Implementierung du dich beziehst, aber das ist sicher nicht der ursprüngliche Ansatz von MOND. Der sah eher so aus: Statt F = m a gilt F = m a µ(a/a0), wobei für die Funktion µ nur die Asymptotik postuliert wird, und die Konstante a0 ein freier Parameter ist. Und beides wird genau so gewählt, dass es die Rotationskurven von Galaxien richtig beschreibt.

    Es gibt inzwischen mehrere weiter ausgearbeitete Versionen von MOND, aber in allen die mir bekannt sind, gibt es einen freien Parameter wie a0 und eine freie Funktion. Und wie in der ursprünglichen MOND werden beide genau so gewählt, dass die Rotationskurven von Galaxien passen.

    Im Gegensatz dazu sieht mir die dunkle Materie, die genau so angenommen wird, dass sie die Krümmung beschreibt eher konstruiert aus. Findest du nicht?

    Nein, finde ich nicht. Man muss im Prinzip nur postulieren, dass es eine Form der Materie gibt, die nicht elektromagnetisch wechselwirkt, dass es davon eine gewisse Menge gibt, und dass diese nicht zu “heiß” ist. Alles andere folgt ziemlich zwanglos aus den bekannten Naturgesetzen. Und wir kennen ja bereits eine Form der Materie, die nicht mit Licht wechselwirkt.

    Zum Beispiel ist es doch seltsamt, dass eben grade Halos von dunkler Materie um die Galaxien auftreten und die DM nicht homogener verteilt ist.

    Gravitation wirkt auch auf dunkle Materie, deshalb “klumpt” sie. Nur “klumpt” sie nicht so stark wie sichtbare Materie, weil dafür eine Art “Reibung” notwendig ist, und bei sichtbarer Materie ist das üblicherweise ein elektromagnetischer Effekt.

    Und müsste man dann nicht außer der Beschleunigung der äußeren Sterne noch mehr Effekte der DM sehen?

    Tut man ja auch. Es gibt eine ganze Reihe von unabhängigen Beobachtungen die mit dunkler Materie übereinstimmen.

    War da nicht irgendwas von Lambda ~ a0^2 oder so?

    Was bedeutet das “~”? Ja wohl nicht “proportional zu”, denn wir kennen ja nur einen Satz von Parametern Λ und a0. Und auch nicht “in etwa gleich groß”, denn Λ und a0² haben ja unterschiedliche Dimensionen (Λ ist eine inverse Fläche, a0² das Quadrat einer Beschleunigung).

  216. #217 Randerscheinung
    10. September 2018

    Irgendwo hab ich den netten Ausspruch gelesen, die dunkle Materie anzunehmen ist wie zu sagen, eigentlich gilt in unserem Sonnensystem für die Gravitation nicht F~1/r^2 – sondern es ist in Wirklichkeit F~^/r^3 – es gibt aber dunkle Materie, die genau so verteilt ist, dass es aussieht wie F~1/r^2.

    Und a0 ist nicht beliebig, sondern für alle Galaxien gleich. Das ist ja das komische, das es zumindest aussieht wie eine Naturkonstante.

  217. #218 alex
    10. September 2018

    Sind wir jetzt auf diesem Niveau angekommen?

    An einer allgemeinen Diskussion “dunkle Materie vs. MOND” habe ich kein Interesse. Dazu habe ich viel zu wenig Ahnung was Details sowohl der Theorien als auch der Beobachtungen angeht.

    Und es fördert mein Interesse nicht gerade, wenn das dann noch mit Aussagen verbunden wird, die man eigentlich nur als Beleidigungen eines großen Teils der an diesem Thema Forschenden interpretieren kann. Und wenn mir Aussagen unterstellt werden, die ich nicht getätigt habe.

    Daher tendiere ich stark dazu, unser Gespräch nun als beendet zu anzusehen.

  218. #219 Randerscheinung
    11. September 2018

    Ich wollte niemanden beleidigen. Wenn das so rüber kam tut mir das Leid.
    Es ist für mich auch in Ordnung, das Gespräch hier zu beenden. Ich habe viel gelernt und konnte meine Gedanken zu dem Thema mit jemandem besprechen, der Ahnung von der Materie hat. Ich habe mich sehr darüber gefreut. Ich danke Dir für Dein Engagement und die gut überlegten Antworten!

  219. #220 Randerscheinung
    2. Oktober 2018

    Hallo, ich hoffe, ihr seid einverstanden mit einer Wiederaufnahme der Diskussion. Ich habe nämlich einen Artikel gefunden, der, wenn ich es richtig verstanden habe, in diese Richtung geht: Von Ram Gopal Vishwakarma “Gravity of R^{ik}=0: A New Paradigm in GR”.
    Er sagt allerdings nicht Masse ist die Quelle der Spacetime sondern umgekehrt, Spacetime ist die Quelle.
    Aber, in seiner Theorie gibt es die Vakuumlösungen, den leeren Raum, nicht.

    Also, ich bin sehr gespannt, was ihr dazu sagt und davon haltet! 🙂

  220. #221 alex
    3. Oktober 2018

    @Randerscheinung:
    Ich bin nicht besonders daran interessiert, das hier weiterzuführen. Ich habe nicht den Eindruck dass das irgendetwas bringt.

    Aber da ich heute wegen des Feiertags ein bisschen Zeit hatte, habe ich das Paper dann noch kurz gelesen. Und ich war mir oft nicht sicher, ob der Autor das wirklich ernst meint.

    Es gibt viele Ungenauigkeiten in dem Paper (und auch eine ganze Reihe von Aussagen die wahr aber nicht neu sind). Ich möchte da nicht im einzelnen drauf eingehen; dafür ist mir meine Zeit zu schade. Stattdessen möchte ich nur ein ganz grundsätzliches Problem ansprechen, das er völlig ignoriert: Mit R_{\mu\nu} = 0 bekommt man im Sonnensystem nicht die Schwarzschild-Metrik (und auch nicht die Kerr-Metrik), sondern die Minkowski-Metrik. D.h. es gibt keine Kraft die die Planeten auf ihren Umlaufbahnen hält.

    Die außen passende Schwarzschild-Metrik bekommt man nur, wenn man einen Punkt im Inneren der Sonne ausspart und man irgendwie dafür sorgt, dass der Schwarzschild-Radius stimmt. Also gibt es entweder einen Punkt im Inneren der Sonne (und im Inneren der Erde, des Mondes, und jedes weiteren Objekts das eine Gravitationsanziehung ausübt), in dem die Feldgleichungen nicht gelten, oder die Raumzeit hat dort jeweils ein Loch. Und selbst wenn das so wäre (warum auch immer), dann gäbe es im Inneren dieser Objekte jeweils ein schwarzes Loch. Und das ist offenbar nicht der Fall, denn sonst wäre die Materie der Sonne ja schon längst dort “hineingefallen”.

    Und selbst wenn man all das ignoriert, hat man noch das Problem, dass es ohne den Energie-Impuls-Tensor auf der rechten Seite der Feldgleichungen nichts gibt, das den Schwarzschild-Radius festlegt. D.h. es gäbe keinen Grund, weshalb unterschiedliche Objekte unterschiedlich starke Gravitationsfelder erzeugen sollten. Wenn der Autor in Gleichung (2) ein G und ein m stehen haben will, muss sich das in irgend einer Form aus den Gleichungen ergeben. In Gleichung (1) steht aber kein G und auch nichts aus dem sich m ergeben könnte.

    Es mag sein dass er auf diese Punkte eine Antwort hat. Und vielleicht taugt ist diese Antwort sogar etwas (obwohl ich das doch sehr bezweifle). Aber dass er diese ganz offensichtlichen Probleme nicht einmal erwähnt, lässt mich doch sehr daran zweifeln. Stattdessen behauptet er einfach “The validity of the equations has already been well-established by the local observations through the classical tests of GR”. Und das stimmt einfach nicht.

    Man könnte auch noch etwas zur Zeitschrift sagen, in der der Artikel veröffentlicht wurde. Z.B. erschien in der selben Ausgabe auch ein Artikel mit dem schönen Titel “Pi is Fundamental Cosmic Frequency in Expanding Universe”. Aber auch das möchte ich mir sparen. Es sei auf den englischen Wikipedia-Artikel zum Verlag Bentham Open und die dortigen Quellen verwiesen.

    Er sagt allerdings nicht Masse ist die Quelle der Spacetime sondern umgekehrt, Spacetime ist die Quelle.

    Nein, das sagt er nicht.

    Aber, in seiner Theorie gibt es die Vakuumlösungen, den leeren Raum, nicht.

    Nein, es ist genau umgekehrt. Seiner Ansicht nach gibt es nur die Vakuumlösungen und sonst keine weiteren Lösungen.

  221. #222 Randerscheinung
    3. Oktober 2018

    Erstmal vielen Dank, dass Du Dir das Paper angeschaut hast! Deine Analyse bringt mich jedenfalls wieder ein Stück weiter.
    Ein Artikel des Autors über das gleiche Thema steht aber jedenfalls unter dem Titel “Mysteries of R ik = 0: A novel paradigm in Einstein’s theory of gravitation” in Frontiers of Physics, vom Springer Verlag, 2014.
    Ich habe mir aber die Arxiv- Version von 2013 geholt, weiß also gar nix vom Bentham open.

    Naja, aber jedenfalls ist die Resonanz auf dieses Paper auch nicht so riesig… und außerdem entspricht es auch nicht so richtig meinem Gedanken mit der umgekehrten Krümmung.

    Irgendwo hast Du geschrieben, dass a0 in MOND eine beliebige Größe ist… wieso? Wieso ist es Deiner Meinung nach keine Naturkonstante? Es ist doch für alle Galaxien das gleiche a0, dachte ich, oder stimmt das nicht?

    Ich finde #179, mein Kästchenanalogon und Deine Antwort 183 sind ein paar Highlights der Diskussion.

    Dann braucht man für V->0 für m->0 ein endliches Volumen pro Masse, das habe ich inzwischen verstanden. Aber das macht ja nix. Was an den Rändern passiert könnte genau das sein was wir als Krümmung durch die dunkle Materie ansehen.

  222. #223 alex
    4. Oktober 2018

    Wie gesagt bin ich nicht besonders daran interessiert, das hier weiterzuführen. Ich bezweifle, dass das hier dir wirklich etwas bringt. Mir bringt es jedenfalls nichts; es kostet mich nur vergleichsweise viel Zeit.

    Wenn du ernsthaft an diesen Themen interessiert bist, wäre es meiner Ansicht nach sinnvoller, wenn du dich zunächst mit den mathematischen und physikalischen Grundlagen beschäftigen würdest, anstatt irgendwelche hochspekulativen Paper zu lesen.

  223. #224 Randerscheinung
    4. Oktober 2018

    Hm, ja, das ist schon ein Punkt. Und tatsächlich, dass Du sagst, Du verstehst nicht was ich meine lässt mich schon sehr an meiner Idee zweifeln.
    Allerdings hab ich zum Beispiel mit dem endlichen Volumen ja jetzt verstanden, dass Du Recht hattest und dass ich Unsinn geschrieben habe.
    Ich würde gern bzgl. der umgekehrten Krümmung auch gern verstehen, warum das nicht gehen soll.

  224. #225 Randerscheinung
    4. Oktober 2018

    Hallo Alex,
    ich hätte einen konkreten Vorschlag einer Metrik, vielleicht kannst Du mir den ja noch widerlegen.
    Zur Schwarzschildmetrik (innere Lösung für rrg) füge ich eine weitere Lösung (nennen wir sie mal “weak field”-Lösung) hinzu, die für r>rw gelten soll. rw ist dabei der Radius, an dem in MOND a<a0 gelten würde.

    ds^2 = -(3/2 \sqrt{1+rs/rw} – 1/2 \sqrt{1 + (rs * r^2 /rw^3)})^2 c^2 dt^2 + (1/ (1 + (rs * r^2 /rw^3))) dr^2 + r^2 dθ^2 + r^2 sin^2 θ dϕ^2

    Das ist die innere Schwarzschild-Metrik mit jeweils einem "+" vor dem r-Term statt einem "-". Dies habe ich gemacht, weil in der inneren Schwarzschild-Metrik im r-Term die Masse/Massendichte steckt und der Krümmungseffekt meines Linienelementes genau umgekehrt sein soll wie der in der inneren Schwarzschild-Metrik (also tatsächlich so, als wäre die Masse "negativ" oder abstoßend statt anziehend).

    Man müsste schauen, ob die äußere Schwarzschild-Lösung am Radius rw gleich der "weak field" Lösung an rw wird bzw. werden kann mit ein paar Normierungen.

  225. #226 alex
    4. Oktober 2018

    1.) Wie ich bereits mehrmals sagte, habe ich nicht den Eindruck dass das hier zu irgendetwas sinnvollem führt. Daher habe ich auch wenig Interesse, Rechnungen mit irgendwelchen postulierten Metriken durchzuführen.

    2.) Im hier betrachteten Fall mit kleinen Geschwindigkeiten und kleinen Beschleunigungen (und damit schwachen Gravitationsfeldern) geht die ART in die Newtonsche Theorie über. Es ist also zumindest in einem ersten Schritt nicht notwendig, eine Metrik aufzustellen. Ein einfaches Gravitationspotential tut es auch. Und man braucht viel weniger Mathematik um das zu verstehen.

    3.) Wenn du die selbe Metrik verwendest die man für dunkle Materie erhält (und du wie in der ART die Geodätengleichung verwendest), dann bekommst du auch die selben Bewegungen von Probekörpern. Wenn du eine andere Metrik verwendest (auch modulo Diffeomorphismen), dann bekommst du im Allgemeinen andere Bewegungen von Probekörpern. Ob es für die in Galaxien auftretenden Bewegungen Unterschiede gibt, ist eine andere Frage.

    4.) Die Schwierigkeit ist nicht, ein Gravitationspotential oder eine Metrik zu finden, die zur beobachteten Bewegung passt. Die Schwierigkeit ist zu begründen, warum gerade diese Metrik vorliegt. Die Grundgleichung der ART ist nicht die Schwarzschild-Metrik, sondern die Einsteinschen Feldgleichungen. Wenn du willst, dass sich bei einer gegebenen Massenverteilung eine andere Metrik ergibt, als das die Einsteinschen Feldgleichungen sagen, dann musst du die Einsteinschen Feldgleichungen modifizieren. Du kannst nicht einfach postulieren, dass es eine andere Metrik gibt. Wenn man deine Metrik in die Einsteinschen Feldgleichungen einsetzt (was ich nicht getan habe, siehe Punkt 1), wird herauskommen, dass es eine andere Massenverteilung gibt als sichtbar ist, also dunkle Materie. Wenn du das nicht willst, musst du die Einsteinschen Feldgleichungen modifizieren. Und das ist nicht einfach.

    Wenn ich mich richtig erinnere, habe ich all das bereits mehrmals gesagt. Und das bestärkt nur meinen Eindruck aus Punkt 1.

  226. #227 Randerscheinung
    4. Oktober 2018

    Zu 3. Und welche Metrik erhält man für die dunkle Materie?

    Zu 4. Beim Einsetzen der Schwarzschild-Metrik in die Feldgleichungen benutzt man aber die Vakuumlösung, dass der leere Raum flach und ungekrümmt wäre. Wenn nun aber das nicht stimmt, sondern das Volumen des Raumes für m->0 gegen 0 geht, dann braucht man die Feldgleichungen nicht zu modifizieren.

  227. #228 Randerscheinung
    4. Oktober 2018

    Zu 2. Und wenn diese Annahme falsch wäre? Wenn also zwar wie du sagst für kleine Beschleunigungen die ART in die Newtonsche Theorie übergeht – aber eben für sehr kleine Beschleunigungen wieder in etwas anderes?
    Weil zB. die Raumzeit begrenzt ist und sich an den Rändern so verhält als würde sich “negative Masse” (Bondi 1957) kugelsymmettisch aufbauen?

  228. #229 alex
    4. Oktober 2018

    Und welche Metrik erhält man für die dunkle Materie?

    Das hängt davon ab, wie die dunkle Materie verteilt ist. Eine Lösung die so einfach ist, dass man sie als kurze Formel hinschreiben kann, wird es (wie in den meisten Fällen in der ART) nicht geben.

    Beim Einsetzen der Schwarzschild-Metrik in die Feldgleichungen benutzt man aber die Vakuumlösung, dass der leere Raum flach und ungekrümmt wäre.

    Nein, das tut man nicht.

    Zu 2. Und wenn diese Annahme falsch wäre?

    Das ist keine Annahme. Das ist ein mathematischer Satz. Das kann man streng mathematisch beweisen.

  229. #230 Randerscheinung
    4. Oktober 2018

    Doch. Die Schwarzschild-Metrik ist die Vakuumlösung. Und die Schwarzschild-Metrik könnte auch nicht stimmen, könnte ergänzt werden müssen mit einem Term der umgekehrten Krümmung für a <a0.
    Das sähe dann vielleicht wirklich so aus wie negative Masse, die sich "außerhalb" befindet.
    Überall, in allen Metriken die wir bis jetzt haben steckt die Annahme der Vakuumlösungen (flacher Raum ohne Masse).

  230. #231 alex
    4. Oktober 2018

    Doch.

    Nein.

    Überall, in allen Metriken die wir bis jetzt haben steckt die Annahme der Vakuumlösungen (flacher Raum ohne Masse).

    Nein.
    Wenn du der Meinung bist, mehr über ART zu wissen als ich, warum fragst du mich dann überhaupt?

  231. #232 Randerscheinung
    4. Oktober 2018

    Zum Beispiel steht in Wikipedia zur äußeren Schwarzschild-Metrik:
    “Die äußere Schwarzschild-Lösung ist die statische Vakuumlösung der Feldgleichungen für den Außenraum einer kugelsymmetrischen Materieverteilung.[1] ”

    Die Frage ist, was passiert, wenn man eine kugelsymmetrische “negative Masse” am Rand hinzufügt? Wäre das dann noch eine Lösung der Feldgleichungen oder definitiv nicht?

  232. #233 alex
    4. Oktober 2018

    Wäre das dann noch eine Lösung der Feldgleichungen oder definitiv nicht?

    Das hängt davon ab was genau du mit “das” meinst.

  233. #234 Randerscheinung
    5. Oktober 2018

    na die Formel aus #225.
    Wie macht man die Formeln überhaupt so schön wie Deine in #216?

  234. #235 alex
    5. Oktober 2018

    Man kann jede beliebige Metrik hinschreiben und in die Feldgleichungen einsetzen. Dann bekommt man quasi als Ergebnis den Energie-Impuls-Tensor, der zu dieser Metrik passt. Ob der dann physikalisch sinnvoll ist, ist eine andere Frage. Aber ich habe wenig Lust, diese Rechnung mit deiner Metrik durchzuführen; ich sehe nicht, welchen Sinn das hätte.

    Im Prinzip ist das aber nicht schwierig, sondern nur etwas länglich. Und die Formeln stehen in jedem Buch zur ART (und auch in der Wikipedia). Und zum reinen Durchführen der Rechnung braucht man auch nicht besonders viel Mathematik; man muss eine 4×4-Matrix invertieren (die ist bei deiner Metrik diagonal, also ist das ziemlich einfach) und jede Menge Ableitungen berechnen. Mehr nicht. Und nichtrelativistisch (was hier ja wie gesagt ausreichen sollte) ist das ganze noch deutlich einfacher.

    Materie die nicht mit Licht wechselwirkt und eine negative Masse hat, ist jedenfalls deutlich spekulativer als Materie die nicht mit Licht wechselwirkt und eine positive Masse hat (also gewöhnliche dunkle Materie). Und dass man “positive” dunkle Materie innen nicht notwendigerweise durch “negative” dunkle Materie außen ersetzen kann, habe ich ja bereits in Kommentar #124 geschrieben.

    Zu Formeln in Kommentaren hat Alderamin mal was geschrieben: https://scienceblogs.de/alpha-cephei/scienceblogs-kommentierungs-faq/#latex

  235. #236 Randerscheinung
    5. Oktober 2018

    Super, vielen Dank für die vielen Hinweise! Auch 124 ist super. Ich muss mir das mit der Kugelsymmetrie (letzter Abschnitt in #124) noch mal durch den Kopf gehen lassen… Kann sein, dass ds der entscheidende Punkt ist, den ich nicht kapiert habe…

  236. #237 Randerscheinung
    5. Oktober 2018

    Zur Kugelsymmetrie, der negativen Masse und meiner umgekehrten Krümmung:
    1. Ich verstehe, dass in der inneren Schwarzschild-Metrik nur die Masse innerhalb r entscheidend ist, weil sich die äußeren Anteile in der Integration zu Null aufsummieren.
    2. Mit einer richtigen negativen Masse wäre das wohl tatsächlich im kugelsymmetrischen Fall genauso.
    3. Masse krümmt den Raum: Es gibt die innere Schwarzschild-Metrik für die homogene Massenverteilung (in der sich Frage an Dich: was eigentlich, die Raumkrümmung? das Gravitationspotential? Die Raumdichte? Gibt es überhaupt irgendeinen slalaren Wert, der mit r wächst? … aufbaut) und die äußere Schwarzschild-Metrik für den Übergang von diesem “aufgebauten irgendwas” zum richtig flachen Raum.
    4. Jetzt kommt meine Idee von der umgekehrten Krümmung: die geht genau dann los, wenn der Raum im Außenbereich der äußeren Schwarzschild-Metrik genügend flach geworden ist (a kleiner a0).
    5. Damit ist diese umgekehrte Krümmung keine negative Masse im eigentlichen Sinn. Diese (die negative Masse) würde wohl auch eine Art “negative Schwarzschild-Metrik” innen und außen erzeugen.
    6. Da der Raum aber für a größer a0 so gut wie flach ist summiert sich in meiner Vorstellung die Gravitationswirkung der umgekehrten Krümmung nicht zu Null auf wie in der inneren Schwarzschild-Metrik. Das ist der entscheidende Punkt.
    Was hältst du davon?

  237. #238 alex
    5. Oktober 2018

    1. Ich verstehe, dass in der inneren Schwarzschild-Metrik nur die Masse innerhalb r entscheidend ist

    Nicht nur bei der inneren Schwarzschild-Metrik. Immer wenn die Massenverteilung kugelsymmetrisch ist. Egal wie das radiale Dichteprofil aussieht. Das ist das entscheidende. Wenn das nur für eine bestimmte Massenverteilung gelten würde, wäre die Aussage ziemlich sinnlos.

    Alles ab Punkt 3 verstehe ich nicht. Und ich bin mir nicht sicher, ob ich es verstehen will.

    Ich weiß nicht wie oft ich das noch sagen soll: Die Metrik ist nicht das entscheidende. Es ist bekannt welche Metrik die Beobachtungen erklärt. Nämlich die, die man aus den Feldgleichungen erhält, wenn man zusätzlich zur sichbaren noch dunkle Materie hinzunimmt. Wenn du diese Metrik ohne dunkle Materie haben willst (warum auch immer), musst du die Feldgleichungen ändern. Alles andere führt zu nichts.

    Einfach zu postulieren, dass die Metrik nicht die ist, die man aus den Feldgleichungen erhält, erklärt gar nichts. Da kann man genauso gut postulieren, dass sich die Sterne in Galaxien einfach so schneller bewegen als sie es sollten.

    Und wenn du der Meinung bist, dass man die Beobachtungen auch mit einer anderen Metrik erklären kann (also mit einer die nicht diejenige ist, die man aus Feldgleichungen + sichtbare Materie + dunkle Materie erhält), dann musst du das auch beweisen. Und du musst erklären, wie man auf diese Alternativmetrik kommt. D.h du musst dunkle Materie einführen und/oder die Feldgleichungen ändern.

    Wenn du das nicht kannst (also wenn du nicht sagen kannst, wie du die Feldgleichungen abändern willst, und du keinen Beweis führen kannst, dass eine alternative Metrik die Beobachtungen erklären kann (inklusive Erklärung, wie man auf diese Metrik kommt)), dann sehe ich keinen Sinn das hier weiterzuführen. (Für beides wäre ich mit nichtrelativistischen Rechnungen vollkommen zufrieden.)

    Es gibt genau zwei Möglichkeiten: Entweder die Gleichungen der ART stimmen. Dann gibt es dunkle Materie. Oder die Gleichungen der ART stimmen nicht. Und wenn man dieser zweiten Meinung ist, dann muss man angeben können, wie die korrekten Gleichungen konkret aussehen sollen.

  238. #239 Randerscheinung
    5. Oktober 2018

    Ok. Noch mal neu. Kugelsymmetrie. Masse im Zentrum.
    1. Es gibt einen Radius”Rw”, ab dem a kleiner a0 gilt, für den sich die Bewegung von Probekörpern nicht mehr allein durch die Schwarzschild-Metrik und die Masse im Zentrum erklären lässt. (Unsere Beobachtung)
    2. Man könnte dem Modell kugelsymmetrisch dunkle Materie für r größer Rw hinzufügen und würde eine bestimmte Metrik erhalten. Für die Probekörper zählt nur die innere dunkle Materie.
    3. Oder man könnte, statt Materie hinzuzufügen, dort Raum wegnehmen. Siehe mein Analogen in 2D in #154. Das ist von 2D auf 3D übertragbar. Was die Zeit macht weiß ich leider nicht – es müsste aber einfach das Gegenteil dessen sein, was in der inneren Schwarzschild-Metrik mit der Zeit passiert. Das ist eine Raumzeitkrümmung, die ohne dunkle Materie auskommt.
    4. Ich möchte also keine Alternativmetrik aufstellen, sondern genau die gleiche wie mit dunkler Materie.
    5. Die Feldgleichungen muss ich dabei auch nicht abändern.
    6. Das einzige, was ich “neu” postuliere ist, dass für m gegen 0 auch V gegen 0 geht. (“Masse = Quelle des Raums”)
    7. Für die Bewegung von Probekörpern zählt auch hier nur die Krümmung, die vom Probekörper aus gesehen innen liegt. Also gibt es keinen Widerspruch zum letzten Abschnitt von #124. (Und zumindest Punkt 6 in #237 war wirklich Quatsch)

  239. #240 alex
    6. Oktober 2018

    1. Es gibt einen Radius”Rw”, ab dem a kleiner a0 gilt, für den sich die Bewegung von Probekörpern nicht mehr allein durch die Schwarzschild-Metrik und die Masse im Zentrum erklären lässt. (Unsere Beobachtung)

    Die Schwarzschild-Metrik ist die Metrik einer Punktmasse. Eine Galaxie ist keine Punktmasse. Auch dann nicht wenn man nur die sichtbare Materie betrachtet.

    3. Oder man könnte, statt Materie hinzuzufügen, dort Raum wegnehmen. Siehe mein Analogen in 2D in #154. Das ist von 2D auf 3D übertragbar.

    Das bezweifle ich doch sehr. Solange ich dafür keinen Beweis gesehen habe (inklusive einer Definition davon was genau “Raum wegnehmen” heißen soll), gehe ich davon aus, dass das nicht stimmt.

    4. Ich möchte also keine Alternativmetrik aufstellen, sondern genau die gleiche wie mit dunkler Materie.
    5. Die Feldgleichungen muss ich dabei auch nicht abändern.

    Das ist schlicht falsch. Wenn du bei einer anderen Massenverteilung die selbe Metrik haben willst, kannst du nicht die selben Feldgleichungen verwenden. Die Feldgleichungen lauten (ohne kosmologische Konstante) R_{\mu\nu} - \frac{1}{2} R g_{\mu\nu} = \frac{8\pi G}{c^4} T_{\mu\nu}. Die linke Seite dieser Gleichung ist durch die Metrik vollständig bestimmt. Wenn diese Gleichung gilt, ist also auch die rechte Seite durch die Metrik vollständig bestimmt. Und auf der rechten Seite steht bis auf ein paar Konstanten nur der Energie-Impuls-Tensor. D.h. für eine gegebene Metrik gibt es genau eine Massenverteilung, für die die Feldgleichungen erfüllt sind.

    Wenn du die selbe Metrik verwenden willst, die man mit dunkler Materie bekommt, und du auch die selben Feldgleichungen verwenden willst, dann sagen dir diese Felgleichungen, dass es dunkle Materie gibt.

    Punkte 6 und 7 sind mir wieder unverständlich.

  240. #241 Randerscheinung
    6. Oktober 2018

    Ok. Das muss ich aber mathematisch Schritt für Schritt nachvollziehen… Hast du da einen Link?

    Was ist eigentlich die Vakuumenergie und braucht man für deren Berechnung die 00- Komponente?

  241. #242 Randerscheinung
    6. Oktober 2018

    Ich hab mir mal von zarm.uni-bremen.de “ART_Einstein.pdf” heruntergeladen.
    Kapitel 11.2, Seite 121 oben wird gefordert , dass im Unendlichen Minkowski gelten soll. Dass muss nicht stimmen.
    Ich weiß zwar noch nicht, was ich mit meiner ungekehrten Krümmung an dieser Stelle fordern würde -aber jedenfalls nicht, dass im Unendlichen Minkowski gelten soll.

  242. #243 alex
    6. Oktober 2018

    Was ist eigentlich die Vakuumenergie und braucht man für deren Berechnung die 00- Komponente?

    “Vakuumenergie” ist in diesem Kontext im wesentlichen die kosmologische Konstante.

    Kapitel 11.2, Seite 121 oben wird gefordert , dass im Unendlichen Minkowski gelten soll. Dass muss nicht stimmen.

    Zunächst einmal ist das lediglich eine Normierung. Wenn man das weglässt, bekommt man eine Metrik die sich von Schwarzschild nur durch eine Konstante unterscheidet: ds^2 = C \left(1 + \frac{\kappa}{r}\right) dt^2 - \left(1 + \frac{\kappa}{r}\right)^{-1} dr^2 - \dots Und diese Konstante kann man durch eine simple Koordinatentransformation loswerden. Physikalisch ist das exakt die selbe Metrik. (Hast du die Rechnung zwischen 11-2.52 und 11-2.56 ohne die Bedingung \lambda + \nu = 0 nicht selbst durchgeführt?)

    Viel wichtiger ist jedoch folgendes: Das ist eine andere Frage.

    In #239 hast du behauptet, dass du die selbe Metrik mit einem anderen Energie-Impuls-Tensor ohne Änderung der Feldgleichungen bekommen kannst. Damit das möglich wäre, müssten die Feldgleichungen zu einer festen Metrik mehrere unterschiedliche Energie-Impuls-Tensoren zulassen. Das ist nicht der Fall.

    Nun stellst du die Frage, ob die Feldgleichungen zu einem festen Energie-Impuls-Tensor unterschiedliche Metriken zulassen. Die Antwort darauf ist “ja” (auch wenn das hier wie oben erläutert kein Beispiel dafür ist), und zwar nicht nur für Diffeomorphismus-äquivalente Metriken. Z.B. gibt es für das Vakuum neben der flachen Minkowski-Raumzeit auch Lösungen mit Gravitationswellen.

  243. #244 Randerscheinung
    6. Oktober 2018

    Ist dir irgendeine Lösung bekannt, die nach außen hin (r gegen unendlich) trotz leeren Raums nicht in den flachen Raum übergeht?

    Tatsächlich brauche ich für meine Idee doch eine andere Metrik als die mit der dunklen Materie. Stimmt nicht, dass es die gleiche wäre (was ich in 4. in #239 geschrieben habe, sorry…). An manchen Punkten wäre die Wirkung auf die Probekörper die gleiche, das hat mich irritiert. Aber die Metrik ist schon verschieden, insgesamt, als ganze Metrik, über den ganzen Raum.
    Es wäre eine Metrik, die den Raum nach außen hin begrenzt.
    Die Zeit müsste wohl am Rand immer schneller werden, das Gegenteil vom Langsamerwerden nahe einem Massezentrum.

  244. #245 alex
    6. Oktober 2018

    Wie ich schon in Kommentar #238 geschrieben habe:

    Die Metrik ist nicht das entscheidende.

    Und:

    Wenn du der Meinung bist, dass man die Beobachtungen auch mit einer anderen Metrik erklären kann (also mit einer die nicht diejenige ist, die man aus Feldgleichungen + sichtbare Materie + dunkle Materie erhält), dann musst du das auch beweisen. Und du musst erklären, wie man auf diese Alternativmetrik kommt. D.h du musst dunkle Materie einführen und/oder die Feldgleichungen ändern.

    Und:

    Es gibt genau zwei Möglichkeiten: Entweder die Gleichungen der ART stimmen. Dann gibt es dunkle Materie. Oder die Gleichungen der ART stimmen nicht. Und wenn man dieser zweiten Meinung ist, dann muss man angeben können, wie die korrekten Gleichungen konkret aussehen sollen.

  245. #246 Randerscheinung
    6. Oktober 2018

    Ok. Habe verstanden, im Vakuum ist (laut Feldgleichungen) der Ricci-tensor null, weil der Energie-Impuls-Tensor null ist. Hast Du mir wahrscheinlich die ganze Zeit versucht zu sagen. —
    Also gibt es keine Krümmung im Vakuum. Ok soweit. Ich bin wieder einen Schritt weiter. Vielen Dank! 🙂

    Nun hätte ich aber noch eine Idee, meinen Gedanken mit der umgekehrten Krümmung in die ART zu integrieren:
    1. Es gibt zu einem Energie-Impuls-Tensor mehrere mögliche Metriken, sagst Du. Nun suche ich für das einfache Modell der Punktmasse (in einem Universum, in dem es nur diese Punktmasse gibt) eine Metrik, die insgesamt zu meinem Modell passt. Es ist ja kein Vakuum, weil Punktmasse im Zentrum.
    2. Ich definiere mir jetzt eine “Normierungsbedingung”, die bewirkt, dass es für r gegen unendlich die entgegengesetzte Krümmung gibt wie im Massezentrum. Also zum Beispiel
    “Das Integral des Ricci-tensors über das gesamte Universum sei Null.”
    Damit gäbe es am Rand des Universums eine der Krümmung am Massezentrum entgegengesetzte Krümmung.
    Wäre das dann eine Lösung der Feldgleichungen?
    Problem vielleicht:
    Ich weiß nicht, was das Integral des Ricci-tensors sein soll. Kann man den überhaupt integrieren?

  246. #247 Randerscheinung
    6. Oktober 2018

    Es gibt Gravitationswellen im Vakuum. Das bedeutet doch, dass es so etwas wie eine negative Krümmung geben muss. Wellen und Täler der Krümmung. Das Vakuum wird komprimiert und auseinandergezogen…
    Und meine Vorstellung (mein letzter Kommentar wird grade moderiert, ich hiffe, das Gerät jetzt nicht zu sehr durcheinander. ..) im letzten Kommentar mit der sich zu Null ergänzenden Krümmung wäre das dann so was wie eine Periode einer Gravitationswelle. Naja.. In der Mitte ist lange alles rel. gerade… Geht damit was?

  247. #248 Randerscheinung
    7. Oktober 2018

    An meiner Vorstellung des ganzen stimmt einiges noch nicht… es wird jetzt vielleicht ein bisschen durcheinander, insbesondere mit dem “wahren” Kommentar 246, der sich gerade in der Moderation befindet und wahrscheinlich irgendwann nachgeliefert wird.

    1. Habe verstanden, dass nach den Feldgleichungen mit T=0 auch R=0 gilt.
    2. Die äußere Schwarzschildmetrik ist eine Lösung der Feldgleichungen im Vakuum.
    3. Dort gibt es eine Krümmung, die äußere Lösung schließt an die innere an. Diese Krümmung wird für r gegen unendlich immer kleiner und geht in den Minkowski-Raum über.
    4. In der äußeren Schwarzschildmetrik gibt es aber keine “Quelle” der Raumzeitkrümmung. Die ist nur in der inneren Lösung zu finden.
    5. Meine Vorstellung (#154) hat etwas mit der Randbedingung für r gegen unendlich (oder bei Begrenztheit gegen sein mögliches Maximum) zu tun.
    6. Da meine Vorstellung praktisch eine Quelle der Raumkrümmung (eigentlich Senke, aber das ist ja das gleiche mit anderem Vorzeichen) beinhaltet muss nach den Feldgleichungen dort ein T sein.
    7. Was ich offensichtlich noch gar nicht verstehe ist der Unterschied zwischen einerseits einer “Quelle” der Raumzeitkrümmung(T) in der inneren Lösung und andererseits dieser Randbedingung, dass innere und äußere Lösung aneinander anschließen. Letzteres ist irgendwie keine Quelle, sondern eben eine Randbedingung.

  248. #249 Randerscheinung
    7. Oktober 2018

    Die Frage ist, wie kann iCh meine Idee des “Raum wegnehmens” (#154) in eine Randbedingung übersetzen???

    Wäre vielleicht so was wie eine Quantelung der Raumzeit in Verbindung mit der Beschleunigung a möglich? Also dass es als Schlussfolgerung dieser Quantelung eine minimal mögliche Beschleunigung pro Raumeinheit gibt – womit dann für a kleiner a0 notgedrungen der Raum “dünner” werden muss?

  249. #250 alex
    7. Oktober 2018

    Ich kann mich nur wiederholen.

    Wenn du der Meinung bist, dass “Raum wegnehmen” die Beobachtungen erklären kann, dann musst du zunächst einmal mathematisch präzise sagen, was genau das eigentlich bedeuten soll. Und dann musst du beweisen, dass sich die Beobachtungen damit tatsächlich erklären lassen. Beides hast du bisher nicht getan.

    Und unabhängig davon: Wenn es keine dunkle Materie gibt, dann müssen die Gleichungen der ART falsch sein. Wenn du also die Beobachtungen ohne dunkle Materie erklären willst, dann musst du angeben, wie die korrekten Gleichungen lauten sollen (und du musst beweisen, dass diese Gleichungen tatsächlich das leisten was du behauptest).

  250. #251 Randerscheinung
    7. Oktober 2018

    … okok, viel zu tun, würde ich sagen 😉
    Setze mich grade mit der neuen Theorie von Erik Verlinde auseinander, das Paper und eine Vorlesunglaubliche auf Soundcloud… Vielleicht geht das ja in die Richtung (and of course: far beyond)

  251. #252 Randerscheinung
    8. Oktober 2018

    Es sollte wirklich Vorlesung heißen, unglaublich wurde sie wohl irgendwie durch die Autokorrekturfunktion meines Handys.
    Schön, dass #246 nach Moderation an die richtige Stelle gerückt ist.
    Ich kann leider immer noch keine Formeln, das latexen funktioniert, zumindest in der Vorschau, nicht. R_\mu\nu

    Ich habe nun endlich kapiert, dass meine Idee von der umgekehrten Krümmung einfach nicht mit der ART vereinbar ist. Vielen Dank für deine Geduld! Für eine Änderung der Krümmung brauche ich den Energie-Impuls-Tensor in der ART. Füge ich andere Randbedingungen ein, verändert sich die Lösung der Feldgleichungen nur um eine Konstante. Aber sie ändert nicht ihre Qualität . An der Stelle, an der meine umgekehrte Krümmung losginge wäre die Metrik nicht mehr differenzierbar. Für solche nichtdifferenzierbaren Stellen braucht es laut ART den Energie- Impuls-Tensor. Stimmt das so?
    Jedenfalls finde ich aber immer noch, dass eine umgekehrte Krümmung äquivalent wäre der dunklen Materie. Habe aber verstanden, dass sie nicht mit der ART vereinbar wäre. Deshalb wird es wohl nicht diskutiert/groß drüber nachgedacht: weil sie scheinbar der ART widersprechen würde und damit scheinbar schon widerlegt wäre.
    Nun zur Theorie von Verlinde, die wirklich sehr spannend ist (und wovon ich vieles noch nicht verstanden habe) Er zeigt einen Weg, wie man die Grenzen der ART erkennen kann. Und dass es andere Theorien gibt, die dann gelten. Wie Phasenübergänge. Wie Welle-Teilchen-Dualismus. Dass das Teilchenmodell den Beugungsversuch nicht erklären kann heißt ja nicht, dass man bewiesen hat, dass Licht kein Teilchen ist. Man muss nicht alle Phänomene mit nur einer Gleichung erklären.
    Und wenn ich sage, dass eine umgekehrte Krümmung eine zur dark matter äquivalente Erklärung wäre möchte ich damit auf keinen Fall sagen, dass ich die Feldgleichungen für falsch halte. Die sind schön und richtig und wunderbar. Nur eben nicht vollständig, möglicherweise.
    Vielleicht wolltest du mir das die ganze Zeit sagen: entweder gilt die ART immer, dann gibt es dunkle Materie – oder sie gilt nicht immer, dann wäre meine umgekehrte Krümmung möglich (woher auch immer).
    Es ist so seltsam, ich hatte ja nur diesen einen kleinen Gedanken (#154), und wollte auf keinen Fall (und schon gar nicht damit) die ART in Frage stellen. Wenn wir ohne die Feldgleichungen zu kennen nur herausgefunden hätten, dass Raumkrümmung = Gravitation ist wäre es ein recht trivialer Gedanke. Da wir aber nun mit dem Energie -Impuls-Tensor gleich die Quelle der Krümmung mitgeliefert bekommen haben scheint der Gedanke falsch zu sein.
    Das ist vielleicht ein weiterer Grund, warum nicht viel (über meinen Gedanken mit der umgekehrten Krümmung) drüber nachgedacht/ diskutiert wird: Weil es wirklich trivial ist wenn man die ART verlässt. Und man dann in den Bereich kommt wo man fragt, wo denn diese Krümmung herkommen soll (wenn denn nicht von dunkler Materie). Und da gehen dann die Theorien erst los.
    Ich danke dir vielmals, Alex, für deine Unterstützung und Geduld. Ich hab jedenfalls viel gelernt. Und Spaß gehabt an der Diskussion.

  252. #253 alex
    8. Oktober 2018

    Jedenfalls finde ich aber immer noch, dass eine umgekehrte Krümmung äquivalent wäre der dunklen Materie.

    Und ich sehe immer noch nicht, warum das der Fall sein sollte. Und wenn ich mich richtig erinnere, sage ich das seit einem Monat.

    Von der Krümmung zur Bewegung von Probekörpern ist es ein langer rechnerischer Weg, auf dem man Systeme partieller und gewöhnlicher Differentialgleichungen lösen muss. Die von dir behauptete Äquivalenz ist meiner Ansicht nach ganz und gar nicht offensichtlich. Von meiner Intuition her würde ich sagen, dass sie falsch ist.

    Wenn du irgend jemanden davon überzeugen willst, dann musst du beweisen, dass es diese Äquivalenz gibt. Oder sie wenigstens plausibel machen. Am besten wäre es, wenn du sie präzise als eine mathematische Aussage formulieren und diese dann beweisen würdest. Oder als ersten Schritt könntest du auch ein Modellsystem konstruieren, in dem sich diese Äquivalenz zeigt (wobei du natürlich beweisen müsstest, dass das tatsächlich der Fall ist). Wie ich oft gesagt habe, wäre es am einfachsten das zuerst nichtrelativistisch zu betrachten.

    Das ist meiner Ansicht nach der wesentliche Grund, weshalb nicht an deiner Idee gearbeitet wird. Weil es einfach keinen Grund gibt anzunehmen, dass diese Äquivalenz tatsächlich existiert. Dass es dann auch noch keine Erklärung gibt, woher diese “umgekehrte Krümmung” kommen soll (jedenfalls keine die plausibler oder auch nur annähernd so plausibel wäre wie dunkle Materie), ist dann das zweite wesentliche Problem mit deiner Idee.

    Meiner Ansicht nach ist dunkle Materie eine äußerst plausible Lösung für das Problem. Wir wissen, dass Materie gravitativ wirkt, und wir haben quantitative Beschreibungen dafür, die sehr gut zu allen sonstigen Experimenten und Beobachtungen passen. Und wir haben bereits Formen der Materie gefunden die nicht mit Licht wechselwirken.

    Und auch wenn ich persönlich MOND für die weniger wahrscheinliche Lösung halte, sind das ebenfalls plausible Lösungen. Es sind konsistente Erweiterungen oder Modifikationen der bekannten Theorien, die die beobachteten Phänomene (oder jedenfalls einen Teil davon; so genau kenne ich mich da nicht aus) beschreiben können. Und es wäre ja nicht das erste Mal, dass eine astronomisch beobachtete Abweichung von der vorhergesagten Bewegung durch eine Änderung des Gravitationsgesetzes erklärt würde.

    Ist es da verwunderlich, dass Forscher eher an diesen Ideen arbeiten, als an einer Idee die schon im Grundsatz zwei ganz wesentliche Probleme hat? Von der also nicht klar ist, ob sie die Beobachtungen überhaupt korrekt beschreibt, und die sie zudem auch nicht erklärt (weil nicht klar ist, woher die “umgekehrte Krümmung” kommen soll).

  253. #254 Randerscheinung
    8. Oktober 2018

    Vielleicht ist die Formulierung “die umgekehrte Krümmung ist äquivalent zur dunklen Materie” nicht so glücklich gewählt.
    Was hältst du davon: Dem in MOND gefundenen Zusammenhang könnte eine Raumzeitkrümmung zugrunde liegen.

    Ich nehme MOND ernst. Man kann so ein Gesetz nicht einfach als geschicktes curvefitting abtun. Ich denke wirklich, dass damit ein Gesetz gefunden wurde.

    Diese Raumkrümmung läge außerhalb der ART. Das hab ich in unserer langen Diskussion nun endlich verstanden. Aber sie wäre eine der dunklen Materie äquivalente Erklärung der Rotationskurven der Galaxien (und des Gravitationslinseneffekts).

    Das ist die Kernaussage, die ich die ganze Zeit versuche zu verteidigen: Dem in MOND gefundenen Zusammenhang könnte eine Raumzeitkrümmung zugrunde liegen.

    Aber ist das wirklich neu? Kann ich mir eigentlich nicht vorstellen, dafür ist es doch zu trivial… oder?

  254. #255 Randerscheinung
    8. Oktober 2018

    Dem in MOND gefundenen Zusammenhang könnte eine Raumzeitkrümmung ohne dunkle Materie zugrunde liegen.

    Dort (a kleiner a0) könnten die Feldgleichungen nicht gelten. Trotzdem könnte gelten, dass Raumkrümmung die Bewegung der Probekörper beeinflusst. Es gäbe dort eine andere Quelle der Raumkrümmung als den Energie-Impuls-Tensor. Zum Beispiel eine Art Quantisierung des Raumes. Aber das würde zu weit führen.

    Meine Aussage wäre einfach nur: Dem in MOND gefundenen Zusammenhang könnte eine Raumzeitkrümmung ohne dunkle Materie zugrunde liegen.

  255. #256 Randerscheinung
    8. Oktober 2018

    habe übrigens grade noch einen Fehler festgestellt: Ich beziehe mich oft auf “meinen einen kleinen Gedanken” und schreibe dann #154 in Klammern. In Wirklichkeit muss es aber #179 heißen. Das Bild, was ich im Kopf habe ist einfach nur, dass diese Raumkrümmung, die wir sehen, von zwei verschiedenen Ursachen kommen könnte: dunkle Materie oder “dünner werdender Raum” für a kleiner a0. Das Bild wird in #179 plausibel gemacht.
    Klar müsste ich dieses Bild auf 4D ausweiten.

    Aber ist diese (klitzekleine) Idee tatsächlich von mir?
    Steht sie nicht schon in hundert Papers? Ist sie nicht völlig trivial?
    Ich weiß auch nicht, ob sie nicht vielleicht Teil von Verlindes Theorie ist, weil ich die noch nicht richtig verstanden habe.

  256. #257 alex
    8. Oktober 2018

    Das

    Jedenfalls finde ich aber immer noch, dass eine umgekehrte Krümmung äquivalent wäre der dunklen Materie.

    und das

    Dem in MOND gefundenen Zusammenhang könnte eine Raumzeitkrümmung zugrunde liegen.

    sind zwei komplett unterschiedliche Aussagen. Und selbstverständlich lassen sich die Effekte, die MOND zu erklären versucht, mit Krümmung der Raumzeit erklären. Das ist genau das was bei der Erklärung dieser Effekte mit dunkler Materie passiert. Wortwörtlich genommen ist diese Aussage also eine Trivialität.

    Dem in MOND gefundenen Zusammenhang könnte eine Raumzeitkrümmung ohne dunkle Materie zugrunde liegen.

    Wie ich bereits mehrmals sagte: Es reicht nicht eine Krümmung zu postulieren. Das erklärt gar nichts. Da kannst du genauso gut postulieren, dass sich Sterne im Außenbereich der Galaxien einfach nicht an die Bewegungsgleichungen halten. Das hätte die selbe “Erklär-Kraft”. Du musst erklären warum es diese Krümmung gibt.

    Und wenn du die Feldgleichungen der ART verändern willst: Das ist wie gesagt nicht einfach (solange man nicht ganz grundlegende Prinzipien über Bord werfen will). Und viele der Varianten die noch am einfachsten sind, sind experimentell widerlegt. Die übrig bleibenden Varianten sind genau die relativistischen MOND-Theorien.

    Es gäbe dort eine andere Quelle der Raumkrümmung als den Energie-Impuls-Tensor. Zum Beispiel eine Art Quantisierung des Raumes.

    Tut mir leid, aber das halte ich für Unsinn. Die Skala auf der die Raumzeit möglicherweise quantisiert ist, liegt von den hier relevanten Skalen viel zu weit entfernt (Planck-Skala vs. 10000e von Lichtjahren). Und Quantisierungseffekte sind auf kleineren Skalen relevanter als auf größeren. D.h. wenn die Raumzeit auf der Skala von Galaxien quantisiert wäre, dann gäbe es auf den uns vertrauten Skalen keine kontinuierliche Raumzeit.

    Du kannst nicht einfach eine wilde Idee äußern und dich dann Fragen warum niemand daran forscht. Insbesondere wenn diese Idee hochgradig unplausibel ist. Du musst es wenigstens plausibel machen dass diese Idee überhaupt eine Chance hat den Effekt korrekt zu beschreiben.

  257. #258 alex
    8. Oktober 2018

    Das Bild wird in #179 plausibel gemacht.

    Tut mir leid, aber meiner Meinung nach ist das einfach nicht der Fall. Es wird für mich aus Kommentar #179 (oder deinen anderen Kommentaren) nicht einmal klar, was genau “dünner werdender Raum” eigentlich sein soll. Und dass das in irgend einer Form äquivalent zu dunkler Materie sein soll, ist für mich noch viel weniger plausibel.

    Aber das hatten wir im vergangenen Monat so oft besprochen, ohne dass sich irgend etwas daran geändert hätte.

  258. #259 Randerscheinung
    8. Oktober 2018

    Es reicht nicht eine Krümmung zu postulieren. Das erklärt gar nichts. Da kannst du genauso gut postulieren, dass sich Sterne im Außenbereich der Galaxien einfach nicht an die Bewegungsgleichungen halten. Das hätte die selbe “Erklär-Kraft”. Du musst erklären warum es diese Krümmung gibt.

    Ich postuliere ja nicht die Krümmung. Ich sage nur, dass MOND äquivalent sein könnte zu einer Krümmung, die nicht von der dunklen Materie herrührt. Und nicht von der ART erfasst wird. Das ist ein Schritt weiter als nur MOND. Es würde den Gravitationslinseneffekt erklären. Und es zeigt auf, dass es vielleicht außerhalb der ART (ohne Energie-Impuls-Tensor) einen Effekt geben könnte, der eine Krümmung erzeugt.

    Die Skala auf der die Raumzeit möglicherweise quantisiert ist, liegt von den hier relevanten Skalen viel zu weit entfernt (Planck-Skala vs. 10000e von Lichtjahren).

    Nein, “10000e von Lichtjahren” ist nicht die hier relevante Skala. Die “Kleinheit” der Beschleunigung ist die relevante Skala. Und die ist eben doch im Bereich der Planck-Skala. Wenn der Raum ab dieser kleinen Beschleunigung quanitisert wäre dann würde sich dass auf einem großen Raumbereich auswirken. In diesem ganzen Raumbereich wäre dann möglicherweise der Raum gekrümmt wegen der Quantisierung. Die Skala ist schon mal kein Widerspruch.

    Außerdem haben Naturgesetze selten “Erklär-Kraft”. Stattdessen eher “Bechreib-Kraft”.
    Und klar, weit geht das nicht…

  259. #260 alex
    8. Oktober 2018

    Ich postuliere ja nicht die Krümmung.

    Doch. Das ist exakt was du tust. Du sagst: Die Sterne im Außenbereich von Galaxien bewegen sich anders als erwartet, weil dort die Krümmung anders ist als erwartet. Damit postulierst du dort Krümmung. Und du sagst nicht woher diese Krümmung kommen soll.

    Das ist ein Schritt weiter als nur MOND.

    Nein, es ist ein Schritt weniger als die üblichen MOND-Theorien. Denn diese haben eine mathematische Beschreibung des Effekts. Z.B. hat TeVeS Feldgleichungen, mit denen man die Krümmung, die Bewegung von Probekörpern, etc. berechnen kann. Das hast du nicht. Du postulierst Krümmung und mehr nicht.

    Wenn der Raum ab dieser kleinen Beschleunigung quanitisert wäre …

    Was soll das bedeuten dass der Raum ab einer gewissen Beschleunigung quanitisert ist? Beschleunigung ist keine Eigenschaft des Raumes.

    … dann würde sich dass auf einem großen Raumbereich auswirken.

    Warum?

    In diesem ganzen Raumbereich wäre dann möglicherweise der Raum gekrümmt wegen der Quantisierung.

    Warum?

    Soweit ich das sehe, versuchst du hier haltlose Behauptungen mit weiteren haltlosen Behauptungen zu belegen.

    Außerdem haben Naturgesetze selten “Erklär-Kraft”. Stattdessen eher “Bechreib-Kraft”.

    Das ist eine Frage davon was man unter “erklären” versteht. Momentan sagst du, dass sich Sterne anders bewegen als erwartet, weil die Raumzeit anders gekrümmt ist als erwartet. Und an dieser Stelle hört deine “Erklärung” oder “Beschreibung” auf. Und das ist jedenfalls deutlich schwächer, als wenn Dunkle-Materie-Forscher sagen: “Sterne bewegen sich anders als erwartet, weil die Raumzeit anders gekrümmt ist als erwartet, weil dort mehr Materie ist als erwartet”. Und es ist auch schwächer als wenn MOND-Forscher sagen: “Sterne bewegen sich anders als erwartet, weil die Gleichungen mit denen wir diese Vorgänge beschreiben nicht stimmen, und die richtigen Gleichungen sind …”

  260. #261 alex
    8. Oktober 2018

    Ach ja, hierfür

    Die “Kleinheit” der Beschleunigung ist die relevante Skala. Und die ist eben doch im Bereich der Planck-Skala.

    wäre ein Beleg auch ganz nett.

    Die “Planck-Beschleunigung”, also Planck-Länge dividiert durch das Quadrat der Planck-Zeit ist im Bereich von 10^50 Meter/Sekunde^2. Die MOND-Beschleunigung ist im Bereich von 10^-10 Meter/Sekunde^2. Das würde ich jetzt nicht exakt die selbe Größenordnung nennen…

  261. #262 Randerscheinung
    8. Oktober 2018

    Klar, die ganze Geschichte mit der Quantisierung ist ganz schön spekulativ. Das ist nur so eine Idee. Milgrom hat in seinem Original-Paper was von einem Unterschied der Masse bei kleinen Beschleunigungen geschrieben. Das will ich auch gar nicht behandeln. Nur: Könnte nicht eine Raumkrümmung außerhalb der Erklärungen der ART für den MOND-Effekt verantwortlich sein?

  262. #263 alex
    8. Oktober 2018

    Klar, die ganze Geschichte mit der Quantisierung ist ganz schön spekulativ.

    Meiner Ansicht nach ist das nicht nur spekulativ sondern komplett unplausibel. 60 Größenordnungen kehrt man nicht einfach so unter den Teppich.

    Milgrom hat in seinem Original-Paper was von einem Unterschied der Masse bei kleinen Beschleunigungen geschrieben.

    Was hat das mit der Quantisierung der Raumzeit zu tun?

    Könnte nicht eine Raumkrümmung außerhalb der Erklärungen der ART für den MOND-Effekt verantwortlich sein?

    Ich habe den Eindruck mich ständig zu wiederholen. Selbstverständlich kann man den Effekt mit Krümmung beschreiben. Genau das macht man schließlich bei dunkler Materie und soweit ich weiß auch bei einigen relativistischen MOND-Varianten.

    Und bleibt man im Theorierahmen der ART (also mit dem Universum als einer 3+1-dimensionalen semi-riemannschen Mannigfaltigkeit, in der sich Teilchen, auf die außer der Gravitation keine weiteren Kräfte wirken, auf Geodäten bewegen), dann ist die einzige Erklärung die nicht mit Krümmung arbeitet eine fünfte Kraft.

    Die entscheidende Frage ist aber: Woher kommt diese Krümmung? Warum ist sie gerade dort, warum hat sie gerade diese Stärke und “Richtung”? Oder mathematischer formuliert: Wenn du diese Krümmung ohne dunkle Materie haben willst, wie genau willst du die Feldgleichungen abändern?

    Sowohl dunkle Materie als auch diverse MOND-Varianten liefern Antworten auf diese Fragen. Du hast das bislang nicht getan. Und du hast auch nicht dargelegt, worin sich deine Idee dann eigentlich von MOND unterscheiden soll.

    Denn es gibt doch genau diese zwei Erklärungsansätze: Entweder unsere Gleichungen sind korrekt, dann gibt es Materie die wir nicht sehen können, also dunkle Materie. Oder die Gleichungen sind nicht korrekt. Und genau das bezeichnet man als MOND.

  263. #264 Randerscheinung
    8. Oktober 2018

    Oder die Gleichungen sind nicht korrekt. Und genau das bezeichnet man als MOND.

    Ok, also MOND ist gleich Krümmung, die von irgendwo her kommt außerhalb der ART?
    Aber die originale MOND sagt ja nichts über die Krümmung. Nur über Newton…

    Die relativistischen MOND-Varianten machen da schon eher was mit Krümmung. Sagst du ja.

    Ich kann beim besten Willen nicht alles erklären. Hab ich das jemals versprochen? Ich hab wirklich nur dieses Bild (179) vor Augen und versuche es zu verteidigen.
    Klar, der eigentliche Effekt fehlt in meinem Bild. Wird nicht erklärt. Warum dieser Raum dort dünner werden sollte – keine Ahnung. Irgendeine ungeklärte Randerscheinung (im Sinne von Randbedingung, weil a kleiner a0).
    Aber ob man dunkle Materie hinzufügt, deren Natur man nicht kennt – oder Raum wegnimmt, ohne das zugrunde liegende Gesetz zu kennen ist doch gehupft wie gesprungen! Ist das nun falsch in deinen Augen oder trivial oder nicht klärbar?

  264. #265 alex
    9. Oktober 2018

    Ok, also MOND ist gleich Krümmung, die von irgendwo her kommt außerhalb der ART?

    MOND als Überbegriff heißt zunächst einmal nur, dass man davon ausgeht, dass die Gleichungen mit denen man die Sache bisher beschreibt falsch sind. Und dass man daher Vorschläge für die richtigen Gleichungen macht und diese dann untersucht.

    Für die Bewegung von Sternen im Außenbereich von Galaxien und die Bewegung von Galaxien in Galaxiehaufen sind relativistische Effekte vernachlässigbar klein. Deshalb kann man versuchen, zunächst die Newtonsche Theorie zu modifizieren. Das ist wesentlich anschaulicher und auch von der Mathematik her wesentlich einfacher. Natürlich ist dabei klar, dass das Resultat dann nicht die endgültige Theorie sein kann. Aber man hat zumindest einen Anhaltspunkt.

    Nicht jede MOND-Theorie ist “Krümmung, die von irgendwo her kommt außerhalb der ART”. In klassischen nichtrelativistischen Theorien gibt es ja keine Krümmung. Und man könnte sich auch in einer relativistischen Theorie überlegen, ob man statt der Feldgleichungen die Geodätengleichung abändert. (In Milgroms ursprünglicher MOND wird ja auch nicht das Gravitationsgesetz geändert, sondern das 2. Newtonsche Axiom.) Oder man könnte sich überlegen den Theorierahmen der ART ganz zu verlassen. Je weiter man sich von der ART entfernt, desto schwieriger wird es natürlich, die bestätigten Resultate der ART zu reproduzieren. Und im Prinzip könnte man auch eine fünfte Kraft einführen. Aber das würde man vermutlich nicht mehr als MOND bezeichnen.

    Aber wenn du möchtest, dass die Krümmung der Raumzeit nicht diejenige ist, die man mit den Feldgleichungen der ART aus der Massenverteilung bekommt, du also die Feldgleichungen modifizieren willst, dann ist das in meinen Augen ein Ansatz der in die Kategorie MOND fällt. Nur ist das (wie ich bereits mehrmals sagte) absolut nicht trivial. Das sieht man schon daran, dass Ansätze wie TeVeS wesentlich komplizierter als die ART sind.

    Ich kann beim besten Willen nicht alles erklären. Hab ich das jemals versprochen?

    Naja, du machst einen Vorschlag der signifikant weniger erklärt als die bisherigen Theorien. Da musst du dich nicht wundern dass niemand daran arbeitet.

    Aber ob man dunkle Materie hinzufügt, deren Natur man nicht kennt – oder Raum wegnimmt, ohne das zugrunde liegende Gesetz zu kennen ist doch gehupft wie gesprungen! Ist das nun falsch in deinen Augen oder trivial oder nicht klärbar?

    In meinen Augen ist nicht einmal klar, was genau “Raum wegnehmen” eigentlich bedeuten soll. Die ART funktioniert nicht so, dass Materie “Raum wegnimmt”. Wenn ich mich richtig erinnere, hast du das bislang nicht erklärt. Von daher ist es nicht klärbar.

    Wenn du damit meinst, dass du exakt die dunkle-Materie-Metrik reproduzierst (wie auch immer, und obwohl das meiner Meinung nach absolut nichts mit “Raum wegnehmen” zu tun hat; insbesondere weil im Newtonschen Grenzfall in dem wir uns ja befinden die Metrik des Raumes gar nicht relevant ist), dann ist die Aussage trivial. Selbstverständlich bekommt man die selben Bewegungen, wenn man von der selben Metrik ausgeht. Nur hast du das in diesem Fall dadurch erkauft, dass deine “Theorie” absolut nichts erklärt. Du sagst dann nur “die Metrik sieht so aus als gäbe es dunkle Materie, aber in Wirklichkeit gibt es keine dunkle Materie” und nichts weiter. Ich hoffe es ist klar dass das keine besonders sinnvolle Theorie ist.

    Und wenn du damit etwas anderes meinst, dann ist die Aussage meiner Ansicht nach vermutlich falsch. Ich sehe nicht, weshalb zwei Metriken die signifikant unterschiedlich sind (also auch modulo Diffeomorphismen) zu den selben Bewegungen führen sollen.

    Wenn du Übereinstimmung nicht für alle möglichen Trajektorien forderst, sondern nur für eine geschickt gewählt Teilmenge, dann gibt es natürlich solche unterschiedlichen Metriken. (Trivialerweise wenn z.B. die Trajektorien ein gewisses Gebiet der Raumzeit ganz aussparen; denn dann kann man die Metrik dort natürlich frei wählen.) Aber dann musst du wesentlich besser klarmachen, wie sich deine Metrik und die dunkle-Materie-Metrik unterscheiden, und weshalb diese Unterschiede im betrachteten Fall keine Rolle spielen. Und du musst selbstverständlich auch erklären, wie man auf deine Metrik kommt, d.h. wie deine modifizierten Feldgleichungen aussehen sollen.

    Es ist ja nicht so, dass man einfach die Existenz von dunkler Materie postuliert und es dann dabei belässt. Das wäre in der Tat ähnlich zu deiner postulierten Metrik ohne jegliche Erklärung. Im Gegenteil, man macht Beobachtungen, Experimente, Rechnungen, und Simulationen um mehr über die dunkle Materie herauszufinden. Man kennt ja heute ein ganze Reihe an Eigenschaften der dunklen Materie und hat ein Modell entwickelt, das von der Nukleosynthese, über die Kosmologie, und die Dynamik in Galaxienhaufen, bis zur Dynamik in Galaxien weitestgehend konsistent mit den Beobachtungen ist.

  265. #266 Randerscheinung
    9. Oktober 2018

    Ich wage mich noch mal in den Dschungel der Spekulationen (das folgende wäre sogar rechnerisch leicht nachprüfbar):
    1. Ich nehme an, es gibt so etwas wie eine Quantelung der Raumkrümmung, Raum kann ohne ein Mindestmaß an Krümmung nicht sein, es gibt ein Mindestmaß an Krümmung, den flachen Raum gibt es nicht.
    2. Das könnte vielleicht zur Folge haben, dass die Beschleunigung pro Fläche (oder Volumen) nicht kleiner als a0/m/m werden darf.
    3.Da die Beschleunigung pro Fläche (oder Volumen) nicht einen bestimmten Wert unterschreiten darf, dehnt sich der Raum aus wenn a kleiner a0 gilt. Diese Raumausdehnung führt zum Dünnerwerden des Raums. =>Raumkrümmung.

    Ich wollte ja schon mal V gegen 0 für m gegen 0 zeigen, um meine umgekehrte Krümmung in die Feldgleichungen zu pressen -was nicht klappt, wie wir ja inzwischen wissen. Damals hatte ich das Problem mit der Begrenztheit von V – was ich jetzt nicht habe. Nun kann das Volumen ruhig unendlich sein. Und trotzdem außen der Raum immer dünner.

    Ich befürchte aber fast, dass du wieder sagst, du verstehst es nicht. Vorher müsstest du 179 verstehen und dass die Raumkrümmung der Feldgleichungen, die durch den Energie-Impuls-Tensor kommt, einem “Dichterwerden” des Raums entspricht (und einem Langsamerwerden der Zeit). Und dass das Umgekehrte dann eine Ausdehnung des Raumes und ein Schnellerwerden der Zeit wäre.

    Ich weiß nicht, ob einige der relativistischen MOND-Varianten so etwas machen was ich mir vorstelle.

    Was meinst du eigentlich mit “fünfter Kraft”? Und was sind die anderen vier?

    Was in den Feldgleichungen nicht enthalten ist, MOND ansatzweise hat und ich in den relativistischen MOND-Varianten noch nicht sehe möchte ich der Theorie hinzufügen: Die Konvergenz gegen eine Randbedingung.

  266. #267 Randerscheinung
    9. Oktober 2018

    In meinen Augen ist nicht einmal klar, was genau “Raum wegnehmen” eigentlich bedeuten soll. Die ART funktioniert nicht so, dass Materie “Raum wegnimmt”.

    Daran merkt man, dass du den Punkt wirklich noch nicht verstanden hast. Materie “nimmt” nicht Raum weg, sondern fügt wenn dann hinzu. Das Volumen mit zentraler Masse ist größer als ohne bei gleichem Unfang, steht in Wikipedia bei der Schwarzschild-Metrik. Das ist ein ganz wichtiger Punkt in meiner Vorstellung!
    Wenn wir uns darüber nicht einig sind kann der ganze Rest nix werden.
    Ich schaue mir oft Verzerrbilder an, zB. diese verzerrten Kästchen, die du bei “kissenförmige” und “tonnenförmige Verzeichnung” in Google findest. Und bin überzeugt, dass so etwas in 3D auch die Raumkrümmung in der ART darstellt. Und die Zeit wird bei Masse langsamer(schwarze Löcher). Das ist im Prinzip das Kernstück meiner ganzen Mini-Theorie. Das ist eine fast comichafte Vorstellung, sehr einfach und sehr bildhaft, aber ich bin sicher, dass sie stimmt. Kannst du mir so weit folgen?

  267. #268 Randerscheinung
    9. Oktober 2018

    ok, die Frage nach der fünften Kraft war ganz schön dumm… Ich dachte, du meinst spezielle vier Kräfte auf die Kosmologie bezogen… Wie auch immer. Ich hoffe, Du verlässt mich nicht wegen solcher Peinlichkeiten (auch die Frage nach der Lagrange-Dichte war so eine…) und gehst trotzdem auf den anderen Kram ein…
    Ich schau mir derweil mal die Quantengravitation und MOG an. Im (englischen) Wiki zur QG steht komischerweise, dass die Quanteneffekte in Bereichen hoher Gravitation nicht vernachlässigt werden können:

    “Quantum gravity (QG) is a field of theoretical physics that seeks to describe gravity according to the principles of quantum mechanics, and where quantum effects cannot be ignored,[1] such as near compact astrophysical objects where the effects of gravity are strong.”

    Das widerspricht ja meiner Vorstellung. Aber vor allem widerspricht es doch der allgemeinen Vorstellung des Geltungsbereichs der Quantentheorie, die doch immer nur für kleine Energien, Felder, Kräfte, gilt??!

  268. #269 alex
    9. Oktober 2018

    das folgende wäre sogar rechnerisch leicht nachprüfbar

    Wie? Was du beschreibst klingt für mich nicht nach einem einigermaßen klaren mathematischen Modell. Wie soll man damit also etwas berechnen können?

    Ich nehme an, es gibt so etwas wie eine Quantelung der Raumkrümmung,

    Was genau soll das bedeuten? Und nur als Hinweis, falls das nicht klar sein sollte: In der Quantenmechanik postuliert man nicht, dass bestimmte Größen quantisiert sind. Die Quantisierung ist eine Folgerung die sich aus der Theorie ergibt.

    Das könnte vielleicht zur Folge haben, dass die Beschleunigung pro Fläche (oder Volumen) nicht kleiner als a0/m/m werden darf.

    Warum? Und in der ART ist die Beschleunigung eines Teilchens nicht nur eine Funktion seiner aktuellen Position in der Raumzeit.

    Da die Beschleunigung pro Fläche (oder Volumen) nicht einen bestimmten Wert unterschreiten darf, dehnt sich der Raum aus wenn a kleiner a0 gilt.

    Warum?

    Ich befürchte aber fast, dass du wieder sagst, du verstehst es nicht.

    Damit hast du recht. Ehrlich gesagt klingt das alles nach weitgehend unzusammenhängenden und haltlosen Spekulationen.

    Was in den Feldgleichungen nicht enthalten ist, MOND ansatzweise hat und ich in den relativistischen MOND-Varianten noch nicht sehe möchte ich der Theorie hinzufügen: Die Konvergenz gegen eine Randbedingung.

    Was soll das bedeuten?

    Materie “nimmt” nicht Raum weg, sondern fügt wenn dann hinzu.

    Nein, das stimmt einfach nicht. Oder (je nachdem was genau “Raum hinzufügen” bedeuten soll) ist das eine so starke Verzerrung der Realität, dass man damit wirklich nichts anfangen kann.

    Und bin überzeugt, dass so etwas in 3D auch die Raumkrümmung in der ART darstellt.

    Tja, was soll man dazu sagen …

    Das widerspricht ja meiner Vorstellung.

    Deine Vorstellung ist falsch.

  269. #270 Randerscheinung
    9. Oktober 2018

    Sehr interessant finde ich das Paper von Calmet und Kuntz 2017: “What is modified gravity and how to differentiate it from particle dark matter?”
    Wenn ich es richtig verstehe, dann bedeutet deren Berechnung, dass neue Tensoren/Skalare/Vektoren in die ART einzufügen (in dem Versuch, dark matter ohne Partikel zu erklären) eine Erzeugung neuer Freiheitsgrade in der Partikelwelt nach sich zieht. Das hat zur Folge, dass sich diese Theorien nicht wirklich qualitativ voneinander unterscheiden:

    We conclude that any attempt to modify the Einstein-Hilbert action, preserving
    the underlying symmetry, leads to new degrees of freedom, i.e., new particles. In that sense, this is not different from including new matter fields by hand in the matter sector that are coupled gravitationally to the standard model matter fields.

    —-
    Das festigt meine Überzeugung, dass meine Randbedingungs-Idee doch etwas neues, anderes ist.
    Nach meiner Vorstellung muss man nicht etwas hinzufügen, sondern im Gegenteil etwas wegnehmen. Das ist etwas Neues. Und es ist offensichtlich nicht trivial (auch wenn das Bild trivial ist). Verstehst du? Bitte versteh!

  270. #271 Randerscheinung
    9. Oktober 2018

    In der Quantenmechanik postuliert man nicht, dass bestimmte Größen quantisiert sind. Die Quantisierung ist eine Folgerung die sich aus der Theorie ergibt.

    Ok, wie? Und woher? Bzw. ich kann ja auch nicht einfach irgendwelche Theorien in die Welt setzen (wie man an unserer Diskussion sieht) und dann passt alles…

    Und in der ART ist die Beschleunigung eines Teilchens nicht nur eine Funktion seiner aktuellen Position in der Raumzeit.

    Aber eine nur Funktion seiner aktuellen Position plus Krümmung an dieser Stelle der Raumzeit?

    Materie “nimmt” nicht Raum weg, sondern fügt wenn dann hinzu.

    Nein, das stimmt einfach nicht. Oder (je nachdem was genau “Raum hinzufügen” bedeuten soll) ist das eine so starke Verzerrung der Realität, dass man damit wirklich nichts anfangen kann.

    Raum hinzufügen nicht unbedingt. Ich meine eben ein “Dichterwerden” des Raumes… Stichwort kissenförmige Verzeichnung.
    Ok. Dann sind wir jetzt genau an dem Punkt, wo meine falsche Vorstellung entwurzelt werden muss. Dann erkläre mir bitte so dass ich es verstehe, wieso das falsch ist oder eine Verzerrung der Realität. Ich bin wirklich gespannt!

  271. #272 alex
    9. Oktober 2018

    Aber eine nur Funktion seiner aktuellen Position plus Krümmung an dieser Stelle der Raumzeit?

    Nein.

    Tut mir leid, aber ich kann dir nicht die Grundlagen von Quantenmechanik und allgemeiner Relativitätstheorie auf die Schnelle in ein paar wenigen Sätzen erklären. Vor allem nicht ohne vorher jede Menge Mathematik einzuführen. Ein Physik-Studium dauert nicht ohne Grund mehrere Jahre.

    Im Blog von Martin Bäker nebenan gibt es ein paar Artikelserien die vielleicht als Einstieg dienen können.

  272. #273 Randerscheinung
    9. Oktober 2018

    Ok, macht nichts.
    Dann erkläre mir wieso mein Bild der Raumkrümmung falsch ist.

  273. #274 Randerscheinung
    9. Oktober 2018

    Martin Bäker, Hier wohnen Drachen, “Das Märchen von Gummituch und Raumkrümmung II: Wie man sich (ein)bettet, so krümmt man”
    Alex, ich weiß nicht ob das die richtige Empfehlung war wenn du mir im Endeffekt meine Vorstellung von der Raumkrümmung ausreden willst – ich fühle mich hiervon bestätigt:

    Wenn ihr einen Kreis um die Kugel malt und dessen Umfang messt, dann sollte der nach der Gleichung U=2πr mit dem Radius zusammenhängen. Wenn ihr aber tatsächlich ins Innere der Kugel eindringt und den Weg messt, dann stellt ihr fest, dass dieser Weg länger ist, als ihr nach dieser Formel erwarten würdet. Es ist sozusagen nach Innen hin “mehr Raum” da, als man eigentlich erwarten würde (das ist eine etwas ungenaue Beschreibung, aber als Veranschaulichung finde ich sie ganz gut).

    Und diese beiden Zitate erklären die Bewegung:

    da geht es nur darum, welche Wege die kürzesten sind.

    …Krümmung der Raumzeit. Die sorgt dafür, dass eine ruhende Kugel, die sich in der Nähe einer Masse befindet und bei der deshalb die Raumzeit gekrümmt ist, bewegt.

    Wenn also mit der Raumzeit ab a kleiner a0 das Gegenteil passiert von dem, was mit ihr in der Nähe der Masse geschieht, dann hat das eine Wirkung, als wäre da zusätzliche anziehende Materie.

    Also, noch mal: Du denkst, dass mein Bild falsch ist. Bitte erklär mir, warum!

    Ich halte das Gummituch-Modell im Übrigen auch für völlig falsch. Es impliziert ja, dass der Raum in einer höheren Dimension gekrümmt wird. Das ist ja aber gerade nicht der Fall.

  274. #275 alex
    9. Oktober 2018

    Wenn also mit der Raumzeit ab a kleiner a0 das Gegenteil passiert von dem, was mit ihr in der Nähe der Masse geschieht, dann hat das eine Wirkung, als wäre da zusätzliche anziehende Materie.

    Warum denn?

    Ich halte das Gummituch-Modell im Übrigen auch für völlig falsch. Es impliziert ja, dass der Raum in einer höheren Dimension gekrümmt wird.

    Nein, das tut es nicht.

    Das ist ja aber gerade nicht der Fall.

    Das stimmt so auch nicht. In der ART spielt nur die Krümmung eine Rolle, die eine Auswirkung auf die Geometrie der Raumzeit selbst hat. Ob es einen Einbettungsraum gibt oder nicht ist damit nicht gesagt. Und ich würde mal vermuten, dass es auch für semi-riemannsche Mannigfaltigkeiten ein Analogon zum Einbettungssatz von Nash gibt …

  275. #276 Randerscheinung
    10. Oktober 2018

    “Raumzeit wird immer dünner” bedeutet für mich, der Raum wird weniger dicht und die Zeit wird kontrahiert, schneller. Eben einfach das Gegenteil der Raumzeitkrümmung aus der ART.
    Das ist das Bild aus #179, nur auf 4D angewendet.
    Es gibt bisher in unserer Vorstellung (und in der ART) einfach kein “Dünnerwerden der Raumzeit”. Daher ist das so schwer zu vermitteln. Aber es ist einfach das mathematische Gegenteil der Raumzeitkrümmung aus der ART. Rein rechnerisch ist es nicht falsch.

  276. #277 alex
    10. Oktober 2018

    Rein rechnerisch ist es nicht falsch.

    Dann zeige mir doch einfach diese Rechnung.

  277. #278 Randerscheinung
    16. Oktober 2018

    Könnte aber dauern…

  278. #279 Randerscheinung
    16. Oktober 2018

    Das könnte aber ein Weilchen brauchen.

  279. #280 Randerscheinung
    26. Oktober 2018

    Wir beobachten eine Galaxie mit dunkler Materie T’:

    R_\mu_\nu - \frac{R}{2}g_\mu_\nu = T_\mu_\nu + T'_\mu_\nu 	\hspace{30}	(1)

    Die Raumkrümmung, die zur dunklen Materie gehört, ist:

    T'_\mu_\nu = R'_\mu_\nu- \frac{R'}{2}g'_\mu_\nu 		\hspace{30}		(2)

    Einsetzen von (2) in (1) ergibt:

    R_\mu_\nu - \frac{R}{2}g_\mu_\nu -  R'_\mu_\nu + \frac{R'}{2}g'_\mu_\nu = T_\mu_\nu \hspace{20} (3)

    Nun nehme ich mir heraus, beides im o.B.d.A. Im euklidischen Raum zu betrachten, damit gilt:

    g_\mu_\nu = g'_\mu_\nu	\hspace{30}	(4)

    Weiteres einfaches Umformen ergibt:
    R_\mu_\nu -  R'_\mu_\nu - \frac{R-R'}{2}g_\mu_\nu = T_\mu_\nu \hspace{20} (3)

    Nun haben wir dort eine „umgekehrte Krümmung“ unbekannter Ursache, die der dunklen Materie entgegengesetzt wäre.

  280. #281 Randerscheinung
    26. Oktober 2018

    Wir beobachten eine Galaxie mit dunkler Materie T’:

    R_{\mu}_{\nu} - \frac{R}{2}g_{\mu}_{\nu} = T_{\mu}_{\nu} + T'_{\mu}_{\nu} 	\hspace{30}	(1)

    Die Raumkrümmung, die zur dunklen Materie gehört, ist:

    T'_{\mu}_{\nu} = R'_{\mu}_{\nu}- \frac{R'}{2}g'_{\mu}_{\nu} 		\hspace{30}		(2)

    Einsetzen von (2) in (1) ergibt:

    R_{\mu}_{\nu} - \frac{R}{2}g_{\mu}_{\nu} -  R'_{\mu}_{\nu} + \frac{R'}{2}g'_{\mu}_{\nu} = T_{\mu}_{\nu} \hspace{20} (3)

    Nun nehme ich mir heraus, beides im o.B.d.A. Im euklidischen Raum zu betrachten, damit gilt:

    g_{\mu}_{\nu} = g'_{\mu}_{\nu}	\hspace{30}	(4)

    Weiteres einfaches Umformen ergibt:
    R_{\mu}_{\nu} -  R'_{\mu}_{\nu} - \frac{R-R'}{2}g_{\mu}_{\nu} = T_{\mu}_{\nu} \hspace{20} (3)

    Nun haben wir dort eine „umgekehrte Krümmung“ unbekannter Ursache, die der dunklen Materie entgegengesetzt wäre.

  281. #282 Randerscheinung
    26. Oktober 2018

    Wir beobachten eine Galaxie mit dunkler Materie T’:

    R_{\mu}_{\nu} - \frac{R}{2} g_{\mu}_{\nu} = T_{\mu}_{\nu} + T'_{\mu}_{\nu} 	\hspace{30}	(1)

  282. #283 Randerscheinung
    26. Oktober 2018

    Die “formula does not parse” – Posts dürfen gerne gelöscht werden… :-/

  283. #284 Randerscheinung
    26. Oktober 2018

    Wir beobachten eine Galaxie mit sichtbarer Materie T und dunkler Materie T’:

    R_{\mu\nu} - \frac{R}{2}g_{\mu\nu} = T_{\mu\nu} + T'_{\mu\nu} \hspace{30pt} (1)

    Die Feldgleichung, die nur zur dunklen Materie gehört, ist:

    T'_{\mu\nu} = R'_{\mu\nu}- \frac{R'}{2}g'_{\mu\nu} \hspace{30pt} (2)

    Einsetzen von (2) in (1) ergibt:

    R_{\mu\nu} - \frac{R}{2}g_{\mu\nu} - R'_{\mu\nu} + \frac{R'}{2}g'_{\mu\nu} = T_{\mu\nu}

    Weiteres einfaches Umformen ergibt:
    R_{\mu\nu} - R'_{\mu\nu} - \frac{R}{2} (g_{\mu\nu}+ \frac{R'}{R}g'_{\mu\nu}) = T_{\mu\nu} \hspace{20pt} (3)

    Nun haben wir dort eine „umgekehrte Krümmung“ unbekannter Ursache, die der Krümmung der dunklen Materie entgegengesetzt wäre.

  284. #285 alex
    26. Oktober 2018

    Gleichung (1) ist korrekt. Gleichung (2) ist physikalisch nicht sinnvoll und wohl auch mathematisch extrem problematisch. Man kann nicht einfach nur einen Teil der Materie betrachten und dann damit eine sinnvolle Metrik berechnen. Schon allein weil in T gewisse Informationen über die Metrik eingehen. Außerdem erfüllt die linke Seite der Feldgleichungen automatisch eine Kontinuitätsgleichung. Also würde nach deiner Gleichung (2) auch der gestrichene Energie-Impuls-Tensor eine Kontinuitätsgleichung erfüllen, und nach (1) dann auch der ungestrichene. Das heißt aber, dass sichtbare und dunkle Materie keine Energie und keinen Impuls austauschen können. Und das ist offensichtlich nicht der Fall.

    Die beiden restlichen Gleichungen sind dann triviale Umformungen, wobei du bei der letzten sogar noch einen Vorzeichenfehler eingebaut hast.

    Und selbst wenn man all das ignoriert, ist Gleichung (3) vollkommen nutzlos, solange du keine Möglichkeit hast, die gestrichenen Größen auf der linken Seite ohne Kenntnis des gestrichenen Energie-Impuls-Tensors zu bestimmen.

    Und nirgends kommt eine wie auch immer geartete “umgekehrte Krümmung” vor. (Und auch nach nun fast zwei Monaten hast du immer noch nicht sauber definiert, was das eigentlich sein soll.)

    Es kommt mir so langsam so vor als wolltest du mich verarschen.

  285. #286 alex
    26. Oktober 2018

    Ich hab mir gerade nochmal deine “Formula does not parse”-posts angeschaut. Insbesondere das hier (die Formel habe ich so korrigiert dass sie hoffentlich korrekt geparst wird):

    Nun nehme ich mir heraus, beides im o.B.d.A. Im euklidischen Raum zu betrachten, damit gilt: g_{\mu\nu} = g'_{\mu\nu}

    Zunächst einmal ist die Annahme dass die Geometrie euklidisch ist eine extrem große Beschränkung der Allgemeinheit. Denn das heißt ja, dass es keinerlei Krümmung und damit auch keine (Auswirkungen der) Gravitation gibt. Wenn du das tatsächlich annehmen würdest, müsstest du zu dem Ergebnis kommen, dass es keine Galaxien gibt.

    Und “euklidisch” heißt außerdem, dass die Metrik die Signatur + + + + hat, dass es also keinen Unterschied zwischen Raum und Zeit gibt (bzw. dass es nur Raum und keine Zeit gibt). Auch das ist offensichtlich Unsinn.

    Aber die Formel die du dann anführst hat mit “euklidischem Raum” rein gar nichts zu tun. Mit ihr behauptest du, dass die ungestrichene Metrik gleich der gestrichenen ist. Zusammen mit Gleichung (2) und Gleichung (1) führt das auf T_{\mu\nu} = 0, d.h. es gibt keine sichtbare Materie. Auch das ist nicht besonders sinnvoll.

  286. #287 Randerscheinung
    26. Oktober 2018

    zu #286: das der dort erwähnte Teil der “does not parse” – posts völliger Unsinn ist ist mir nach dem post selbst aufgefallen. Vz-Fehler: ja, schlimm.

    Man kann nicht einfach nur einen Teil der Materie betrachten und dann damit eine sinnvolle Metrik berechnen.”

    Aber bei Berechnungen zur Raumkrümmung der Sonne (beispielsweise Periheldrehung des Merkur) wird auch nicht die ganze Galaxis herangezogen. D.h., wenn die beiden T weit genug auseinander liegen, kann ich schon beide getrennt betrachten.

    Und nirgends kommt eine wie auch immer geartete “umgekehrte Krümmung” vor. (Und auch nach nun fast zwei Monaten hast du immer noch nicht sauber definiert, was das eigentlich sein soll.)

    Raum und Zeit würden “umgekehrt” gekrümmt zur Krümmung in der ART.

    In der ART bewirkt eine Krümmung immer eine Zeitdilatation und eine Längenkontraktion. An Punkten näher zur Masse gibt es (bei Relativgeschwindigkeit 0) immer eine Längenkontraktion und eine Zeitdilatation. “Umgekehrte Krümmung” wäre Längendilatation plus Zeitkontraktion.
    Außerdem wissen wir inzwischen immerhin:
    1.dass man eine Metrik für eine “umgekehrte Krümmung” in “einfacher Herleitung” berechnen könnte, die zumindest die Geschwindigkeit der äußeren Sterne richtig beschreibt. Es wäre sehr interessant, ob sie dies auch für den Gravitationslinseneffekt täte.
    2. dass eine umgekehrte Krümmung nicht durch Randbedingungen der Feldgleichungen und auch nicht durch negative Masse im Außenraum aus der ART berechnet werden könnte.

    Das heißt aber, dass sichtbare und dunkle Materie keine Energie und keinen Impuls austauschen können. Und das ist offensichtlich nicht der Fall.

    Woran sieht man denn, dass sie Energie und Impuls austauschen? Heißt es nicht immer, dunkle Materie wechselwirkt nicht, außer durch die Gravitation?

  287. #288 alex
    26. Oktober 2018

    Aber bei Berechnungen zur Raumkrümmung der Sonne (beispielsweise Periheldrehung des Merkur) wird auch nicht die ganze Galaxis herangezogen. D.h., wenn die beiden T weit genug auseinander liegen, kann ich schon beide getrennt betrachten.

    Das ist aber hier ganz offensichtlich nicht der Fall. Und dann rechnet man auch nicht mit zwei unterschiedlichen Metriken (wie du es hier versucht hast).

    Raum und Zeit würden “umgekehrt” gekrümmt zur Krümmung in der ART.

    Wo bitteschön steht das in deiner Gleichung?

    An Punkten näher zur Masse gibt es (bei Relativgeschwindigkeit 0) immer eine Längenkontraktion und eine Zeitdilatation.

    Das kann man meiner Ansicht nach so allgemein nicht sagen. Es wäre schön, wenn du für solche Behauptungen einen Beweis anbringen könntest.

    Außerdem wissen wir inzwischen immerhin:
    1.dass man eine Metrik für eine “umgekehrte Krümmung” in “einfacher Herleitung” berechnen könnte, die zumindest die Geschwindigkeit der äußeren Sterne richtig beschreibt.

    Nein. “Wir” wissen, dass man eine Metrik konstruieren kann, bei der die Rotationskurven einen beliebigen gegebenen Verlauf haben. Mit “umgekehrter Krümmung” hat das überhaupt nichts zu tun.

    Woran sieht man denn, dass sie Energie und Impuls austauschen? Heißt es nicht immer, dunkle Materie wechselwirkt nicht, außer durch die Gravitation?

    Der zweite Satz beinhaltet die Antwort auf die Frage aus dem ersten Satz.

  288. #289 Randerscheinung
    26. Oktober 2018

    An Punkten näher zur Masse gibt es (bei Relativgeschwindigkeit 0) immer eine Längenkontraktion und eine Zeitdilatation.

    Das kann man meiner Ansicht nach so allgemein nicht sagen. Es wäre schön, wenn du für solche Behauptungen einen Beweis anbringen könntest.

    Ich habe dafür leider keinen Beweis. Ich dachte, das wäre so. Und ich dachte, der Ausflug in Martins Blog hätte das bestätigt. Das ist wohl der entscheidende Punkt, an dem sich unsere Auffassungen unterscheiden bzw. wo meine Überzeugung (leider ohne Mathe) herkommt. Wenn dem nicht so ist, dann habe ich etwas ganz wesentliches falsch verstanden. Wenn Du nun also zeigen könntest, dass dem nicht so ist (beweisen, oder wenigstens plausibel machen), dann wäre ich jedenfalls einen Schritt weiter.
    Ich wiederhole: Das ist der wichtige Punkt, alles andere ist Geplänkel (und ein paar hilflose Versuche meinerseits, meine vage Idee mal schnell auf mathematische Beine zu stellen). Meine Vorstellung ist, dass es nahe der Masse im Vergleich zu weiter weg immer eine Längenkontraktion und Zeitdilatation gibt. Daher kommt auch die Vorstellung von der umgekehrten Krümmung – mehr ist es ja nicht – dass es rein theoretisch auch eine Zeitkontraktion mit Längendilatation geben könnte, die nicht von der Masse, sondern von etwas anderem (z.B. Abstand von der Masse) herrührt. Und die nicht in der ART beschrieben wird.

  289. #290 alex
    27. Oktober 2018

    Wenn Du nun also zeigen könntest, dass dem nicht so ist …

    Zunächst einmal gilt auch hier: Du hast eine Behauptung aufgestellt. Also ist es deine Aufgabe zu zeigen dass diese Behauptung stimmt oder wenigstes plausibel ist.

    Wenn du das nicht kannst, wäre es vielleicht sinnvoll, nicht ständig solche Behauptungen aufzustellen. Oder sie wenigstens klar als deine persönlichen Vermutungen darzustellen anstatt als unumstößliche Fakten.

    Ein meiner Ansicht nach ganz wesentliches Problem mit Aussagen wie

    An Punkten näher zur Masse gibt es (bei Relativgeschwindigkeit 0) immer eine Längenkontraktion und eine Zeitdilatation.

    ist, dass nicht wirklich klar ist (jedenfalls nicht mir), was genau du damit eigentlich meinst. Du verwendest eine ganze Reihe von Begriffen und Konzepten (“näher zur Masse”, “Relativgeschwindigkeit 0”, “Längenkontraktion”, “Zeitdilatation”), von denen man vielleicht eine vage Vorstellung hat, aber was exakt du damit jeweils meinst, ist nicht unbedingt klar. Wie würdest du diese Dinge z.B. (gedanken-)experimentell feststellen?

    Ein zweites großes Problem ist, dass du auch nach nun fast zwei Monaten immer noch keine Erklärung geliefert hast, warum deine “umgekehrte Krümmung” den beobachteten Effekt überhaupt erklären können soll. Bei dunkler Materie ist das offensichtlich. Bei den üblichen MOND-Varianten ist es ebenfalls einigermaßen klar. Bei “umgekehrter Krümmung” ist das überhaupt nicht klar. Wenn “umgekehrte Krümmung” den selben Effekt erklären soll wie dunkle Materie, und das ebenfalls nur über die Metrik vermittelt werden soll, dann wäre es das natürlichste, dass die Metrik im wesentlichen so aussieht wie bei dunkler Materie. Und in der dunkle-Materie-Metrik gibt es keine “umgekehrte Krümmung”. Auf diesen ganz offensichtlichen Widerspruch gehst du jedoch nicht ein.

    Soweit ich das sehe, gibt es in deinen Ideen mehrere große Lücken, die du einfach ignorierst. “Längenkontraktion” und “Zeitdilatation” sind nicht das selbe wie Krümmung. Und Krümmung ist nicht das selbe wie gravitative Anziehung. Und der Zusammenhang zwischen der gravitativen Anziehung und der Umlaufgeschwindigkeit von Sternen in einer Galaxie ist auch nicht ganz trivial. Selbstverständlich hängen all diese Dinge zusammen, aber dieser Zusammenhang ist nicht immer ganz einfach. Aber du tust so als wäre alles ungefähr das selbe, springst vom einen zum nächsten, und kümmerst dich nicht um die Zusammenhänge.

  290. #291 Randerscheinung
    27. Oktober 2018

    Deine Kritikpunkte aus #290 haben allesamt Berechtigung. Es liegt daran, dass ich wirklich, außer meiner Vorstellung/Idee, nichts Handfestes habe, keine Mathematik, die das belegt, keine Messungen. Außer dem, was bekannt ist. Und selbst das interpretiere ich manchmal falsch, rechne falsch, ziehe die falschen Schlüsse. Aber: erkenne die Probleme an, die Fehler. und sehe bisher keinen Widerspruch, keine Widerlegung der Grundidee. Allein, sie ist recht vage und es fehlt die mathematische Beschreibung.

    Ausgelöst durch dein (gutes) Argument, dass die linke Seite der Feldgleichung eine Kontinuitätsgleichung ist kam mir nun eine Idee, die mir sehr gut gefällt, und ich bin sehr gespannt, was du dazu sagst. Es ist wieder nur eine Idee, ohne Mathematik, aber sie zeigt einmal mehr, wo das ganze hingeht. Und sie löst einige Probleme. Die Idee ist (Trommelwirbel): Es gibt keine Grenze. Ich meine damit keinen Übergang, kein “hier gilt das eine und hier das andere”. Zwei Feldgleichungen. Eine beschreibt die Raumkrümmung durch die Masse, die andere beschreibt das Verschwinden des Raums in der Unendlichkeit. Zwei sich überlagernde Felder. Auch nahe der Masse wäre die “umgekehrte Krümmung” schon vorhanden – allerdings so klein, dass man sie nicht messen kann. Andererseits, im Unendlichen ist die Massenanziehung noch vorhanden. Was, dadurch, dass es sich eben bei der umgekehrten Krümmung nur um eine Raumkrümmung handelt, dazu führt, dass die Probeteilchen eine zusätzliche Beschleunigung in Richtung Massezentrum erfahren.
    Dann kann man eine Betrachtung anstellen, einerseits: Feld nahe der Masse=ART und andererseits: Feld fern von der Masse=umgekehrte Krümmung.

    r(a0) “zeigt” sich nur als Grenze: dort fängt die umgekehrte Krümmung an, sichtbar zu werden, weil a klein genug ist.
    Gleichung (2) von oben gilt für r >> r(a0) und T’>>T. Dann betrachte ich die Raumkrümmung, die wir messen. Und die ist der Raumkrümmung durch die Masse entgegengesetzt, das ergibt Gl. (3), mit richtigem Vorzeichen.

  291. #292 alex
    27. Oktober 2018

    … und sehe bisher keinen Widerspruch, keine Widerlegung der Grundidee.

    Das kann ich mir nur so erklären, dass du sämtliche Gegenargumente einfach ignorierst.

    Ausgelöst durch dein (gutes) Argument, dass die linke Seite der Feldgleichung eine Kontinuitätsgleichung ist …

    Bitte lege mir keine Aussagen in den Mund die ich nicht getätigt habe. Insbesondere wenn diese kompletter Unfug sind. Ich schrieb: “Außerdem erfüllt die linke Seite der Feldgleichungen automatisch eine Kontinuitätsgleichung.”

    …kam mir nun eine Idee, die mir sehr gut gefällt, und ich bin sehr gespannt, was du dazu sagst.

    Ich sage dazu genau das was ich seit fast zwei Monaten sage: Du kannst/willst anscheinend nicht erklären, was genau “umgekehrte Krümmung” eigentlich sein soll. Und du kannst/willst anscheinend nicht erklären, warum diese ominöse “umgekehrte Krümmung” den beobachteten Effekt korrekt beschreiben können soll. Unter diesen Voraussetzungen halte ich es für unmöglich, ernsthaft über Ideen zu diskutieren.

    Gleichung (2) von oben gilt für r >> r(a0) und T’>>T. Dann betrachte ich die Raumkrümmung, die wir messen. Und die ist der Raumkrümmung durch die Masse entgegengesetzt, das ergibt Gl. (3), mit richtigem Vorzeichen.

    Dass das Unsinn ist, habe ich dir bereits weiter oben erklärt.

  292. #293 Randerscheinung
    28. Oktober 2018

    Ich sage dazu genau das was ich seit fast zwei Monaten sage: Du kannst/willst anscheinend nicht erklären, was genau “umgekehrte Krümmung” eigentlich sein soll.

    Betrachten wir eine allgemeine statische, sphärisch symmetrische Metrik:

    ds^2 = B( r) c^2dt^2 -A( r) dr^2 - r^2 (d\theta^2 + sin^2\theta d\phi^2) \hspace{20pt} (1)

    und die Schwazschildmetrik mit

    B( r) = 1- \frac{r_s}{r}  \hspace{20pt} (2)

    und
    A( r) = \frac{1}{1-\frac{r_s}{r}} \hspace{20pt} (3).

    Für

    r_{s}  \leq r_{1} \leq r_{2} \hspace{20pt} (4)

    gilt in der Schwarzschildmetrik stets

    A(r_{1}) \leq A(r_{2} \hspace{20pt} (5),

    das ist die Längenkontraktion. Und außerdem

    B(r_{2}) \leq A(r_{1}) \hspace{20pt} (6),

    das ist die Zeitdilatation. Für r gegen unendlich geht A von oben gegen 1, B geht von unten gegen 1. A kann nicht kleiner als 1 werden, B kann nicht größer als 1 werden.
    Die “umgekehrte Krümmung” wäre ein B größer 1 und ein A kleiner 1. Eben genau die Metrik aus der “einfachen Herleitung”, die man über v = const. bekommt.

  293. #294 alex
    28. Oktober 2018

    … das ist die Längenkontraktion

    … das ist die Zeitdilatation

    Sollen das deine Definitionen der Begriffe “Längenkontraktion” und “Zeitdilatation” sein?

    Die “umgekehrte Krümmung” wäre ein B größer 1 und ein A kleiner 1.

    Also hätte “umgekehrte Krümmung” nichts mit Krümmung zu tun?

    Eben genau die Metrik aus der “einfachen Herleitung”, die man über v = const. bekommt.

    Wie kommst du auf die Idee?

  294. #295 Randerscheinung
    28. Oktober 2018

    Also hätte “umgekehrte Krümmung” nichts mit Krümmung zu tun?

    Wieso? Diesen Satz verstehe ich nicht.

  295. #296 alex
    28. Oktober 2018

    Der reine Wert von A oder B sagt nichts über die Krümmung.

  296. #297 Randerscheinung
    28. Oktober 2018

    Der reine Wert von A oder B sagt nichts über die Krümmung.

    Wieso, natürlich haben A und B was mit der Krümmung zu tun! A( r) jedenfalls, und B( r).
    Bei dieser ganzen Betrachtung trenne ich die Krümmung des dreidimensionalen Raums A( r) und die Zeitdilatation. Die Krümmung des vierdimensionalen Raums ist in der äußeren Schwarzschildmetrik null, nicht aber die des dreidimensionalen Unterraumes. Falls du darauf hinaus willst. In der inneren Schwarzschildmetrik ist die Krümmung ungleich null, A( r) und B( r) sind aber ebenso stets kleiner bzw. größer als 1.

    Mal zur “einfachen Herleitung” der Metrik für a größer a0 und v = const.:
    Ich stelle die allgemeine Form der statischen, sphärisch symmetrischen Metrik auf. Daraus dann die Geodätengleichung. Berechne die Komponenten der Christoffelsymbole. Mit theta = Pi/2 fallen einige davon weg. Erhalte dann (Fließbach Gleichung 25.16):

    A(\frac{dr}{d\lambda})^2 + \frac {l^2}{r^2} - \frac{F^2}{B}=const.

    Ist dabei $latexdr/d\lambda$ die Bahngeschwindigkeit? Und kann ich einfach l=mrv setzen? Und F =0 für den “freien Fall”?
    Oder muss ich mir erst r(t) herleiten? Irgendwie steh ich auf dem Schlauch…

  297. #298 alex
    28. Oktober 2018

    Wieso, natürlich haben A und B was mit der Krümmung zu tun!

    Ich sagte: “Der reine Wert von A oder B sagt nichts über die Krümmung.” Siehst du tatsächlich den Unterschied nicht?

  298. #299 Randerscheinung
    28. Oktober 2018

    Du sagtest, in #294:

    Also hätte “umgekehrte Krümmung” nichts mit Krümmung zu tun?

    Ich vergleiche in #293 nicht einfach die Werte von A und B.
    Außerdem wäre bei der umgekehrten Krümmung R ungleich null, ebenso wie in der inneren Schwarzschild-Metrik. Das würden wir aber alles viel konkreter diskutieren können, wenn wir endlich die entsprechende Metrik berechnet hätten. Ich versuche nun also, die schon erwähnte “einfache Herleitung” einer Metrik im Außenbereich der Galaxie nachzuvollziehen.

    Kreisbahn um die Galaxie. r (t) ist konstant. Theta ist pi/2. Die Bahngeschwindigkeit ist dr/d\lambda . Ich verstehe nicht, wie man von Gleichung 25.16 im Fließbach auf eine Differentialgleichung für die Geschwindigkeit kommen soll… Was meinst du? Bin ich vorher schon irgendwo falsch abgebogen oder geht der Weg dort weiter?

  299. #300 alex
    28. Oktober 2018

    In Kommentar #293 sagtest du

    Die “umgekehrte Krümmung” wäre ein B größer 1 und ein A kleiner 1.

    Was soll das sein wenn nicht ein Vergleich der Werte von A und B mit einer Konstanten? Und exakt darauf bezog sich mein

    Also hätte “umgekehrte Krümmung” nichts mit Krümmung zu tun?

    Das sollte aus Kommentar #294 eigentlich offensichtlich sein.

  300. #301 Randerscheinung
    28. Oktober 2018

    Können wir bitte die Metrik berechnen? Dann wird alles viel konkreter.

  301. #302 alex
    28. Oktober 2018

    Ich habe reichlich wenig Interesse daran, das dir hier vorzurechnen. Im Fließbach stehen ja die Komponenten der Geodätengleichung. Damit ist die Rechnung nun wirklich nicht schwierig. Wie so oft in der ART liegt die Schwierigkeit darin, das Ergebnis richtig zu interpretieren.

  302. #303 Bullet
    28. Oktober 2018

    Warum nur habe ich den deutlichen Eindruck, daß Alex deutlich mehr Ahnung von dem hat, was er hier schreibt?

  303. #304 PDP10
    28. Oktober 2018

    @Bullet:

    Und es würde mich nicht wundern, wenn er in einer kleinen Stadt in Südwestfalen studiert hätte …

  304. #305 Randerscheinung
    29. Oktober 2018

    Ich setze für die Geschwindigkeit
    v= r\cdot\frac{d\phi}{d\tau} \hspace{20pt} (1)

    und für eine stabile Kreisbahn

    \frac{dr}{d\tau} = 0 \hspace{20pt} (2)

    in die radiale Bewegungsgleichung der ART ein und erhalte:

    1+\frac{c^2}{v_{0}^2}= B( r)(\frac{dt}{d\tau})^2 \hspace{20pt} (3)

    wobei v die konstante Geschwindigkeit im Außenraum ist.

  305. #306 Randerscheinung
    29. Oktober 2018

    oje… ein Fehler… es muss v^2/c^2 heißen (statt c^2/v^2) in (3).

    Allerdings kann ich dann mit dem dt/dtau nix anfangen.

    Vielleicht mit dB/dt = const die rechte Seite von (3) in dB/dtau umwandeln und dann integrieren? Dann wäre B=(1+v^2/c^2)*r weil alle anderen partiellen Ableitungen von B null sind?!

  306. #307 Randerscheinung
    29. Oktober 2018

    …wieder ein Fehler… Also ich kann das B in die Ableitung nach tau reinziehen, behalte dann aber ein 1/B. Dann steht dort dB/dr =sqrt(B (1+v^2/c^2)). Das kann ich dann integrieren zu

    $latex B( r) = \frac{2}{3}\sqrt{ r^3 } \sqrt {1 + \frac{v^2}{c^2}} \hspace{30pt} (4)

  307. #308 Randerscheinung
    29. Oktober 2018

    B( r) = \frac{2}{3}\sqrt{ r^3 } \sqrt {1 + \frac{v^2}{c^2}} \hspace{30pt} (4)

    Dabei ist dann v die konstante Geschwindigkeit im Außenraum der Galaxie.

    Einverstanden?

  308. #309 alex
    29. Oktober 2018

    @Randerscheinung:

    Ich setze für die Geschwindigkeit …

    Das ist problematisch, denn das ist in Grunde die Strecke die der Stern aus Sicht eines Beobachters mit konstantem r, θ, ϕ zurücklegt dividiert durch die Eigenzeit des Sterns die dabei vergeht. D.h. du misst die räumliche und zeitliche Distanz in unterschiedlichen Systemen.

    und für eine stabile Kreisbahn

    Naja, ob die Bahn stabil ist, ist damit nicht gezeigt.

    in die radiale Bewegungsgleichung der ART ein und erhalte: …

    Das ist nicht “die radiale Bewegungsgleichung der ART”. Das ist die Normierungsbedingung der Vierergeschwindigkeit.

    Keine Ahnung wie du auf Gleichung (4) kommst. Besonders sinnvoll sieht das Resultat jedenfalls nicht aus. Schon allein aus Dimensionsgründen. Und da ein konstanter Faktor in B stets durch Umskalierung von t beseitigt werden kann, würde deine Metrik effektiv nicht von v abhängen. Und anscheinend hast du ja die Geodätengleichung überhaupt nicht verwendet.

    Das ganze hier langweilt mich mittlerweile nur noch. Ich weiß noch nicht ob ich in Zukunft hier kommentieren werde; falls nicht, möchte ich dir für die Zukunft alles Gute und viel Spaß wünschen. Und als kleinen Ratschlag: Vielleicht solltest du versuchen, zunächst die physikalischen und mathematischen Grundlagen zu verstehen, bevor du behauptest sie revolutionieren zu können.

  309. #310 Randerscheinung
    29. Oktober 2018

    Besonders sinnvoll sieht das Resultat jedenfalls nicht aus. Schon allein aus Dimensionsgründen.

    stimmt. Allerdings könnte man ja einen konstanten Faktor mit der Einheit Länge hoch 3/2 davorsetzen, wie du sagst.

    Und anscheinend hast du ja die Geodätengleichung überhaupt nicht verwendet.

    Es ist praktisch Kapitel 25 im Fließbach, S. 140-141. Von der Geodätengleichung im Zentralfeld kommt man auf drei Gleichungen: Energie- und Impulserhaltung sowie Gleichung 25.15. Die hab ich in dem Skript Rela2.pdf der Uni Stuttgart ein bisschen besser für Schwarzschildkoordinaten gefunden (dort S. 31 oben). Da hab ich dann A, B, v und dr/dtau=0 eingesetzt, B ins Integral nach tau reingezogen und integriert.
    Die Geodätengleichung ist also schon die Grundlage.

    Ich will sie ja nicht revolutionieren, ich wollte ja nur meine kleine Idee checken…
    Ich würde mir wünschen, dass du noch ein bisschen weiter kommentierst. Unabhängig davon wünsche ich dir auch alles Gute und danke dir für die vielen schönen gutüberlegten Kommentare, es hat mich sehr gefreut.

  310. #311 Randerscheinung
    29. Oktober 2018

    oh mann, der Fehlerteufel… der konstante Faktor müsste Länge hoch minus 3/2 als Einheit haben…

  311. #312 PDP10
    29. Oktober 2018

    @alex:

    Entschuldige, wenn ich mich mit einer persönlichen Frage einmische:

    Aber hast du denn nun in der kleinen Stadt in Südwestfalen studiert?

  312. #313 Randerscheinung
    29. Oktober 2018

    Ich würde mir wünschen, dass du noch ein bisschen weiter kommentierst.

    Was zum Beispiel noch auf der Todo-Liste stünde wäre
    – den Gravitationslinseneffekt zu checken
    – zu checken, was TeVes und MOG eigentlich machen
    – leider ist ja nun doch in der Herleitung keine zweite Feldgleichung aufgetaucht(oder?*). Welche Feldgleichung würde zur obigen einer Metrik führen?
    – Was würde eine zweite Feldgleichung anderes machen als TeVes und MOG? Wäre es überhaupt etwas anderes? Wenn es zwei Felder sind, dann können die ja vielleicht auch in “der einen” Feldgleichung enthalten sein…
    Naja. Ein weites Feld. Ich hab jedenfalls (dank dir, Fließbach und anderen, ungenannten) schon das Gefühl, ein bisschen weiter gekommen zu sein. Und sorry für manche Uneinsichtigkeiten/Unüberlegtheiten.

    (*)Vielleicht ist ja Gleichung (3) eine Art Vorstufe einer solchen Feldgleichung. Vielleicht muss man “Zeitfeld” und “Raumfeld” wieder trennen und kommt dann auf sogar zwei zusätzliche Feldgleichungen. Naja. Jetzt geht wieder die Fantasie mit mir durch. 😉 Bye.

  313. #314 Randerscheinung
    29. Oktober 2018

    Du meinst, alex ist ein Schüler von Pavel Kroupa in Bonn?

  314. #315 alex
    29. Oktober 2018

    @Randerscheinung:
    Ein letztes Mal noch (und dass ich ständig solche Dinge korrigieren muss, ist nicht besonders motivierend):

    Es ist praktisch Kapitel 25 im Fließbach, S. 140-141.

    Nein, das stimmt einfach nicht. Von deinen vier Gleichungen ist nur Gleichung (3) keine Definition o.ä. und kein Ergebnis deinerseits. Und diese Gleichung hat mit der Geodätengleichung nichts zu tun. Anscheinend hast du also diesen Abschnitt im Fließbach nicht verstanden.

    @PDP10:
    Ich weiß nicht welche Stadt du meinst. In Südwestfalen habe ich jedenfalls nicht studiert.

  315. #316 PDP10
    29. Oktober 2018

    @alex:

    Ich habe wegen dem Fließbach gefragt. Ich glaube, der war nirgendwo anders Standard-Lehrbuch …

  316. #317 alex
    29. Oktober 2018

    @PDP10:
    Der Fließbach kam nicht von mir. Ich weiß nicht wie Randerscheinung auf den kam.

  317. #318 PDP10
    29. Oktober 2018

    @alex:

    Oh. Ok. Ich habe zugegebenermassen nicht die ganze Diskussion gelesen.

    Jetzt wundert mich der Teil den ich gelesen habe aber auch nicht mehr ganz so sehr. Da hat jemand mit einem sehr schlechten Grundverständnis von Physik ein schlechtes Lehrbuch gelesen …

  318. #319 Randerscheinung
    29. Oktober 2018

    Ein letztes Mal noch (und dass ich ständig solche Dinge (Versuche, mich als jemanden hinzustellen, der irgendwas einfaches nicht verstanden hat – was im Übrigen die Waffen eines Unterlegenen sind, denn normalerweise streitet man mit Argumenten, nicht mit Diffamierungen) korrigieren muss, ist nicht besonders motivierend):

    Im Fließbach steht auf Seite 140 oben die Geodätengleichung. Dann werden die einzelnen Komponenten mit den Christoffelsymbolen explizit für eine Bewegung im allgemeinen Zentralfeld (=allgemeine, sphärisch symmetrische Metrik) ausgerechnet. Daraus folgt (neben zwei Erhaltungssätzen) die “zentrale Bewegungsgleichung”:

    A (\frac {dr}{d\tau})^2 + r^2 (\frac{d \phi}{d\tau})^2 - Bc^2(\frac{dt}{d \tau})^2 = - \epsilon = -c^2

    Ich dachte, das wäre dir klar. Dort setze ich dann die “Definitionen” aus (1) und (2) ein und erhalte damit (3).

  319. #320 alex
    29. Oktober 2018

    Aus Kommentar #309:

    Das ist nicht “die radiale Bewegungsgleichung der ART”. Das ist die Normierungsbedingung der Vierergeschwindigkeit.

    D.h. du hast etwas einfaches nicht verstanden. Und du hast meinen vorherigen Kommentar, in dem ich dich darauf hingewiesen habe, nicht aufmerksam gelesen.

  320. #321 Randerscheinung
    30. Oktober 2018

    Das sind alles nur Begrifflichkeiten. In dem einen Buch heißt es so, in dem anderen so.

    Wichtig ist: Die Anfangsfrage war: Könnte eine in Zeit und Raum “umgekehrte Krümmung” statt der dunklen Materie für die beobachteten Effekte verantwortlich sein? Und die Antwort ist: ja, natürlich. Man kann sie aus den Messungen berechnen, die äußeren Sterne “tasten sie für uns ab”. Nur – sie ist eben nicht Bestandteil der ART. Und ihre Ursache wäre uns nicht klar.

    Mehr wollte ich ja erst mal gar nicht wissen.

  321. #322 alex
    30. Oktober 2018

    Das sind alles nur Begrifflichkeiten. In dem einen Buch heißt es so, in dem anderen so.

    Nein.

    Könnte eine in Zeit und Raum “umgekehrte Krümmung” statt der dunklen Materie für die beobachteten Effekte verantwortlich sein? Und die Antwort ist: ja, natürlich.

    Nein.

  322. #323 Randerscheinung
    30. Oktober 2018

    Doch.
    ooh (Louis de Funès)

  323. #324 alex
    30. Oktober 2018

    Wie ich dir in schon in grauer Vorzeit gesagt habe, gibt es genau zwei Möglichkeiten:

    Wenn du genau die Metrik postulierst, die man in den üblichen dunkle-Materie-Modellen bekommt, dann kommt damit selbstverständlich die selbe Rotationskurve heraus. Aber das ist absolut trivial. Um das zu erkennen, muss man nicht zwei Monate lang Unsinn posten. Und vor allem ist da nichts “umgekehrt grkrümmt” (was auch immer das heißen soll). Alles ist genau so gekrümmt wie in der gewöhnlichen ART mit gewöhnlicher dunkler Materie.

    Und wenn du eine signifikant andere Metrik postulierst (z.B. eine mit “umgekehrter Krümmung”, was auch immer das bedeuten soll), dann kommt auch eine andere Rotationskurve heraus.

    Doch du möchtest lieber in deiner eigenen Welt leben und ignorierst daher selbst dieses einfache und offensichtliche Gegenargument. Und deshalb ist für mich hier nun endgültig Schluss.

  324. #325 Randerscheinung
    Nürnberg
    15. Mai 2021

    Hi alex, bist du noch da? Ich habs jetzt…

  325. #326 Randerscheinung
    17. Mai 2021

    Hm, schade.

    eben dachte ich noch, ich schreib eine Mail an Dich, @Florian Freistetter und frag mal, was Du von meinem (vielleicht verrückten, aber jetzt vollständigen, in sich geschlossenen und eigentlich sehr schönen) Komplettmodell hältst. Aber mir gefällt Dein Buchtitel des Buches über Newton so gar nicht. Vielleicht war er ja berechtigt ärgerlich. Manchmal freut man sich einfach nur über eine schöne Idee, die man hat (zufällig man selbst… Wenn es doch auch jeder andere hätte sehen können, schließlich ist es ja nun mal da). Und über die darin an die Bildfläche tretende Schönheit des Universums. Und wird dann von allen als hochnäsig, überheblich, größenwahnsinnig, verrückt, eitel und selbstgefällig angesehen. Da kann man auch schon mal böse werden. Völlig verständlich. Und erst recht in der damaligen Zeit, wenn es doch heute immer noch so ist. :-/

    Schade, @alex, dass Du nicht mehr da bist. Ich hätte Dir gern etwas gezeigt. Du wars einer der wenigen, die mitgerechnet und mitgedacht haben. Und immer die Meinung gesagt, wenn eine Idee Quatsch war. Aber eben mitgedacht. Das war extrem hilfreich.

  326. #327 Florian Freistetter
    17. Mai 2021

    @Randerscheinung: “eben dachte ich noch, ich schreib eine Mail an Dich, @Florian Freistetter und frag mal, was Du von meinem (vielleicht verrückten, aber jetzt vollständigen, in sich geschlossenen und eigentlich sehr schönen) Komplettmodell hältst. “

    Wäre vermutlich eh keine gute Idee gewesen: https://scienceblogs.de/astrodicticum-simplex/2019/01/15/warum-ich-wissenschaftliche-arbeiten-von-privatgelehrten-nicht-pruefen-kann/

    “Aber mir gefällt Dein Buchtitel des Buches über Newton so gar nicht.”

    Tja. Es ist erstaunlich, für wie viele Menschen es ein Problem ist, dass jemand wie Newton GLEICHZEITIG ein absolutes Genie UND ein unangenehmer Mensch gewesen ist.

  327. #328 Randerscheinung
    17. Mai 2021

    Ok, stimmt wohl (das es keine gute Idee gewesen wäre).

    Ich finde es nicht erstaunlich, dass es für so viele Menschen ein Problem ist, es macht mir im Gegenteil Hoffnung. Denn niemand wird als Arschloch geboren, davon bin ich überzeugt. Aber wenn man grundlos schlecht behandelt wird, dann werden eben manche auch zu einem Arschloch. Das ist nur die manifestierte Boshaftigkeit, die hinein gegeben wurde.
    Du hättest stattdessen, anstatt ihn so fertig zu machen, Dich auch fragen können, wo/wann er denn so schlecht behandelt worden ist, dass er so wütend wurde.

    Zum Glück ist das ja alles ein paar Jahrhunderte her. Leute, die neue Ideen haben, werden ja heutzutage in Foren und generell immer mit sehr viel Respekt und vor allem überschäumender Freundlichkeit behandelt, zum Glück. Nur in ganz seltenen Ausnahmefällen kommt es mal zu einem bösen Wort.

    Es gibt nämlich eine ganz labile Phase, in der neue Ideen geboren werden, in der man selber nicht genau weiß, ob man verrückt ist oder nicht. Und vor allem eines nicht versteht: Warum die Menschen die logischen Schlüsse, die man geht und die schlicht einfach nur 1+1 zusammenzählen, einfach nicht verstehen wollen. Klar stehen die Schlussfolgerungen im Widerspruch zu irgendwas, sonst wäre es ja jedem bereits klar, aber sie sind trotzdem richtig, weil es ja logische Schlüsse sind. Nur man kann die Widersprüche noch nicht auflösen. Aber nur weil ein Widerspruch entsteht heißt es doch noch lange nicht, dass etwas falsch ist. Man hat nur das Gesamtbild noch nicht verstanden. Sich von einem bestehenden Gesamtbild wegzubewegen (das vielleicht vieles, aber nicht alles erklärt) bedeutet, auch Widersprüche hinzunehmen, anzuerkennen, anzuschauen. Bis dann die Lösung kommt. Die oft überraschend einfach ist. Die dann plötzlich jeder (mit etwas Hilfe bestimmt auch die, die selbst nicht drauf kommen) verstehen kann. Aber die Last des Widerspruchs tragen die auf den Schultern, die den Weg weg von den Standardmodellen gehen. Und dafür für verrückt, hochnäsig und überheblich gehalten werden. Ohne Grund. Sie denken einfach nur selbst. Ein bisschen durch unbekanntes Terrain. Kann man doch machen. Ein bisschen herumdenken. Ist nicht verrückt, und schon gar nicht hochnäsig, nicht überheblich, nicht größenwahnsinnig. Liegt einfach in der Natur des Menschen und macht Spaß. Der Spaß an den Grenzen ist immer am größten: An den Grenzen der eigenen Kräfte, des eigenen Könnens, der Schwerkraft. Ronja soll aufpassen, nicht in den Abgrund zu stürzen. Wie kann sie das am besten tun? Indem sie am Rand des Abgrunds klettert, und das macht sie dann, den ganzen Tag. 🙂

    Wenn jedenfalls in dieser labilen Phase (in der man selbst von den Widersprüchen, die auf einem lasten, gebeutelt ist) einer mit dem Finger auf einen zeigt und sagt, man ist wahnsinnig, dann ist das eine sehr große Verletzung. Und man ist doch so verwundbar. Und da kann man schon mal, dann, wenn man dann alles kapiert hat und es lupenrein und glasklar ist, sauer werden und zum Arschloch. Diese Entwicklung hättest Du bei Newton nachvollziehen können. Sanft, vorsichtig, sein Inneres auspacken. Das hätte ich gern gelesen. Stattdessen zeigst Du mit dem Finger. Finde ich nicht schön. Aber gut, ich habe Dein Buch gar nicht gelesen. Eben weil ich diesen Titel schon so unmöglich finde. Aus oben genannten Gründen. Vielleicht hätte es mich ja gar nicht enttäuscht? Wer weiß.

  328. #329 Florian Freistetter
    17. Mai 2021

    @Randerscheinung: “Du hättest stattdessen, anstatt ihn so fertig zu machen, Dich auch fragen können, wo/wann er denn so schlecht behandelt worden ist, dass er so wütend wurde. “

    Du hättest vielleicht mein Buch lesen sollen, anstatt aus dem Titel abzuleiten, was darin wohl stehen mag um es dann zu kritisieren… (Spoiler: Weder habe ich Newton “fertig gemacht”, sondern sehr explizit beschrieben, wie sehr ich ihn bewundere, noch habe ich ignoriert, was ihn zu dem gemacht hat, was er wurde – das ist genau das, worum es in meinem Buch geht).

    “Das hätte ich gern gelesen. Stattdessen zeigst Du mit dem Finger. Finde ich nicht schön. Aber gut, ich habe Dein Buch gar nicht gelesen.”

    Ernsthaft?! Du schreibst hier nen langen Kommentar und erklärst mir, warum mein Buch schlecht ist. Und am Ende kommst du an und sagst, dass du kein Wort davon gelesen hast? Sei mir nicht böse, aber dann kann ich deine Kommentare dazu nicht mehr ernst nehmen und werde mir erlauben, sie in Zukunft zu ignorieren. Meine Güte…

  329. #330 Randerscheinung
    17. Mai 2021

    OK. Ich bin nicht böse. Aber sei Du mir auch nicht böse: Ich KANN Dein Buch nicht lesen. Bei diesem Titel. Wenn mir einer sagt in dem Hundehaufen dort ist ein Diamant würde ich auch nicht nachgucken.

  330. #331 Florian Freistetter
    17. Mai 2021

    @Randerscheinung: “Ich KANN Dein Buch nicht lesen.”

    Wenn dein Gemüt so empfindlich ist, dass du ein harmloses Wort wie “Arschloch” nicht erträgst, ist das Internet vielleicht generell kein guter Ort für dich…
    Aber tröste dich: Ich habe viel empörte Post bekommen von Leuten, die mir wortreich erklärt haben, warum mein Buch ob des Titels schlecht und unlesbar ist. Wahrscheinlich hätte ich es “Newton: Wie ein eher unsympathischer, unfreundlicher, aber natürlich eh netter, nur ein bisschen missverstandener Mensch das Universum neu entdeckt hat”. Wäre halt nur nicht das gewesen, was Newton war. Es gibt ein Wort, das exakt beschreibt, was für ein Mensch Newton war. Das Wort ist “Arschloch”. Newton war ein Arschloch und er war ein Genie, wie es seitdem und davor kein zweites gegeben hat. Man kann so tun, als wäre Newton kein Arschloch gewesen. Ich wollte aber den echten Mensch beschreiben, nicht eine fiktive Version. Vor allem, weil man durchaus etwas daraus lernen kann, wenn man Newton als Mensch betrachtet und nicht als erhabene Lichtgestalt.

    Übrigens: Neben der “Um Himmels Willen, man darf kein Wort wie “Arschloch” verwenden, das ist urböse und so ein Buch kaufe ich sicher nicht!”-Kritik war der zweite häufige Vorwurf: “Du hast doch nur das so enorm schlimme Wort “Arschloch” verwendet, damit du mehr Bücher verkaufen kannst!!” Entscheidet euch doch mal…
    (Dass ich das Wort verwendet habe, weil es das Wort ist, dass das am besten beschreibt, was ich beschreiben wollte, kommt den Leuten erstaunlich selten in den Sinn).

    So. Und jetzt lassen wir die Diskussion. Wenn du über mein Buch diskutieren willst: Gerne. Aber dann musst du es vorher auch lesen.

  331. #332 Randerscheinung
    1. Juni 2021

    So, @Florian, ich habe meine Meinung geändert. Aus persönlichen Gründen: Ich habe mir grade Dein Buch bestellt und werde es lesen. Vielleicht haben all diese weichgespülten Möchtegern-Revolutionäre auch nur eine Beisshemmung, inclusive mir. Vielleicht muss man ein Arschloch sein um nicht für blöd gehalten zu werden. Seltsame Welt: Es sind doch überwiegend Blöde unter den Arschlöchern.

  332. #333 Karl-Heinz
    Graz
    1. Juni 2021

    @Randerscheinung

    Und wann gibt es eine Buchrezension von Dir? 🙂

  333. #334 stone1
    1. Juni 2021

    @Randerscheinung

    Schon mal etwas von Emotionaler Intelligenz gehört? Die muss nicht unbedingt mir der kognitiven Intelligenz übereinstimmen. Mir fielen einige Beispiele ein, bei denen der IQ und der EQ stark voneinander abweichen, und ich bin mir ziemlich sicher, dass so gut wie jede/r solche Personen in ihrem/seinem Umfeld kennt. Warum sollte es also überwiegend Blöde unter den A…ern geben? Denke eher, dass das relativ ausgewogen sein wird. Auch im umgekehrten Fall.

  334. #335 Florian Freistetter
    2. Juni 2021

    @Randerscheinung: Sag mir danach, wie du es gefunden hast.

  335. #336 stone1
    2. Juni 2021

    @Florian Freistetter

    Ich hab dein Newton-Buch übrigens gelesen, fand es unterhaltsam und gut geschrieben, hab die meisten Details über den großen Physiker zwar schon wieder vergessen, aber es war ein idealer Begleiter für ein, zwei längere Zugfahrten und ein bisschen Vorm-Einschlafen-Lektüre. Ich bin ja (leider) nicht so ein unglaublicher Schnellleser wie Du. : (
    ; )
    Das Sciencebusters-Klimaschönsaufbuch, das ich beim A.S.-Rätsel gewonnen habe, fand ich auch super, da hab ich damals nach der Preisverleihung auch nichts mehr zu geschrieben, nochmal Danke dafür an dieser Stelle!

    Bei der aktuellen Science-Busters-Staffel im ORF bist Du mir bisher irgendwie zu selten untergekommen, deine Kollegen sind aber eh auch gut, mir gefällt dieser Gegensatz und die Dynamik zwischen dir und Prof. Jungwirth mit seinem Woody besonders gut.

    So gut wie damals™ mit Oberhummer und Gruber und MC Puntigam wirds zwar nie mehr werden, aber das ist nur meine Romantisierung der Vergangenheit. ; )

    Sorry für OT, aber im Kontext hats grad hergepasst.

  336. #337 Florian Freistetter
    2. Juni 2021

    @stone: “mir gefällt dieser Gegensatz und die Dynamik zwischen dir und Prof. Jungwirth mit seinem Woody besonders gut. “

    Also in der aktuellen Staffel sind Helmut und ich noch nicht gemeinsam in einer Folge gewesen. Helmut ist mit Lisa Oberzaucher und Peter Weinberger aufgetreten und wenn mich mit einem der beiden verwechselt hast, dann würde ich einen Besuch beim Optiker anraten 😉

  337. #338 stone1
    2. Juni 2021

    @FF

    n der aktuellen Staffel sind Helmut und ich noch nicht gemeinsam in einer Folge gewesen

    Ja eh, aber ich habe ja auch die anderen Staffeln gesehen. ; )